Sei sulla pagina 1di 164

Orthopaedic In-Training Exam 1998

Question 1 -
A patient has a grade 2 open tibial fracture with 70% cross sectional bone
loss that extends 2 cm in length. Treatment consists of stabilization with a
nonreamed interlocking nail and successful secondary wound closure. Four
months after the injury, no callus is visible. Management at this time should
include which of the following?

1. Observation for three months


2. Dynamization of the nail
3. Exchange to a reamed nail
4. Autogeneous bone grafting
5. Electrical stimulation

Answer: 4

Question 2 -
A patient who had been previously treated for multiple injuries as a result
of a motor vehicle accident is now requesting all medical records for unspecified
reasons. A review of the patient record shows numerous office visits, all carrying
a large unpaid balance. With regard to the release of records, the patient should
be informed that

1. Copies of the record can be released to the patient


2. Records can be released only to another physician
3. No records can be released until the balance due is paid
4. Records can be released to the patient for 24hrs, but then must be returned.
5. Because of probable future legal action, records can be released only by court
order.
Answer: 1

Question 3 -
Which of the following findings is the most specific for ankylosing
spondylitis?
1. Positive HLA-B27 antigen
2. Chest expansion limited to less than 2.5cm
3. Generalized ligamentous laxity
4. Positive rheumatoid factor (greater than 1 to 64)
5. The presence of Achilles tendonitis for more than three months

Answer: 2
Question 4 -
A 7-year old boy has a painless abductor limp and limited hip abduction.
Radiographs show varus of the proximal femur, with a narrow upper femoral
epiphysis that has the shape of an inverted "Y". Which of the following factors
best predicts the risk that the deformity will become worse?

1. Length of the femoral neck


2. Retroversion of the upper femur
3. Size of the proximal femoral epiphysis
4. Size of the greater trochanteric apophysis
5. Angle between the proximal femoral physis and Hilgenreiner's line

Answer: 5
Hoffa in 1905 recognized a disturbance in the epiphyses as the cause of coxa
vara deformity. The slip occurs on the metaphyseal side of the epiphyseal plate,
breaking away from the plate at its distal end to separate off a flake (triangular
fragment) of metaphysis (the inverted 'Y'). H-E angles >60 degrees, progression
was documented (thought to be the result of increased bending moment or
shear stress across the vertical defect
Question 5 -
A 17-year old boy who sustained a closed clavicle fracture after he was
ejected from an all-terrain vehicle was treated with a figure-of-8 brace 1 year ago.
He now reports continuous pain at the site of the fracture and is unable to
actively raise his arm above his head. A radiograph is shown in Figure 1.
Management should now include

1. an onlay bone graft


2. electrical stimulation
3. resection of the distal clavicle
4. plate fixation and a bone graft
5. smooth Kirschner wire fixation and a bone graft

Answer: 4
The radiograph illustrates a middle third clavicular fracture with bone loss.
According to Jupiter and associates, the biomechanics of the clavicle predisposes
the middle third to be prone for fracture secondary to both moments of tension
and bending and also torsional forces. In their study, fixation was best accomplish
with plate fixation and a bone graft.[JBJS 1987, 69-A pg. 753-759]
Selection (1) would not provide adequate fixation to promote healing. (2)
Electrical stimulation would not be sufficient for the above reasons. (3) Resection
of the distal clavicle would not be indicate for this case because it promote
further instability of the clavicle and increasing the affected forces to the clavicle.
(5) Kirschner wire fixation with bone graft, the author stated would provide
fixation, but they achieved better results with plate fixation and bone graft
Question 6 -
A 75-year-old woman sustains a fracture below the level of a total hip
prosthesis. Radiographs demonstrate loosening of the prosthetic component.
Treatment should consist of

1. a cast brace
2. a spica cast
3. plate fixation
4. allograft strut fixation
5. long stem revision

Answer: 5
The key to this question lies in the radiographic evidence of loosening of the
prosthetic component. The long stem revision is clearly indicated in this case
because of various factors, one decreases impingement of the loose stem against
the lateral femoral cortex. A non-surgical approach in the elderly patient will only
increase the many risk factors such as atelectasis, pneumonia, and
thromboembolic disease.[Instructional Course 44 pg. 293-303]
Question 7 -
In revision hip arthroplasty, which of the following is the 5- to 10-year
reported graft failure rate when using structural acetabular allografts in the repair
of acetabular deficiencies?

1. 20% with cemented and uncemented sockets


2. 40% with cemented and uncemented sockets
3. 60% with cemented sockets
4. 90% with cemented and uncemented sockets
5. 90% with cemented sockets

Answer: 2
This answer was based on studies by Hooten, Engh. They found that the overall
failure rate was 44 %. Selections 1, 3, 4, and 5 were incorrect. They also reported
an increase failure rate if more than 50% of the cup rested on allograft. When
there is no satisfactory alternative to a bulk allograft available, close radiographic
monitoring was recommended. [JBJS 1994, 76B pg. 419-422.
Question 8 -
What is the treatment of choice for an adult who has an isolated fracture
of the ulna at the junction of the distal and middle thirds, with 5 degrees apex
dorsal angulation and 25% displacement?

1. Intramedullary rodding
2. Functional bracing
3. Closed reduction and a long arm cast
4. Closed reduction and application of an external fixator
5. Open reduction and internal fixation with a dorsal plate

Answer: 2
This is the correct answer for various reasons, based on the question. Key points isolated
fracture, distal and middle thirds, and only 25% displace. The author is implying minimal
displacement. According to Gebuhr, Holmich a fracture such as describe in the question which
does not require close reduction and only initial mobilization are better satisfied with a
functional brace. Their study revealed that elbow extension/flexion and forearm
pronation/supination had no difference with long arm cast, but wrist extension/flexion greatly
improved with the functional bracing. Selections (1) more indicated for midshaft (3) is not
inappropriate, but the authors felt it was not necessary because there was greater patient
satisfaction with functional bracing and same results except wrist motion was better. (4), (5)
are indicated for greater severity of fracture and failed union.
Question 9 -
Physical exam of a high school soccer player who sustains a knee ligament
injury reveals grade III tears of the anterior cruciate and medial collateral
ligaments. In addition, an MRI scan reveals a lateral meniscus injury. Delaying an
anterior cruciate ligament surgery until the patient has full, pain-free range of
motion will decrease the risk of
1. patellar chondromalacia
2. failure of meniscus repair
3. arthrofibrosis
4. varus-valgus instability
5. anteroposterior instability

Answer: 3
Question 10
Spondyloepiphyseal dysplasia congenita and tarda, precocious
osteoarthropathy, and Stickler syndrome are caused by a mutation in the gene
coding for

1. type-I collagen
2. type-II collagen
3. cartilage oligomeric protein (COMP)
4. fibroblast growth factor 2 (FGFR2)
5. fibroblast growth factor 3 (FGFR3)

Answer: 2
Precocious osteoarthropathy is due to a mutation at 12q13.11-q13.2. It causes
type-II collagen disorders. Spondyloepiphyseal dysplasia congenita and tarda are
autosomal dominant and recessive disorders, respectively. They also affect the
gene site for coding for type-II collagen (12q13.11-q13.2). Stickler syndrome, or
hereditary arthro-ophthalmopathy, is secondary to changes in type-II collagen.
Stickler syndrome without eye involvement is due to mutations in type-XI
collagen. Achondroplasia involves a mutation in FGFR3. Alpert syndrome is due
to defects with FGFR2. Ehler-Danlos is an example of type-I collagen mutations.
Type I Multiple epiphyseal dysplasia and Pseudoachondroplasia involves a defect
in cartilage oligomeric protein.
Question 11
Which of the following techniques is most commonly used to classify
tissue type in soft-tissue tumors?

1. karyotyping
2. flow cytometry
3. transmission electron microscopy
4. immunohistochemistry staining
5. scanning electron microscopy

Answer: 4
Microscopy allows for visualization of the cells; however, special staining is
required for identification and classification of specific cells. Karyotyping refers
only to the number and structure of chromosomes in a cell. Flow cytometry is
used to determine the amount of DNA in a cell, and are predictive for the degree
of malignancy. Staining methods are currently the most commonly used method
to classify tissue type.
Question 12 -
An 18-year-old man with hemophilia A has a painful ankle, which is an
isolated target injury. Range of motion of the ankle is from neutral to 15 degrees
of plantar flexion. Radiographs show the ankle joint space to be irregular and
almost obliterated. The remaining joints in the foot appear normal. Which of the
following procedures will most likely provide lasting pain relief?
1. ankle arthrodesis
2. ankle synovectomy
3. total ankle arthroplasty
4. transplant of cartilage cells
5. lengthening of the Achilles tendon

Answer: 1
Question 13 -
A patient who had previously undergone a salvage pelvic (Chiari)
osteotomy now requires a total hip arthroplasty. The most frequent complication
of this procedure is
1. fracture of the acetabulum
2. protrusion of the acetabulum
3. inadequate inferior coverage
4. inadequate superior coverage
5. inadequate anterior and posterior coverage

Answer: 5
The Chiari osteotomy is recommended for patients with inadequate femoral head coverage and an
incongruous joint. The osteotomy shortens the affected leg. It also medializes the hip's center of rotation.
The osteotomy involves cutting the ileum at a spot above the acetabulum, which in effect abducts the
acetabulum into a more vertical and medial position. The iliac wing then serves as a superior buttress.
Answer #1 makes no sense. Answer #2 is wrong because the articular portion of the acetabulum
remains unchanged. Answer #3 is incorrect because inferior coverage remains unchanged. Answer #4 is
completely incorrect because superior coverage INCREASES with a Chiari osteotomy.
Question 14
A patient reports a sudden recurrence of sciatica on her left side after
undergoing a successful left L4-L5 diskectomy 15 months ago. Which of the
following imaging modalities is most accurate in differentiating recurrence of the
disk herniation from postoperative epidural scar tissue?
1. Myelography
2. Thermography
3. CT Scan
4. IV contrast-enhanced CT Scan
5. Gadolinium-enhanced MRI scan

Answer: 5
The main complication of recurrence at the same level is the presence of scar tissue from the
previous operation. In addition to a careful history and physical examination, contrast-
enhanced MRI is useful for differentiating scar from reherniation. MRI has become the test of
choice for assessing degenerative or herniated disks-even over that of CT. Answer #1 is
incorrect because it will only provide information of cord or nerve root encroachment, not the
cause of the encroachment. Answer #2 makes no sense. Answers #3 and #4 are incorrect
because the MRI is now the standard imaging modality.
Question 15 -
A 55 year-old man has multiple lytic lesions in the humeri, clavicles, and
scapulae. Which of the following diagnostic studies best confirms a diagnosis of
multiple myeloma?
1. CT scan of the chest
2. Bone marrow biopsy
3. Complete blood cell count
4. Lateral radiograph of the skull
5. Erythrocyte sedimentation rate

Answer: 2
Myeloma is a malignant bone tumor derived from plasma cells. It is associated with
abnormalities of protein synthesis. It is the most common primary malignant tumor of bone.
The disease is most common between the ages of 50 and 80. Patients most commonly present
with bone pain, usually in the spine and ribs. Biopsy is necessary to establish the diagnosis in a
solitary lesion and is still the gold standard for diagnosis of any lesion. Definitive diagnosis of
multiple myeloma is established by bone marrow aspiration. However, multiple myeloma may
also be diagnosed with considerable confidence based on radiographs and lab data. Bence
Jones proteins may be found in the urine. Serum protein abnormalities may cause formation
of a rouleaux in the peripheral blood smear. Serum and urine protein electrophoresis are
usually abnormal. Answers #3, #4, and #5 may provide some information but the definitive
diagnosis is based on bone marrow aspiration. Answer #1 provides little information.
Question 17 -
Analysis of which of the following proteins is used to establish the
diagnosis of Becker muscular dystrophy?
1. Myosin
2. Troponin
3. Tropomyosin
4. Fibrillin
5. Dystrophin

Answer: 5
Becker muscular dystrophy is an X-linked inherited disorder present in approximately 1 in
30,000 live male births. The responsible gene is located on the xp21 region of the X
chromosome including sixty-five exons that encode for the protein dystrophin. Duchenne
muscular dystrophy is also related to a mutation of the dystrophin gene. Muscle biopsies for
dystrophin have been extremely successful for identifying these dystrophies and distinguishing
them from other clinically similar autosomal recessive myopathies. (Ref: Shapiro, Hoffman)
Question 18 -
A 25-year-old woman with spastic diplegia has a painful progressive
bunion deformity that has failed to respond to nonsurgical treatment.
Examination reveals tenderness and erythema over the bunion prominence;
however, the hallux metatarsophalangeal joint has full range of motion. A
standing AP radiograph shows a hallux valgus angle of 30 degrees and a 1-2
intermetatarsal angle of 13 degrees. Treatment should now consist of
1. arthrodesis of the hallux metatarsophalangeal joint.
2. arthrodesis of the first tarsometatarsal joint.
3. excision of the medial eminence and medial capsular reefing of the
metatarsophalangeal joint.
4. osteotomy of the distal first metatarsal.
5. proximal metatarsal osteotomy with distal soft-tissue realignment.

Answer: 1
Treatment of hallux valgus in a patient with cerebral palsy is largely dependent on the degree of
spasticity and the pattern of gait. The only way to adequately eliminate spastic deforming forces is with
an arthrodesis of the MTP joint. Any other procedure will most likely lead to a high incidence of either
hallux varus or recurrent hallux valgus.
The optimal arthrodesis angle is 25-30 degrees, and the metatarsal inclination angle should be 25-30
degrees also. Sagittal plane position should be checked intraoperatively and the proximal phalanx
should clear the table by 5-10mm with simulated WB
Question 19
A 9-month-old infant with torticollis has failed to gain weight and has
recurrent lower respiratory tract infections. Neurologic examination is normal,
and there is no palpable mass in the sternocleidomastoid muscle. Radiographs of
the neck show no bony anomalies. Management should now include
1. a skeletal survey.
2. audiometric screening.
3. physical therapy for stretching.
4. contrast studies of the upper gastrointestinal tract.
5. reassurance to the parents that there is no underlying pathology.
Answer: 4
Question 20 -
Which of the following radiographic findings would be characteristic of
the knee joints of a patient with neuropathic osteoarthropathy of the knee?
1. Fragmentation and subluxation of the normal joint articulation
2. Varus deformity with medial subchondral sclerosis
3. Preferential narrowing of the medial tibiofemoral compartment
4. Narrowing of the medial, lateral, and patellofemoral compartments
5. Bone proliferation at the patellar tendon and ligament insertion sites

Answer: 1
Neuropathic osteoarthropathy (a.k.a. Charcot joint) develops most often in weight-bearing
joints. The most likely cause is diabetes mellitus, but it is also associated with syphilis, leprosy,
yaws, congenital insensitivity to pain, spina bifida, myelomeningocele, syringomyelia,
aerodystrophic neuropathy, amyloid neuropathy, peripheral neuropathy of alcoholism, spinal
cord injury, peripheral nerve injury, post-transplant neuropathy, and intraarticular steroid
injections.
The loss of sensation to the joint is followed by severe degenerative changes, osteophyte
formation, articular and subchondral fractures, and often calcification of surrounding soft
tissues. In the knee, this is a tricompartmental disease and will not selectively affect one
compartment over another.
Question 21 -
Radiographs of the cervical spine of a 73-year-old man who fell down
stairs reveal cervical spondylosis without evidence of fracture or dislocation. MRI
and CT scans are consistent with the plain radiographs. After 72 hours,
neurologic evaluation reveals intact sensation; however, weakness of the upper
extremities is greater than that of the lower extremities. What is the most likely
diagnosis?
1. Central cord syndrome
2. Anterior cord syndrome
3. Posterior cord syndrome
4. Brown-Sequard syndrome
5. Cervical nerve root injury

Answer: 1
Central cord syndrome is the correct answer. Common in the older patient, sacral sparing,
upper extremity involvement greater than the lower extremity. Functional recovery expected
in 75% of patients. 2-Anterior cord syndrome complete motor deficit; trunk and lower
extremity deep pressure and proprioceptive preserved. 3-Posterior cord syndrome is rare with
loss of deep pressure, deep pain, and proprioception. 4-Brown-Sequard syndrome-
Uncommon-Ipsilateral motor deficit, contralateral pain and temperature deficit. 5-Cervical
nerve root injury- functional impairment of the cervical spine. Symptoms are often acute and
severe, dependent on the level of the lesion. An infraforaminal protrusion may compress only
the spinal root ganglion resulting in severe brachialgia with paresthesia and numbness but
with little or no motor involvement.
Question 22 -
A 23 year old man has a minimally comminuted midshaft fracture of the
femur with 2cm entrance and exit wounds as a result of a low-velocity gunshot.
Definitive management should be
1. Debriding the skin edges and performing plate fixation of the fracture
2. Debriding the skin edges and intramedullary rodding of the fracture
3. Extending the wounds, debriding the bone ends, and applying distal femoral
traction
4. Extending the wounds, debriding the bone ends, and performing plate
fixation of the fracture
5. Extending the wounds, debriding the bone ends, and intramedullary rodding
of the fracture

Answer: 2
There is no clear treatment of these fractures the reference literature reviewed shows that soft-
tissue tracks of low-velocity gunshot wounds are not rendered sterile by the bullet force.
Despite this information, the majority of studies that have followed the healing of fractures
secondary to gunshot wounds reveal a surprising low infection rate.
Question 23 -
Which of the following events is most likely to occur following a complete
transection of a peripheral nerve?
1. The cell body nucleus migrates centrally
2. Schwann cells distal to the transection die
3. Axoplasm in the proximal stump drains out
4. Myelin distal to the transection is phagocytized
5. Cell body protein synthesis decreases for the first 10 to 14 days

Answer: 4
Reference-Within a few hours of injury to a nerve fiber the cell body swells and Nissl granules
disappear. The axon distal to the site of injury rapidly undergoes Wallerian degeneration with
the loss of the axon and breakup of the surrounding myelin.
Question 24 -

A right-handed, 53 year old man reports


pain in the left shoulder following a fall on an abducted externally rotated
shoulder 3 months ago. Examination reveals pain on elevation and tenderness
localized to the anterior aspect of the shoulder. Results of the lift-off test are
inconclusive due to limited internal rotation. Figure 2 shows the T1-weighted
axial image from an MRI-arthrogram. Treatment should include

1. Labral repair
2. acromioplasty
3. excision of the coracoid process
4. an arthroscopic Bankart procedure
5. subscapularis repair

Answer: 5

Subscapularis repair-Traumatic rupture of the tendon of the subscapularis muscle


is caused by forceful hyperextension or external rotation of the adducted arm. A
simple clinical maneuver called the "lift-off test", reliably diagnosed or excluded
clinically relevant rupture of the subscapularis tendon.

Question 25 -

Figures 3a and 3b show the


finger deformity that a 13-year-old girl has had since birth. What is the most likely
diagnosis?
1. Clinodactyly
2. Camptodactyly
3. Symbrachtyly
4. Kirner‘s deformity
5. Digiti minimi adductus

Answer: 1
The figures show a little finger to be incurving towards the ring finger (3a) and no flexion deformity (3b).
This in itself is the definition of Clinodactyly. In addition to this curvature in the radioulnar/frontal plane,
the deformity is one that affects the middle phalanx, usually bilateral, and autosomal dominant. The
definitions of the other distracters are self-explanatory (Staheli, Fundamental of Pediatric Orthopedics,
1998): Camptodactyly: non-painful flexion deformity of the PIP joint (i.e. AP Plane); abnormality of the
insertions of the intrinsics and extrinsics flexor tendons; both hereditary and non-hereditary forms.
Symbrachydactyly: digital absence in the form of transverse efficiency; teratogenic etiology resulting in
absence distal metacarpals with finger nubbins. Kirner‘s deformity: rare progressive deformity of the
terminal phalanx of the small fingers with fingernails a curved and clubbed appearance; usually bilateral
and appears during adolescence. Digiti minimi adductus: no such deformity found mention in the
literature.
Question 26 -
Which of the following provides the most stable fixation for comminuted
fractures of the posterior acetabular wall?
1. Cable
2. Buttress plate
3. Methylmethacrylate
4. Multiple lag screws
5. Multiple Kirschner wires

Answer: 2
According to the first sited article (Goulet et al, JBJS, 1994) ―…comminuted fractures so close to posterior
rim (i.e. comminuted fractures)… are amendable only to stabilization with a plate…a buttress plate
enhances stability of fixation for comminuted fxs of the poster wall of the acetabulum.‖ ―Fixation requires
rigid fixation to prevent loss of fixation resulting in incongruity and instability.‖
According to Rockwood & Green and Browner & Jupiter the key to fixing these fractures is to re-establish
a congruent articular surface. The other four distracters (i.e. cable, methylmethacrylate, multiple lag
screws, and multiple K-wires) may achieve initial stability, but reduction will not be maintained. In
addition, further studies show a higher incidence of mortality and complications with the use of lag
screws and K-wires alone (Browner & Jupiter, Skeletal trauma, 1998)
Question 27 -
What percentage of the maximum aerobic capacity is required for normal
walking with an above-knee amputation?
1. 10%
2. 20%
3. 40%
4. 60%
5. 80%
Answer: 4
Maximum aerobic capacity is related to oxygen consumption and heart rate during walking.
According to the first cited article (Waters et al, JBJS 1976), it was found that above the knee
amputees has the greatest O-2 consumption as well as the highest heart rate during walking.
Question 28 -
What posterior pelvic ring injury is most commonly associated with
neurologic compromise?

1. Sacral fracture lateral to the foramina


2. Sacral fracture medial to the foramina
3. Sacroiliac fracture-dislocation
4. Sacroiliac dislocation
5. Iliac wing fracture

Answer: 2
This question requires one to recall pelvic/sacral anatomy along with classification of sacral
fractures which according to the first cited article ) Denis, Clinical Orthopedics, 1988) include
the following 3 zones: Zone 1-Ala region, lateral to the foraminal line (i.e. distracters #1 and #5);
associated with partial damage to the 5th lumbar root (5.9%). Zone 2-Sacral foramina region,
outside of the sacral canal (i.e. distracters #3 and #4); associated with sciatica (28.4%) but rarely
bladder dysfunction. Zone 3-Central sacral canal (AKA: Transverse Fx) (i.e. correct answer #2);
associated with saddle anesthesia and loss of sphincter tone; neurologic damage (56.7%) and
involvement of bowel, bladder, and sexual dysfunction (76.1%).
Question 29 -
A patient has a noncomminuted displaced fracture of the radial head with
a distal radioulnar dissociation. What is the most appropriate treatment for the
radial head?

1. Allograft Replacement
2. Radioulnar synostosis
3. Excision of the radial head
4. Open reduction and internal fixation
5. Silicone radial head replacement

Answer: 4
This is a rare injury, and it is important to recognize both the proximal and distal concurrent
injuries. In the past, the radial head excision has been the primary form of treatment, but this
has shown poor long-term results; silicone replacement has been fraught with reactive
synovitis. Radial ulnar synostosis is a complication, not a planned course of treatment.
Question 30 -
In a patient with T10-level spinal cord injury, which of the following
prognostic signs most likely suggests functional recovery in the lower extremities?

1. T10 sensory pin-prick level


2. Retained vibratory sensation at the ankles
3. Presence of sacral sparing
4. Retained spontaneous respiratory function
5. Priapism

Answer: 3
Sacral sparing is evidenced by perianal sensation, rectal motor function and great toe flexor
activity. Presence of sacral sparing indicates an incomplete cord injury and the potential of
more function after the resolution of spinal shock. If there is no evidence of spinal cord
function below the level of the injury, including sacral sparing, and the bulbocavernosus reflex
has not returned, no determination can be made regarding the completeness of the lesion.
Question 31 -

Examination of a 45 year old


construction worker who was crushed by falling dirt and buried to midchest level
reveals hemodynamic instability; however, radiographs of the chest are normal,
and results of a diagnostic peritoneal lavage are negative. Despite the
administration of a fluid bolus and packed red blood cells, hemodynamic
instability persists. A radiograph of the pelvis is shown in Figure 4. The next step
in the management should be
1. Application of a pelvic external fixator
2. A pelvic sling
3. Angiography of the pelvis
4. Open reduction and internal fixation
5. Open packing of the pelvic hematoma

Answer: 1
In patients with pelvic fractures who are hemodynamic unstable upon presentation, the initial
management starts with application of external pelvic fixation. If there is no response
(stabilization of vital signs and decreased fluid requirements), pelvic angiography with possible
embolization is the next course of treatment. Open reduction internal fixation can be
performed following General surgical management of associated abdominal injuries. Pelvic
fractures have a high association of retroperitoneal bleeding which may not be necessarily be
picked up from diagnostic peritoneal lavage.
Question 32 -
A 12 month old infant has congenital complete absence of the tibia.
Examination reveals that the femur in the abnormal limb is 3 cm short, with a
normal ipsilateral hip. The patient has an intact fibula, an equinovarus foot with
four rays, and moderate popliteal skin webbing. Management should consist of
1. Syme‘s amputation
2. Arthrodesis of the knee
3. Disarticulation of the knee
4. Centralization of the fibula
5. Prosthetic fitting to accommodate the present deformity
Answer: 3
Congenital longitudinal deficiency of the tibia (tibial hemimelia) is a rare anomaly. It is
characterized by leg length inequality, malrotation of the extremity, and an equinovarus
position of the foot. Currently, the primary treatment option is disarticulation at the knee.
Centralization of the fibula has been performed in the past with poor long term results, which
have required reoperations leading to amputations. The Syme amputation and prosthetic
fitting of the present deformity has led to poor results.
Question 33 -
A 20-year-old football player reports hearing a ―pop‖ in his knee as a result
of a noncontact deceleration injury. Examination 24 hours later reveals a large
effusion. The incidence of a rupture of the anterior cruciate ligament in this
situation is closest to
1. 15%
2. 30%
3. 70%
4. 90%
5. 95%
Answer: 3 ACL injuries often occur in noncontact situations. People report hearing or feeling
a "pop" in the knee at the time of injury up to 81% of the time
Question 34
A 12-year-old child with L4 myelomeningocele who is schedules for foot
surgery has a functioning ventriculoperitoneal shunt and has no history of
allergies. Management should include
1. Use of regional rather than general anesthesia
2. Observation of a latex-avoidance protocol
3. Latex skin allergen testing
4. Premedication with corticosteroids and antihistamines
5. Avoidance of prophylactic antibiotics derived from penicillin

Answer: 2
The high prevalence of latex allergy in patients with myelomeningocoele is thought to result
from a heavy degree of latex exposure throughout life, including closure of the spinal defect,
multiple orthopedic, urologic, and neurologic procedures, and repeat bladder catheterization.
As many as 50% of these patients may have the allergy. Appropriate perioperative
management includes utilization of a latexfree protocol.
Question 35 -
An 8-year-old girl has a supracondylar fracture of the distal humerus. Her
neurovascular status is intact. Radiographs show hyperextension of 10 degrees of
the distal fragment and an angle between the humeral shaft and capitellar physis
(Baumann‘s angle) of 88 degrees. Management should consist of

1. Olecranon pin traction


2. Closed reduction and pin fixation
3. Open reduction and internal fixation
4. Cast immobilization in this position
5. An arteriogram to rule out an occult intimal tear of the brachial artery
Answer: 2
Supracondylar fracture of the humerus accounts for 3 percent of childhood fractures. It is the
commonest fracture of the elbow region in children and accounts for 80% of elbow injuries.
Common complications include cubitus varus, ischemic contracture, and neurovascular lesions.
This question chooses closed reduction with Kwire fixation as the correct method of treatment.
However, the literature discusses olecranon screw traction and open reduction as legitimate
options in treatment. This study recommends olecranon traction for severely displaced
fractures left unstable by closed reduction.
Question 36
A 27-year-old man sustained a nondisplaced fracture of the clavicle. The
fracture is healed; however, the patient has atrophy and absent function of the
clavicular head of the pectoralis major muscle. Which of the labeled structures in
Figure 5 was most likely injured?

1. A

2. B
3. C
4. D
5. E

Answer: 4
This patient has a palsy of the clavicular portion of the pectoralls major. The pectoralis major is
innervated by the medial and lateral pectoral nerves. The lateral pec. nerve innervates the
clavicular portion of the muscle. This branches off the lateral cord of the plexus and runs below
the mid portion of the clavicle.
Reference: Grants Anatomy.

Question 37 -
Item Deleted by AAOS

Question 38 -
Figure 6 shows the clinical photograph of a 3-
year-old boy who started to walk at the age of 10 months and has a gait that is
appropriate for his age. His height is in the 40th percentile for his age.
Management should consist of

1. Follow-up in six months.


2. AP and lateral radiographs.
3. AP and lateral radiographs, and a bone scan.
4. AP and lateral radiographs, and serum levels for ca, ph, and creatinine.
5. AP and lateral radiographs, blood serum levels for calcium, phosphate, and
creatinine, and a 24-hour urine collection for vitamin D metabolites.

Answer: 1
The diagnosis is Developmental Genu Valgum. There are several clues in the question that
suggest that this is a benign process. First they tell you that the child walked at 10 months of
age (within normal limits) which helps rule out several neuromuscular disorders. Second, the
child is three years of age. A moderate amount of ―knock knees‖ is considered physiologic up
to six years of life. Third, the child is in the 40th percentile for his height (again within normal
limits).
The correct treatment for children with physiologic genu valgum up to six years of life is
OBSERVATION, if the condition persists past six years standing AP radiographs and a
metabolic work-up are indicated. Because response (1) is the only answer not including x-rays,
this is the one you should have selected.

Question 39 Item Deleted by AAOS


Question 40 -
Figure 8 shows the AP radiograph of a 38-yea-old woman who has
constant pain in her shoulder as the results of a motor vehicle accident.
Examination reveals marked restriction in external rotation. Which of the
following studies should be ordered to aid in making a diagnosis?
1. MRI scan
2. Bone scan
3. Arthrogram
4. Axillary lateral radiograph
5. CT arthrogram

Answer: 4

Figure eight demonstrates a posterior dislocation. Therefore the simplest, and


best test would be the Axillary lateral. Rockwood and Green (fourth edition)
discusses several radiographic signs that indicate that a posterior dislocations
exists. Absence of the normal elliptical overlap shadow; the vacant glenoid sign;
presence of the Trough line; Loss of profile of the neck of the humerus; and void
in the inferior or superior glenoid fossa. Furthermore the clinical picture of loss of
external rotation should also make one suspect of a posterior dislocation.
Question 41 -
A 39-year-old woman jammed her long finger playing softball 24 hours
ago. She is unable to actively extend the proximal interphalangeal joint; however,
when the joint is brought passively into full extension, she is able to maintain that
position. Management should consist of
1. Open repair of the central slip of the extensor mechanism
2. Open repair of the terminal tendon of the extensor mechanism
3. Closed splinting with the proximal interphalangeal joint
4. Closed splinting with the proximal interphalangeal joint in 30 degrees of
flexion
5. Closed splinting with the proximal interphalangeal joint in 45 degrees of
flexion

Answer: 3
Disruption of the central slip of the extensor tendon at the PIP joint with volar migration of the lateral
bands will result in the so-called boutonniere deformity, which includes loss of extension at the PIP joint
and compensatory hyperextension at the DIP joint. The lesion is most often secondary to closed blunt
trauma with acute forceful flexion at the PIP joint. This produces avulsion of the central slip from its
insertion on the dorsal base of the middle phalanx with or without fracture and/or laceration of the
extensor tendon at its insertion. In closed injuries the characteristic boutonniere deformity may not be
apparent at the time of injury and may not be noted until 10 to 21 days after injury. Two diagnostic tests
that are useful in early recognition of this lesion are: (1) a 15 deg to 20 deg or greater loss of active
extension of the PIP joint when the wrist and MP joint are fully flexed and (2) extravasation of intra-
articular radiopaque dye dorsal and distal to the PIP joint. Weak extension against resistance has also
been noted to be a helpful diagnostic finding. Treatment in acute cases before fixed contractures have
occurred may be achieved by progressively splinting the PIP joint into full extension and at the same time
performing active and passive flexion exercises of the DIP joint. In a closed boutonniere deformity
operative intervention is indicated under two circumstances. (1) when the central slip has been avulsed
with a bone fragment which is lying free over the PIP joint and (2) a long-standing boutonniere
deformity in a young person.
Question 42 -
Posterior cruciate insufficiency diagnosed using the quadriceps active test
is confirmed with tibial translation
1. Anteriorly at 20 to 30 degrees of flexion
2. Anteriorly at 70 to 90 degrees of flexion
3. Posteriorly at 20 to 30 degrees of flexion
4. Posteriorly at 70 to 90 degrees of flexion
5. Anteriorly with the knee in full flexion

Answer: 2
In the quadriceps active drawer test, with the subject supine, the leg is relaxed and supported with the
knee flexed 70 to 90 degrees. The examiner must adequately support the patient‘s thigh so that the
subject‘s muscles are completely relaxed. The patient is then asked to perform a gentle quadriceps
contraction without extending the knee. In the normal knee in 90 degrees of flexion, the patellar
ligament is already slightly posterior, and contraction of the quadriceps does not result in an anterior shift
of the tibia. However if the PCL is ruptured, the tibia sags into a posteriorly subluxed position, and the
patellar ligament is then directed anteriorly. In this situation, contraction of the quadriceps muscle in the
posterior cruciate deficient knee results in an anterior shift of the tibia 2mm or more. The examiner can
visualize this anterior shift of the tibia with the knee is 90 degrees of flexion.
Question 43
Evaluation of an 8-year-old girl for scoliosis reveals a normal gestation,
birth, and family history. Her parents state that she stopped gaining new motor
skills at age 6 months. Examination shows the patient can sit independently, but
she is nonverbal and she makes repetitive hand clapping movements. She has a
30-degree thoracolumbar kyphoscoliosis, and mildly increased tone in the
hamstrings and gastrocnemius-soleus complex. What is the most likely diagnosis?

1. Rett syndrome
2. Cerebral palsy
3. Myotonic dystrophy
4. Fragile-X syndrome
5. Adrenoleukodystrophy

Answer: 1
Rett syndrome is a progressive encephalopathy of unknown etiology observed only in girls, who are
apparently normal physically and mentally until the age of 6-18 months. It is characterized by autism, gait
apraxia, dementia, stereotypical hand movements, loss of hand motor skills, hyperreflexia, spasticity, jerky
trunk ataxia, seizures, and acquired microcephaly. Neurologically, abnormal development starts with
hypotonia and is followed by ataxia and finally spasticity. The orthopaedic aspects of Rett syndrome have
been mentioned only briefly in the literature. They include scoliosis, kyphosis, flexion contractures of the
joints, and bilateral tight heel cords. Scoliosis is the major orthopaedic deformity in Rett syndrome. Eight
of Ten girls in one series had developed scoliosis at the average age of 11 years. All eight girls had C-
shaped thoracolumbar neuromuscular curves with pelvic obliquity. The right thoracolumbar curve was
by far the most common pattern, occurring in seven patients (88%), whereas only one patient (12%) had
a left thoracolumbar curve.
Question 44 -
In a fatigue test, the maximum stress under which the material will not fail,
regardless of how many loading cycles are applied, is defined as

1. Endurance limit
2. Failure stress
3. Critical stress
4. Yield stress
5. Elastic limit

Answer: 1
Fatigue failure occurs with repetitive loading cycles at stress below the ultimate tensile strength. Fatigue
failure depends on the magnitude of the stress and number of cycles. If the stress is less than a
predetermined amount of stress, called the endurance limit, the material may be loaded cyclically an
infinite number of times without breaking.
Question 45 -
A skeletal survey is more accurate than a bone scan for detecting skeletal
involvement in which of the following neoplastic diseases?
1. Ewing‘s sarcoma
2. Osteogenic sarcoma
3. Multiple myeloma
4. Metastatic prostate carcinoma
5. Metastatic breast carcinoma
Answer: 3

Multiple Myeloma lesions are cold on bone scan and because of this a skeletal
survey is more useful.

Question 46
At the time of acetabular revision, retention of well-fixed femoral
components inserted with first-generation cementing technique is most
commonly associated with which of the following factors?
1. Higher subsequent loosening rate of the femoral component
2. Higher subsequent polyethylene wear rate
3. Higher subsequent dislocation rate
4. Higher infection rate
5. Unaltered subsequent survival rate of the femoral component

Answer: 5
Peters et al reviewed the results of 37 hips in which a well fixed femoral component that had originally
been implanted with first generation cementing techniques was left in place at the time of acetabular
revision. There was no significant increase in loosening, infection, dislocation or polyethylene wear. The
predicted rate of survival of the femoral component after revision of only the acetabular component was
88% at 48 months and 78°% after 88 months. This author recommends retaining a well fixed femoral
component that has been implanted with 1st generation cementing techniques during acetabular
revision.
Question 47 -
What is the primary mechanism of wear of polyethylene acetabular
components?
1. Crevice corrosion
2. Oscillatory fretting
3. Oxidative degradation
4. Adhesion and abrasion
5. Fatigue and delamination

Answer: 4
Although previous theories on acetabuIar wear implicated fatigue cracking and delamination which is a
major mode of polywear in knees, the primary mechanism of wear of polyethylene acetabular
components has been shown to be adhesion and abrasion. In an analysis of 128 componenets retrieved
at autopsy or revision surgery, wear appeared to occur mostly at the surface of the components and to
be due to large strain plastic deformation and orientation of the surface layers into fibrils that
subsequently ruptured during multidirectional motion. It was also shown conclusively that 32 mm
displayed significantly more wear (volumetric wear) than with either 22 or 26/28 mm heads ( 1 mm
increase in size increased volumetric wear by 10%). The wear at the articulating surface was
characterized by highly worn polished areas superiorly and less worn areas inferiorly separated by a
ridge. Abrasion was very common, occurring after adhesion and plastic deformation of poly fibrils, and
abrasion secondary to third body wear. As well, wear rates decreased with longer survival of
components, indicating a "wearing in" phenomenon, arguing against oxidative and fatigue wear.
Crevice corrossion = occurs in fatigue cracks with low 02 tension (under screw heads,etc.) Oscillatorry
fretting = cyclical outer surface abrading from small movements. Fatigue and delamination =
predominant in total knees, where stresses are maximum just below the surface of the poly, causing
fatigue over time with susequent delamination. In contrast, hip wear occurs primarily at the surface of
the poly.
Question 48

A 53-year-old man with insulin-dependent diabetes


has the ulcer on his heel shown in Figure 9. Radiographs and an MRI scan are
consistent with osteomyelitis of the calcaeus, contiguous with the ulcer itself.
Arterial flow to the foot is adequate. Management should consist of
1. a total contact cast.
2. partial calcanectomy
3. Syme‘s amputation
4. transtibial amputation.
5. nonweightbearing and IV antibiotics.
Answer: 2
Smith looked at 12 pts (7 pts with DM) with a large ulcer and osteomyelitis of the calcaneus all
treated with a partial calcanectomy. All pts had an ABI >.45, Transcutaneous P02 > 28mmHg,
Albumin level > 3.0 and WBC > 1500. 10 of 12 healed and retained mobility. A total contact cast
could be used in a pt with adequate blood flow, and no osteomyelitis A syme's
amputation(through ankle) requires the use of a healthy plantar soft tissue flap for coverage . A
transtibial (BKA) amputation would be the choice for a failed partial calcanectomy or in a patient
who had an ABI < .45 and who wasn't a candidate for a revasculization procedure.
Nonweightbearing and IV antibiotics would not be adequate in a diabetic pt with osteo of this
extent.
Question 49

A 4-year-old boy sustains the injury shown


in Figures 10a and 10b as a result of being hit and dragged by a car. Wound
closure is best accomplished by

1. Sural artery island flap.


2. Free rectus abdominis flap.
3. Extensor digitorum brevis flap.
4. Staged cross leg flap.
5. Split-thickness skin graft.

Answer: 2
The indications for a free flap coverage for the foot and ankle include loss of skin and
subcutaneous tissue with exposed bone, tendon, and neurovascular structures and, on occasion,
surgical hardware; local options are not available or feasible and coverage is necessary to provide
healing and survival of the underlying structures, prevent infection, and provide a durable and/or
flexible cover. Furthermore, the standard cross-leg flap no longer is used because of significant
problems with positioning and joint stiffness, and because it provides inadequate coverage
material. For the various surfaces of the ankle and dorsum of the foot, very few local tissues are
available to provide coverage when free skin grafting is insufficient. Thus, out of the possible
answers, free rectus abdominis flap is the preferred response.
Question 50 -
A 32-year-old man has swelling of the knee as a result of falling with the
knee flexed and his foot in plantar flexion. A Lachman‘s test reveals an apparent
increase in anterior translation. Passive external tibial rotation at 30 degrees and
90 degrees is equal to the contralateral side, and the quadriceps active test is
positive on the affected side. The neurovascular examination is normal.
Treatment should consist of
1. An anterior cruciate functional knee brace.
2. A physical therapy program.
3. Reconstruction of the posterior cruciate ligament and the posterolateral corner.
4. Reconstruction of the posterior cruciate ligament.
5. Reconstruction of the anterior cruciate ligament.

Answer: 2
The question describes a patient with an isolated PCL injury. The mechanism of most athletic PCL
injuries is a fall on the flexed knee with the foot in plantar flexion. The results of the physical
examination—Lachman‘s test with increased anterior translation and a positive quadriceps active
test—suggests a PCL injury. The passive external tibial rotation at 30 degrees and 90 degrees
being equal to the contralateral side suggest an intact posterolateral corner. In light of these
findings, the patient appears to have an isolated PCL injury. In acute isolated posterior cruciate
ligament tears with less than 10mm of posterior laxity at 90 degrees of flexion, current knowledge
suggests nonoperative treatment that stresses aggressive quadriceps rehabilitation.
Question 51 -
Which of the following is considered an advantage of an unreamed
intramedullary nail over a reamed intramedullary nail?
1. Avoids the risk of marrow emboli
2. Avoids injury to the intramedullary nutrient vessels
3. Results in faster healing of fractures
4. Results in more secure fixation
5. Results in faster regeneration of the endosteal blood supply

Answer: 5
The advantage of an unreamed intramedullary nail is a loose-fitting intramedullary rod.
Placed in the medullary canal, it allows the endosteal circulation to regenerate rapidly
and completely where space has been left between the nail and the endosteal surface.
The endosteal blood vessels are destroyed during introduction of reamed intramedullary
nail.
Question 52
A 4-year-old girl who is undergoing chemotherapy for acute lymphocytic
leukemia sustains a displaced fracture through an osteolytic lesion in the
metaphysis of the distal femur as a result of a fall. Treatment should include
1. Above-knee amputation
2. En bloc resection of the lesion and reconstruction with a bone graft
3. Closed reduction and immobilization in a cast
4. Open reduction and internal fixation, followed by radiation therapy
5. Open reduction, curettage, and cementing of the lesion

Answer: 3
In the article by Gallagher, et al, they note a 12% incidence of pathologic fractures associated
with acute leukemic lesions. The fracture should be treated using standard methods. Thus, the
treatment of displaced fracture through an osteolytic lesion in the metaphysis of the distal femur
in a 4-year-old girl would be closed reduction and immobilization in a cast.
Question 53 -
Which of the following conditions associated with a closed fracture of the
clavicle indicates the need for open reduction and internal fixation?
1. Injury to the subclavian artery
2. Injury to the brachial plexus
3. Segmental fracture
4. 100% displacement
5. Associated displaced surgical neck fracture of the humerus

Answer: 1
Injuries to underlying vascular structures associated with clavicle fractures require exploration and
stabilization. Brachial plexus injuries recover spontaneously in two thirds of patients. Displaced
and segmental fractures may undergo closed reduction. Open treatment of clavicle fractures have
been discouraged secondary to technical difficulties and nonunion. A floating shoulder (displaced
clavicle and scapular neck fractures is an indication for ORIF but not humeral neck fracture.)
Question 54 -
Figure11 shows the shoulder radiograph of a 52-year-old woman who has
severe pain in the shoulder as a result of a bicycle accident 9 months ago.
Management should consist of
1. humeral arthroplasty

2. repair of the rotator cuff


3. closed reduction and immobilization
4. open reduction and immobilization
5. open reduction and early passive motion

Answer: 1
Humeral arthroplasty is indicated when there is a defect equivalent to more than 45 percent of
the articular surface of the humeral head or dislocation that had been present for more than six
months. Posterior dislocations is often missed during initial management. Twenty-five of forty-one
dislocations were diagnosed in less than six months.( Hawkins RJ,JBJS 87)

Question 55
A 47-year-old woman with no history of trauma has had a painful, stiff
shoulder for the past 3 months. Treatment consisting of subacromial injection
and nonsteroidal anti-inflammatory drugs has been ineffective. Her active range
of motion is painful and is limited to 90 degrees of abduction, 60 degrees of
elevation, 30 degrees of external rotation, and internal rotation to the posterior
superior iliac spine. Plain radiographs of the cervical spine and shoulder are
normal. Management at this time should consist of
1. arthroscopic capsular release
2. manipulation under anesthesia
3. a physical therapy program
4. an intra-articular corticosteroid injection
5. administration of high-dose oral corticosteroids

Answer: 3
Idiopathic adhesive capsulitis usually responds to nonoperative therapy or closed manipulation,
but shoulder stiffness due to trauma or surgery may necessitate an arthroscopic or an open-
release procedure. For most patients, a supervised physical therapy program will be successful in
treating adhesive capsulitis.
Question 56 -
The Injury Severity Score (ISS), using point scores from five different body
systems, is a method that aids in predicting the chances of mortality in a patient
with multiple injuries by
1. adding the scores, in all five body systems
2. adding the squares of the scores in the three most severely injured systems
3. doubling the cumulative score for head and chest injuries
4. combining the scores from the most and least injured systems
5. correcting the score in the most severely injured system for age

Answer: 2
The Abbreviated Injury Scale (AIS) is made up of scores from 5 body systems
(head/neck, face, chest, abdomen, extremity/pelvis) graded from 1 minor to 5
critical. The ISS is the sum of the squares of the highest AIS grade in each of the
three most severely injured areas. The AIS pertains to individual injuries. The ISS is
used for multiple injuries. Using the ISS dramatically increased the correlation
between severity of injury and mortality.
Question 57 -

Figure 12 shows the frog-lateral radiograph


of a 45-year-old man who has a painful left hip. What is the most likely diagnosis?
1. traumatic femoral head fracture
2. osteonecrosis
3. osteoarthritis
4. neuropathic joint
5. rheumatoid arthritis

Answer: 2
PHASE V: the crescent sign and articular collapse. The supporting bony architecture may become
sufficiently weakened by continued resorption of trabecular bone and subchondral bone plate
along the reactive interface that the stress of weight-bearing can result in subchondral bone plate
fracture with focal articular cartilage buckling and eventual collapse. This is best seen in the frog-
lateral radiograph.
Question 58
A 30-year-old man has had a slowly enlarging mass on the plantar medial
aspect of the foot for the past 6 months. The mass is now 1 cm in diameter,
adherent to the plantar fascia, and painful with weightbearing. The overlying
skin is mobile. Management at this time should consist of
1. low-dose radiation
2. steroid injection
3. a load-relieving insert and shoe modification
4. complete excision of the mass and the entire plantar fascia
5. wide excision of the mass with a 2 cm margin of normal fascia

Answer: 3
The only reason to treat plantar fibromatosis is to relieve the associated symptoms that often
result from local extension and invasion. An indolent lesion can invade the neurovascular
structures, necessitating operative intervention. Non-operative treatment begins with the
construction of a well molded, padded shoe and an orthosis.
Question 59 -
A 60-year-old man with no history of cancer has a destructive lesion in the
proximal femur. He has a long history of tobacco use, but stopped smoking 5
years ago. A needle biopsy specimen of the lesion shows adenocarcinoma.
Which of the following studies will most likely pinpoint the source of the primary
tumor?
1. CT scan of the chest
2. technetium bone scan
3. bone marrow aspiration
4. serum protein electrophoresis
5. lateral skull radiograph

Answer: 1
Skeletal metastasis of unknown origin - ...plain radiographs of the chest established the diagnosis
of carcinoma of the lung in seventeen patients (43%).......
Unlike skeletal metastasis of known origin - most often breast or prostate - a metastasis of
unknown origin usually originates in the lung or kidney. In the present series, the most common
occult primary site was the lung (63%) and the second most common was the kidney (10%).
Question 60 -
What factor is most commonly associated with malignant transformation
of a giant cell tumor?
1. high-grade histology of the initial tumor
2. multiple local recurrences after curettage
3. previous treatment of the tumor with cryotherapy
4. previous treatment of the tumor with radiation therapy
5. extraosseous extension into two or more adjacent compartments

Answer: 4
Most authors have agreed that radiation therapy should be avoided in the treatment of giant cell
tumor, as there is a high prevalence of sarcomatous degeneration.
Question 61 -
A 69-year-old man has neurogenic claudication after walking one block,
but he does not report significant back pain. Management consisting of exercise,
nonsteroidal anti-inflammatory drugs, and epidural steroids over a period of 6
months has failed to provide relief. Plain radiographs show degenerative changes,
but no abnormality in alignment or instability. CT myelography shows central
and lateral recess at L3-4 and L4-5, and foraminal stenosis bilaterally at L3-4.
Treatment should now include
1. Dorsal rhizotomy and facet joint fusion
2. Multilevel corpectomy and spinal stabilization
3. Central and lateral recess decompression and bilateral foraminotomy
4. Central decompression and facet joint fusion
5. Central decompression, foraminotomy, and spinal fusion from L2 to L5.

Answer: 3
Postoperative vertebral subluxation with pain, restricted movement, and further neural
compression does occur following extensive decompressive laminectomy. Resection of posterior
elements has been regarded as benign, and some degree of forward slipping is not always
associated with an increase in pain. Postoperative instability appears to be predominantly related
to extenuating circumstances, such as a primary neural disorder, rheumatoid arthritis,
degenerative spondylolisthesis, or recurrent severe trauma after decompression. An analysis of
182 patients with extensive decompression involving at least one part of one facet joint showed
that in thirteen of them progressive spondylolisthesis developed postoperatively. Three of the
patients had been treated for a herniated disc and ten, for degenerative spondylolisthesis. When
subluxation did occur, it was within the first few weeks after operation and progressed for as long
as two years. All patients with progression were older than fifty-two-years and were in more pain
than those whom subluxation did not occur. An extensive review of 6000 patients in whom a
wide resection was performed, including hemifacetectomy and either a total facetectomy or
removal of the pars interarticularis, or both, found that only 2 percent of those patients required
fusion or instability. An admonition was given by that author not to remove facet joints or pars
interarticularis areas in patients who are less than thirty-years old, since these patients are most
susceptible to postoperative instability. Older individuals with advanced degenerative changes at
the level of the disc, as well as posteriorily, tolerate extensive laminectomy better. When the disc is
maximally narrowed, no further settling of the space is possible and marginal osteophytes
enhance stability.
Extensive laminectomy in patients with degenerative spondylolisthesis does result in
postoperative instability. In the study by White and Wiltse, further subluxation did occur in 66
percent of patients who were operated on for degenerative spondylolisthesis , whereas it was
observed in only 2 percent of the spondylolisthesis or disc patients who did not have
spondylolisthesis postoperatively. The extent of decompression and facet removal must be limited
in the patient with degenerative spondylolisthesis or a fusion of the transverse processes included
as part of the treatment. Internal fixation devices have been used in these circumstances to
prevent further subluxation while the fusion is consolidating. Wiltse outlined some guidelines for
spinal fusion in spinal stenosis: (1) the patient who is less than sixty years old and had
degenerative spondylolisthesis with a total loss of posterior stability due to removal of the articular
processes (a one-level fusion of the transverse processes); (2) the patient who is less than fifty-five
and had a midline decompression for degenerative spondylolisthesis with facet preservation; and
(3) the patient who is less than fifty years old with isthmic spondylolisthesis, if the posterior
elements have been removed. Simple degenerative spinal stenosis seldom requires a fusion, even
if all posterior stability has been lost. The problem with obtaining a successful spinal fusion is real
and conditions are less than optimum in these instances.
Question 62 -
Item Deleted by AAOS
Question 63 -
A 27-year-old woman has recurrent subluxation and pain in the elbow 1
year after sustaining a right posterolateral dislocation of the elbow with an
associated fracture involving the tip of the coronoid process. What is the most
likely cause of the persistent instability?
1. Inadequate rehabilitation
2. Displacement of the coronoid process fracture
3. Insufficiency of the lateral ulnar collateral ligament
4. Insufficiency of the anterior band of the medial collateral ligament
5. Insufficiency of the posterior band of the medial collateral ligament

Answer: 3
Injury to the Lateral Collateral Ligament Complex. This complex is made up of the radial collateral
ligament, the annular ligament, the accessory collateral ligament, and the lateral ulnar collateral
ligament. The cause and presentation of this injury differs from that of the UCL injury, (the most
common cause), elbow subluxation and sprains, iatrogenic causes following surgery for tennis
elbow (25% of those with failed tennis elbow surgery have a lateral collateral ligament complex
injury) and following radial head excision with inadvertent injury to this complex (especially if
one‘s approach is too posterior) and, rarely, isolated varus stresses.
Indications are recurrent dislocations or subluxations which occur with elbow
extension/supination and are reduced with pronation/flexion. The diagnosis is confirmed by the
―lateral pivot shift of the elbow,‖ which demonstrates posterolateral rotary instability by a
technique by a technique analogous to that of the pivot shift in the ACL-deficient knee. Simple
varus stressing has a high false negative rate. The most significant pathology is insufficiency of the
lateral ulnar collateral ligament which allows the radial head to slip beneath the capitellum in
supination.
Treatment is essentially surgical, except for patients in whom an extension block splint
holding the forearm in pronation may help. The ligament is repaired or, more often,
reconstructed using a technique analogous to UCL reconstruction. The patient is immobilized
from 1 to 5 weeks in an extension block splint (at 30 degrees) in full pronation. At 3 months, full
motion is allowed and from 3 to 6 months strength is enhanced. At 6 months, activity with no
varus stress is allowed as tolerated, and unheeded full activity is permitted at 1 year. The results
are 64% good to excellent. In poor results, recurrent instability is a problem.
ghfhgf
Question 64
An 18-month-old child has a congenital anterolateral bowing of the tibia. The
radiograph shown in Figure 13 reveals increased density with obliteration of the
medullary canal at the apex of the 40-degree bow. Treatment should consist of
1. Osteotomy and intramedullary rod fixation
2. Electrical stimulation
3. Strut-autografing the concavity the tibia
4. A patellar tendon-bearing brace
5. Percutaneous injection of demineralized bone matrix

Answer: 4
Anterolateral Tibial Bowing-Congenital pseudoarthrosis of the tibia is the most common cause of
anterolateral bowing. It is often accompanied by neurofibromatosis (50%-but only 10% of
patients with neurofibromatosis have this disorder). Classification (Boyd) is based on bowing and
the presence of cystic changes, sclerosis, or dysplasia and cystic changes are most common. Early
treatment includes a total contact brace to protect from fractures, intramedullary fixation with
excision of hamartomatous tissue, and autogenous bone grafting (osteosynthesis) for nonhealing
fractures. Vascularized fibular graft or Ilizarov methods should be considered if bracing fails.
Osteotomies and electrical stimulation alone are contraindicated. Amputation (Symes) and
prosthetic fitting are indicated after two or three failed surgical attempts.
Question 65 -
When taken concomitantly with which of the following, erythromycin can
cause an increase in the blood level of
1. digoxin
2. sucralfate
3. clindamycin
4. alcohol
5. neuromuscular blocking agents

Answer: 1
When erythromycin is taken concomitantly with digoxin, carbamazapine, cyclosporine, oral
anticoagulants and theophylline, it can increase their blood levels. Erythromycin has no
significant effect in the blood levels of the other listed agents. (Clindamycin enhances
neuromuscular blocking effects and amphoterecin B may cause digoxin toxicity.)
Question 66 -
What is the most important surface geometry design parameter associated
with decreased contact stress and wear reduction in total knee prostheses?
1. Unrestrained roll-back
2. Unrestrained rotational conformity
3. Medial-Lateral conformity
4. Anteroposterior conformity in flexion
5. Anteroposterior conformity in extension

Answer: 3
The analysis of contact stress as a function of thickness of the polyethylene insert for tibial
components has shown that a thickness of more than 8-10 millimeters should be maintained
when possible. The contact stress in the tibial components was reduced most when the
articulating surfaces were more conforming in the medial-lateral direction. Contact stresses were
much less sensitive to changes in geometry in the anterior-posterior direction.
Question 67 -
A 40-year old man has limited, painful motion in dorsiflexion at the
metatarsophalangeal (MTP) joint of the right great toe, despite nonsurgical
treatment. Radiographs show dorsal and medial osteophytes and minimal
narrowing of the articular space. Treatment should consist of
1. Arthrodesis of the MTP joint
2. A Silastic implant of the MTP joint
3. Resection arthroplasty of the MTP joint
4. Cheilctomy of the MTP joint
5. Osteotomy of the base of the proximal phalanx

Answer: 4
Cheilectomy, the excision of an irregular osseous rim that interferes with motion of a joint was
performed on the distal part of the metatarsal of patients who had hallux rigidus. In this study by
Mann, published in JBJS 1988, they were able to conclude that cheilectomy is a better method of
treatment for hallux rigidus than arthrodesis, resection arthroplasty, or arthroplasty with the use
of a flexible implant. In older adults who present late, with more severe X-Ray changes, Keller
procedure is indicated.
Question 68

Which of the following conditions is typically associated


with the ankle deformity shown in figure 14?
1. Genu varum
2. Tarsal coalition
3. Degenerative ankle arthrosis
4. Osteochondritis dissecans of the talus
5. Hemihypertrophy of the ipsilateral lower extremity

Answer: 2
Figure 14 depicts an X-ray of a child with a Ball-and-Socket ankle. This is a rare deformity
associated with such pre-existing conditions as congenital shortening of the lower limb, coalition
of tarsal bones, absent digital rays and aplasia or hypoplasia of the fibula. Ball-and-socket joint is
not congenital, but is an acquired deformity formed in compensation for the loss of inversion and
eversion caused by tarsal coalition or various other pre-existing congenital conditions.
Question 69 -
Crush fractures of the vertebral body are a particularly common problem
in type 1 (postmenopausal) osteoporosis because
1. Trabecular bone is preferentially resorbed in this high bone turnover state
2. Loss of water content in the disk increases impact load to the vetrebral bodies
3. Stress is imposed by the relative stiffness of the arthrtic facet joints
4. Increased energy demands are imposed by decreased circulation to the
vertebral body
5. The thick cortical bone found in the vertebral body resorbs rapidly following
estrogen withdrawal

Answer: 1
The spine is composed primarily of trabecular bone, compared with cortical bone, it has a high
surface-to-volume ratio. Because metabolic activity (remodeling) occurs on bone surfaces,
trabecular bones in general and the vertebral bodies in particular are resorbed preferentially in
times of skeletal loss. Osteoporosis is characterized by trabeculae of decreased size and number. It
has also been demonstrated in osteoporosis that there is a thinning of the cortex as well as a
change in the shape of the trabecular bone from plates to narrow bars. In the phases of bone loss,
vertebral body density declines before a similar loss is detected in cortical areas. The body
accommodates bone loss by redistribution. A 10 percent shift of bone mass outward from the
epicenter through an enlargement of the bone diameter will compensate for 30 percent decrease
in the bone mass against applied bending and torque stresses but not against axial loading. This
differential resorption explains the timing and patterns of the fracture syndromes seen in
osteoporosis. The incidence of vertebral crush fractures rises immediately after menopause,
secondary to high trabecular content and estrogen deficiency. (precise mechanism of effects of
estrogen withdrawal unclear)
Question 70
Which of the following changes in the parameters of the gait cycle occurs
in the transition from normal walking to running?
1. Increased time in stance and swing phase
2. Addition of a double leg float phase
3. Decreased vertical ground reaction forces
4. Decreased arc of motion in the hip, knee, and ankle
5. Decreased joint reaction forces in the hip, knee, and ankle

Answer: 2
The same basic mechanisms that have been described for the biomechanics of the foot and ankle
are not significantly altered during running. The major differences observed during running are
that the gait cycle is altered considerably; the amount of force generated, as measured by force
plated data, is markedly increased; the range of motion of the joints of the lower extremities is
increased; and the phasic activity of the muscles of the lower extremities is altered. During
walking one foot is always in contact with the ground; as the speed of gait increases a float phase
incorporated into the gait cycle, during which time both feet are off the ground. There also is no
longer a period of double limb support. As the speed of gait continues to increase, the time the
foot spends on the ground, both in real time and in percentage of cycle, decreases considerably.
Question 71 -
Examination of a 5-year-old child who has fibular hemimelia reveals the
foot has two rays and is stiff in equinus and valgus. The level of the foot is just
proximal to the midshaft of the contralateral tibia, and the knee has full active
flexion and extension, but slight valgus. Treatment should include
1. Amputation through the midshaft of the tibia
2. Correction of the foot deformity and lengthening with a ring fixator
3. Knee disarticulation
4. Ankle disarticulation
5. Ankle disarticulation and contralateral epiphyseodesis of the proximal tibia
Answer: 4
Congenital absence of the fibula, also called fibula hemimelia, has several manifestations. These
range from complete absence of the fibula with missing lateral rays of the foot (i.e. terminal
longitudinal deficiency) to absence of only a portion of the fibula without foot involvement (i.e.
intercalary longitudinal; deficiency). Congenital fibular deficiency usually occurs sporadically
without a known cause. The child with complete absence of the fibula presents clinically with an
anterolateral bow of the tibia, an equinovalgus deformity of the foot, and a tarsal coalition. The
talotibial joint is usually malformed, with the fused talocalcaneus having a flat upper surface that
articulates with the tibia in a valgus and equinus position. The foot may be missing one or two
lateral rays. There is always significant shortening of the epsilateral femur. Treatment: There is a
reasonable consensus that complete fibular hemimelia is best treated by performing an ankle
disarticulation in early childhood and fitting a Syme-type prosthesis. Children with lesser (<5cm) at
birth may be a candidate for lengthening procedures, but the exact indications and results of
these procedures have not been well defined. Children with large discrepancies (>5cm) at birth
and those with major foot deformities are better managed with amputation.
Question 72 -
A 67-year-old woman has chronic left foot pain that has failed to improve
despite appropriate brace and shoe modifications. Examination reveals the
absence of single limb heel rise on the affected side, an obvious unilateral flatfoot
deformity, loss of posterior tibial tendon function, mild subfibular tenderness, and
fixed hindfoot valgus. The best surgical option available for this patient is
1. Talonavicular arthrodesis
2. Medial displacement calcaneal osteotomy
3. Flexor digitorum longus tendon transfer with spring ligament advancement
4. Triple arthrodesis
5. Calcaneocuboid distraction arthrodesis and repair of the posterior tibial tendon
Answer: 4
Rupture of the posterior tendon is common and occurs past the age of 41. Symptoms are chronic
and insidious. Invariably a history of congenital pes planus is obtained. Patients notice one of the
arches gradually becomes flatter and is associated with swelling and discomfort over medial
aspect of ankle. Exam reveals excessive valgus of the involved foot. Inversion is weak and it is
impossible to stand tip toe on affected foot. Tibialis posterior degeneration is classified by the
severity of progressive pathologic changes and surgical management is based upon stage of
deterioration and degree of peritalar subluxation.
Stage IA (mild, occult)-mild symptoms for 4 months to 1 year. Minimal swelling and mild
tenderness about medial malleolus. Slight weak inversion power and very minimal valgus. Surgery:
repair splits and plicate tibialis posterior if elongated. Do side to side anastomosis of tibialis
posterior to flexor digitorum longus above and below degeneration.
Stage IB (moderate0symptoms present 1-1.5 years. Clinical findings more apparent with
definite tibialis posterior tenderness, swelling, weakness and moderate pes planus and heel
valgus. Surgery: Same as for IA with routine plication of tibialis posterior.
Stage II (advanced)-symptoms present 1.5-2.5 years. Flatfoot deformity pronounced. Considerable
heel valgus and moderate prominence of head of talus medially on weight bearing. Valgus
deformity essentially fixed. Surgery: release tibialis posterior from adhesions, plicate elongated
section and anastomose tibialis posterior to flexor digitorum longus as in stage I. If tibialis
posterior too thinned out, augment by free extensor digitorum longus grafts or a triple
arthrodesis.
Stage IIIA (complete)-symptoms 2.5-4 years. Valgus progressed further and increasing
peritalar subluxation with more obvious prominence of talar head medially on weight bearing .
Surgery: all cases triple arthrodesis with iliac bone graft indicated.
Stage IIIB (complete with peritalar dislocation)-symptoms over 4 years some up to 20
years. Valgus is severe with increasing peritalar subluxation up to point of dislocation. Calcaneus
is rotated severely into valgus and displaced literally. Talar head completely dislocates medially out
of navicular and bears considerable weight indicated by callus on underlying skin. Pain becomes
predominantly lateral on weight bearing owing to abutment of superior edge of distal calcancus
against undersurface of talar neck in sinus tarsus. Surgery: triple arthrodesis with lilac graft. Any
lateral abutment projections are excised and used for bone graft.
Question 73
Figures 15a and 15b show the AP and
lateral radiographs of the lumbar spine of a 51 year old woman who has had
back pain that radiates into the right thigh for the past 3 months. Her medical
history is unremarkable except for a mastectomy for breast cancer 12 years ago.
What is the most likely diagnosis?
1. Lymphoma
2. Hemangioma
3. Osteosarcoma
4. TB of the spine
5. Metastatic breast carcinoma

Answer: 5
Metastatic disease of the spine occurs in as many as 70% of patients with disseminated cancer
and may result in vertebral collapse, spinal instability, and progressive neurologic compromise.
Three fourths of these originate from breast, prostate, kidney, or lung carcinoma or myeloma or
lymphoma. The vertebral body is affected due to a rich blood supply and sinusoidal vascular
distribution. Cord compression is the extrusion of tumor tissue and detritus of bone or disk in the
spinal canal following the partial collapse of a vertebral body that has been infiltrated and
weakened by a metastatic deposit.
Question 74 -
A patient sustained a joint depression-type fracture of the calcaneus that
healed despite lack of treatment. The loss of dorsiflexion the patient is now
experiencing is most likely the result of
1. widening and shortening of the heel.
2. weakness of the gastrocnemius-soleus complex.
3. anterior impingement from a horizontal talus.
4. unrecognized compartment syndrome of the foot.
5. degenerative arthritis of the tibiotalar joint.

Answer: 3
Closed treatment is geared toward padding the heel externally. The horizontal talus may abut the
tibia anteriorly. This is caused by a joint depression type fracture. The primary fracture line begins
in the sinus tarsi and propagates obliquely across the posterior facet to the medial wall. The
posterior facet is no longer under the talus and the talus settles into a position parallel to the
ground. When the talus is parallel to the floor, it is often fully dorsiflexed, even with the foot in
neutral position. The changes in the subtalar joint also affects the transverse tarsal joint. Because
of the relative positions of the calcaneus at the cuboid (neutral) and the talus at the navicular
(dorsiflexed) are different, this has the net effect of locking the transverse tarsal joint.
Question 75
A radiograph of a 27 month old child with bilateral genu valgum and
internal tibial torsion shows the metaphyseal-diaphyseal angle of Levine and
Drennen is 12 degrees on the right and 13 degrees on the left. Based on this
finding, management should consist of
1. a corrective osteotomy
2. application of braces
3. medial physeal stapling until the varus corrects
4. observation
5. application of corrective casts

Answer: 4
This paper showed that the older the child was at the time of presentation the more likely it was
that the angle would be smaller in a child who had physiological bowing and larger in a child
who had Blount disease. Physiologic bowing is common in children who are less than 3 years old
whereas Blount disease is reported to be less than 1% at this age. Corrective bracing is initiated
for presumed early Blount disease only is the metaphyseal-diaphyseal angle is more than 16
degrees. If the angle is less than 9 degrees the patient is observed. Between 9 and 16 degrees,
bracing is considered only if there is instability on walking. The patient is then evaluated on 4
month intervals.

Question 76 -
A 50 year old man who has insulin-dependent diabetes mellitus with
associated neuropathy has a stress fracture of the fourth metatarsal. Examination
of the foot reveals acute swelling, warmth, and erythema; however, the patient
reports very little pain. Treatment should include
1. a total contact cast.
2. electrical stimulation.
3. an off the shelf fracture brace.
4. an elastic compression bandage and crutches.
5. a hard soled shoe until the patient is asymptomatic.

Answer: 1
The approach to the diabetic foot is a challenging problem. The team approach has taken
various forms and venues with the basic principles being the same: education, prevention, follow
up and aggressive care. Custom shoes, custom-molded inserts, and increased depth shoes are
being used in treatment as well as prevention. Brand documented that repeated low-energy
trauma is the cause of most skin breakdown with impaired sensation. Responses 3 and 5 can be
eliminated since they are not custom. Compression bandage can cause minor trauma and
ischemic problems if sensation is altered. Electrical stimulation is not used for acute fractures. A
total contact cast will relieve pressure areas. This should be monitored closely since swelling will
decrease and with movement in the cast, minor trauma may occur.
Question 77 -
Which of the following types of sarcoma of the bone is most sensitive to
external beam radiation?
1. Ewings tumor
2. Parosteal osteosarcoma
3. Dedifferentiated chondrosarcoma
4. Low grade intramedullary chondrosarcoma
5. High grade intramedullary osteosarcoma
Answer: 1
Parosteal osteosarcoma occurs on the surface of the metaphyseal regions of the distal femur or
the proximal humerus most commonly. The treatment is wide surgical resection versus limb
salvage. Dedifferentiated chondrosarcoma has a moth eaten appearance and may occur as a
transformation of chondrosarcoma. Treatment is resection and prognosis is poor. Low-grade
intramedullary chondrosarcoma is also treated with surgical resection. High-grade intramedullary
osteosarcoma is usually treated with pre-op chemo and resection. The only tumor listed where
radiation is an option is Ewings tumor.
Question 78 -
Which of the following injuries is most commonly associated with a fracture of
the scapular body?
1. Vascular injury
2. Tear of the rotator cuff
3. Injury to the brachial plexus
4. Fracture of the upper thoracic rib
5. Fracture of the proximal humerus

Answer: 4
Ada and Miller reviewed 148 fractures in 113 scapulae. Ninety-six percent had
associated injuries, the most common being fracture of an upper thoracic rib.
Other associated injuries included lung trauma, head injury, cervical spine injury,
clavicle fractures and brachial plexus injury.
Question 79 -
Which of the following shoulder girdle muscles is most active during
forward flexion?
1. Biceps
2. Trapezius
3. Infraspinatus
4. Pectoralis major
5. Serratus anterior

Answer: 5
The percentage of recorded EMG activity indicates the level of activity of a given muscle but does
not indicate the force generated. During forward elevation, the upper portion of the trapezius,
levator scapulae, and serratus anterior contract to produce a scapular rotating force upward,
increasing the stability of the glenohumeral joint. The essential muscles for forward elevation are
the deltoid, the supraspinatus, the trapezius and the serratus anterior. EMG reveals the serratus to
be most active.
Question 80 -
Figure 16 shoes the AP radiograph of a 32-year-old man with a fracture
cephalad to the fovea of the femoral head. A CT scan shows a single head
fragment. After closed reduction of the hip, there is 5 mm of residual articular
incongruity. Management should now include

1. Hybrid total hip arthroplasty


2. Noncemental hemiarthroplasty of the hip
3. Closed reduction and percutaneous pin fixation
4. Open reduction through an anterior approach to the hip
5. Excision of the head fragment

Answer: 4
There seems to be no controversy in treating Pipkin 3 and 4 femoral head fractures. These are
treated with an ORIF in concert with the femoral neck fracture or the acetabular fracture
respectively. The difficulty stems in how to treat types land 2. In their review the authors
recommend that residual joint incongruity of >2mm or an unstable hip requires and ORIF; their
preferred approach was anterior. Swiontkowski Thorp Hansen Operative management of
displaced femoral head fractures. J Orthopaedic Trauma. 1992 Vol 6 No 4; 437442
Question 81 -
Figure 17a shows the postoperative AP hip radiograph of a 35-year-old
woman who sustained an isolated fracture of the femoral neck while skiing 7
months ago. Treatment consisted of open reduction and screw fixation. She now
reports continuous pain in the groin and an inability to bear weight. AP and
lateral radiographs shown in Figures 17b and 17c reveal no evidence of healing
of the fracture. Management at this time should consist of

1. a quadratus femoris pedicle


bone graft
2. a proximal femoral allograft
3. intertrochanteric osteotomy
4. total hip arthroplasty
5. hip hemiarthroplasty

Answer: 3
Question 83 -
Examination of a 32-year old woman who has pain in her shoulder as a
result of a head-on motor vehicle accident reveals tenderness directly over the
scapula and painful motion of the shoulder. Radiographs show a displaced extra-
articular fracture of the scapula. Which of the following studies would best
detect commonly associated injuries?
1. Echocardiogram
2. Electrocardiogram
3. Radiograph of the chest
4. CT scan of the shoulder
5. Ultrasound of the shoulder

Answer: 3
Ninety-six percent of patients with scapular fractures has associated injuries, with rib fractures in
the upper thorax being the most common. Pulmonary injuries were second in frequency (37%)
with hemopneumothorax (29%) and pulmonary contusion (8%). Head injury was third (34%)
and there were nine skull fractures. Clavicle fractures on the ipsilateral side occurred in 25%. The
most frequent level of spinal cord injury was cervical (12%). Four patients suffered a permanent
cord injury: two quadriplegics, one paraplegic, and one Brown-Sequard Syndrome. There were
four brachial plexus injuries. Three recovered and the one with a persistent deficit also had a
reflex sympathetic dystrophy. His injury was caused by a self-inflicted shotgun blast. Radiograph
of the chest would provide the best overall survey for evaluation. The remaining studies would
only evaluate isolated areas.
Question 84 -
Which of the following surgical approaches to the hip is associated with
the highest incidence of heterotopic ossification?
1. Ilioinguinal
2. Extended iliofemoral
3. Combined ilioinguinal and Kocher-Langenbeck (posterior)
4. Kocher-Langenbeck (posterior)
5. Kocher-Langenbeck (posterior) with trochanteric osteotomy

Answer: 2
Significant extopic bone formation results from a combination of initial trauma to the gluteal
muscle mass and surgical exposure of the lateral surface of the pelvis. Extensile (extended
iliofemoral or triradiate) approaches are associated with the highest incidence of ectopic bone
formation, whereas the ilioinguinal approach is rarely associated with this complication. Many of
the fractures described in this chapter require a posterolateral or extensile approach in order to
achieve acceptable fracture reduction. When these approaches must be used, local measures
may be helpful in reducing the incidence of heterotopic ossification. Debridement of devitalized
muscle, particularly the gluteus minimus, has been shown to limit the extent of ectopic bone
formation.
Prevention:
1. incision choice: ilioinguinal if possible
2. radiation
3. indocin (give pepcid with it)
Question 85

A 25-year-old volleyball player reports


pain and clicking in his dominant shoulder during overhand serving. Three
months of physical therapy fail to provide relief. Radiographs are normal, and an
MRI scan is shown in figures 18a and 18b. Atrophy and weakness are most likely
to be localized to which of the following muscles?
1. Deltoid
2. Supraspinatus
3. Subscapularis Infraspinatus
4. Infraspinatus
5. Infraspinatus and teres minor

Answer: 4
The MRI of the shoulder shows multiple ganglion type cysts of the genoid labrum. These cyst
have a correlation with overhead type repeative motion. It has been suggested in the volleyball
players that the rapid deceleration after a spike can lead to a SLAP(superior labral) lesion. This in
turn can lead to genoid cyst formation. Now remember that the suprascapular nerve comes off
the superior trunk of the Brachial plexus, goes under the superortransverse scapular ligament (in
the scapular notch, nerve under artery above). It then descends right behind the posterior
glenoid/labrum. Therefore, a large cyst in this area will impinge/entrap the nerve. This nerve
supplies the infraspinatus muscle and over time will give you atrophy/ pain of this muscle. 87.
Question 86
Figure 19 shows the radiograph of an 18-year-old recreational soccer
player who has had pain in the lateral foot for the past 4 weeks. He reports no
specific injury and has not undergone any treatment. Initial management should
consist of

1. an orthosis.
2. observation.
3. electrical stimulation.
4. open reduction and internal fixation.
5. application of a nonweightbearing short leg cast.

Answer: 5
This is a basic knowledge question about Jones fractures of the 5th metatarsal. First you have to
recognize that this is a Jones fx and not a styloid fx, which occurs at the watersheld area (or
metaphyseal flare) of the proximal 5th metatarsal. The treatment is 46 weeks of cast
immobilization and nonwieghtbearing. If healing does not occur, then operative intervention is
necessary, which is an intramedullary lag screw fixation.
Question 87

Figure 20 shows the plain radiograph of a 70-year-old


woman who has shoulder pain and is unable to reach above chest level as a
result of a fall 3 months ago. An MRI scan of the shoulder shows a large rotator
cuff tear. Examination reveals atrophy of the infraspinatus muscle, active forward
elevation of 40 degrees, active external rotation of 30 degrees, passive forward
elevation of 150 degrees, and passive external rotation of 60 degrees. The patient
has no external rotation strength against resistance. Treatment should include
1. repair of the rotator cuff.
2. rehabilitation of the shoulder
3. replacement of the humeral head.
4. arthroscopic acromioplasty and debridement.
5. immobilization is a sling until pain resolves.

Answer: 2
This defines a 70y/o lady who 3 months ago sustained a large, to massive rotator cuff tear, not
only by MRI, but by physical exam as well. In any age group or duration from injury, massive
rotator cuff tears do poorly with surgical intervention. Now add in 3months duration and 70 yr
age and boy doesn't rehabilitation sound good.
Question 88 -
Which of the following factors is used to determine torsional rigidity of a
long bone fracture under internal or external fixation?
1. Bone rotation versus torque applied
2. Bone deflection versus bending moment applied
3. Axial displacement versus tension applied
4. Lateral translation versus shear force applied
5. Fracture gap closing versus compressive force applied

Answer: 1
Torque is defined as: T=r x F, where r is the moment arm and F is the force applied. The moment
arm is the perpendicular distance from the line of action or axis of rotation. Thus torque is a vector
quantity having a magnitude and direction. Torsion involves shear and tensile stresses that cause
deformation. Thus torsional rigidity is related to bone rotation and the torque applied to it.
Question 89
A 33-year-old woman reports a 3-month history of pain in both feet while
running. Examination reveals bilateral point tenderness over the plantar fascia at
its origin, and the pain is accentuated when the ankle is dorsiflexed.
Management should consist of
1. steroid injection
2. stretching of the heel cord
3. surgical release of the plantar fascia
4. application of a short leg cast for 6 to 8 weeks
5. wearing dorsiflexion night splints

Answer: 2
This question refers to plantar fascitis. Heel spurs are noted in approximately 50% of the cases of
subcalcaneal pain syndrome. In this patient, diagnosis should rule out lumbar radiculopathy since the
symptoms are bilateral.
The most common site for heel pain is where the plantar fascia and intrinsic muscles arise from the medial
calcaneal tuberosity on the anteromedial aspect of the heel.
First line treatment is NSAID‘s, Physical therapy involving heel cord stretching and an orthosis. Second line
therapy after these treatments are unsuccessful involve steroid injection and plaster immobilization. Surgical
intervention should be the very last choice in the options given.
Question 90 -
Radiographs of a 10-year-old boy who has a painless limp show widening
of the symphysis pubis and bilateral coxa vara. Examination reveals frontal
bossing, limited hip abduction, scoliosis, a bilateral Trendelenburg gait, and
height in the 20th percentile for his age. Which of the following conditions is also
likely to be present?
1. Open bladder
2. Bilateral ―hitchhiker‘s‖ thumbs
3. Bilateral defects in the midclavicles
4. Rhizomelic shortening of the extremities
5. Radiographic fragmentation of all major epiphyses
Answer: 3
In this question, the $ referred to is Cleidocranial Dysplasia which is an AD involving primarily
bones formed by intramembraneous ossification. (i.e. facial bones, cranium and clavicles) CP
include diminished stature between the 5th and 50th percentile, craniofacial abnormalities
manifested by bipariatal cranial enlargement, midface hypoplasia, frontal bossing, high arched
palate and nonexistent nasal bone development. These patients have absence of all, or a
significant portion of the clavicle. Many can completely oppose their shoulders anteriorly. Pectus
excavatum is also common.
PXR: clavicle abnormalities , scoliosis ±, hemivertebrae, hypoplasia of the iliac crest, coxa vara,
ossified femoral epiphysis at birth. Surgical intervention is usually only required for treatment of
coxa vara which may need a intertrochanteric valgus osteotomy for severe cases.
-―Hitchhiker‘s‖ thumbs is seen in—Diastrophic dysplasia.
-Rhizomelic shortening of the extremities is seen in both Achondroplasia and
Pseudoachondroplasia.
-Radiographic fragmentation of all major epiphyses is seen in Multiple epiphyseal dysplasia.
Question 91
Which of the following ligaments provides the major static restraint to
lateral patellar displacement?
1. Medial patellotibial
2. Medial patellofemoral
3. Medial patellomeniscal
4. Lateral patellofemoral
5. Lateral patellotibial

Answer: 2
The medial patellofemoral ligament is found to arise from the adductor tubercle and pass deep to
the VMO and inserts on the proximal aspect of the medial patella and on the undersurface of the
distal aspect of the quadriceps mechanism. The ligament varies in size in each patient but is the
major soft tissue restraint to lateral displacement of the patella. Conlin and Garth, et al. found that
the medial patellofemoral ligament contributed 53% of the total force against lateral
displacement of the patella.
The medial patellotibial band was found to be functionally unimportant and the medial
patellomeniscal ligament was found to contribute 22% to the lateral displacement force.
Question 92 -
Which of the following rehabilitation methods should be used for the first
24 hours following a blunt injury to the quadriceps musculature to avoid short-
term stiffness?
1. Heat
2. Gentle active flexion-extension exercises
3. Isokinetic strengthening
4. Electrical muscle stimulation
5. Immobilization of the limb with the knee in full flexion

Answer: 5
A West Point study utilizing a three-phase protocol after quads contusion was cited. Phase I was
to limit hemorrhage. Rest, ice, compression and elevation were used for 24 to 48 hours
depending on the severity of the contusion. Rest involved ace wrap to entire leg and hip and
knee flexed to tolerance. When the patient was pain free at rest and thigh girth had stabilized
Phase II had begun. The purpose of this phase was to restore ROM. Ice and cool whirlpool were
continued, gravity assisted motion and active flexion and extension exercises are started.
Weightbearing to tolerance in continued and crutches are discontinued when 90 degrees of
motion, no limp and good quad control is attained. Phase III starts when there is 120 degrees of
pain free active motion and participation in noncontact sports is allowed, when full strength,
motion and endurance is achieved contact sports can be resumed. A thigh pad is worn for 3-6
months.
In the past immobilization in full extension was recommended, but it was noticed that the lack of
flexion prolonged disability. Flexion of the knee during the first 24 hours also aids in limiting the
extent of intramuscular hematoma.
Myositis ossificans is higher in any patient presenting after a quad contusion and has active knee
ROM of less than 120 degrees and delay in treatment greater than 3 days.
Question 93 -
Which of the following procedures is associated with the highest risk of
osteonecrosis of the metatarsal head?
1. Distal chevron osteotomy with soft-tissue release
2. Distal soft-tissue realignment only
3. Closing wedge osteotomy (Aken) of the proximal phalanx
4. Proximal first metatarsal osteotomy only
5. Soft-tissue realignment with a proximal metatarsal osteotomy
Answer: 1
1 – Complications of distal chevron osteotomy with soft tissue release are incomplete correction
and avascular necrosis (1 – 2%). 2 – Complications of distal soft tissue realignment only are
recurrence of deformity, inadequate lateral release, are hallux varus. 3 – Complications of the
Aken procedure are an increase in the hallux valgus deformity. 4 ,5 – Complications of proximal
first metatarsal osteotomy are hallux varus and shortening.

Question 94
A patient reports changes in vocal quality after undergoing a right-sided
anterior cervical approach to C6. Which of the following nerves has most likely
been injured?
1. Vagus
2. Phrenic
3. Hypoglossal
4. Recurrent laryngeal
5. Inferior thyroid

Answer: 4
Discussion: Generally, a left sided approach to the anterior cervical region is preferred because of
the more constant anatomy of the recurrent laryngeal nerve, which results in changes in vocal
quality.
Question 95 -
A 30-year-old man underwent replantation of his dominant thumb at the
metacarpophalangeal joint level 2 days ago. Since replantation, the temperature
of the thumb has been between 87.8 F (31 C) and 93.2 F (34 C). The temperature
is now 82.4 F (28 C), and there is brisk capillary refill and venous engorgement.
Management at this time should include
1. Surgical exploration
2. Application of leeches
3. Stellate ganglion blocks
4. Intra-arterial streptokinase
5. Elevation and reevaluation in 1 hour

Answer: 1
Discussion: The patient is experiencing impending failure of the replanted thumb. In a study by
Moneim and Chacon, they found that vascular thrombosis in the postoperative period is the
major factor in failure after replantation. When it occurs, it has to be aggressively dealt with by
surgical exploration and revision of the vascular repair. The best results are obtained within 11
hours of the repair and nonsurgical management uniformly led to failure.
Question 96 -
Surgery is being considered for a 30-month-old child who has the
deformity shown in Figure 21. Which of the following studies should be ordered
before proceeding with surgery?
1. Liver profile
2. Myleogram
3. Platelet count
4. CT scan of the head
5. Angiogram of the extremity

Answer: 3
Discussion: Radial deficiency is associated with other abnormalities. 40% of patients with
unilateral and 77% of bilateral involvement will have associated malformations:
1. Holt-Oram syndrome – Radial deficiency and cardial septal defect.
2. Fanconi syndrome – Severe aplastic anemia
3. TAR syndrome
4. Trisomy 17
5. VATER syndrome
Because of the association with severe aplastic anemia, a platelet count should be done before
any surgical intervention.
Question 97 -
A 15-year-old dancer who has had brief episodes of back pain over the
past 2 years now reports a 3 month history of low back pain and a decreased
tolerance for dance. Figure 22a-c show flexion and extension x-rays and CT scan.
She reports no leg pain, nor any bowel or bladder difficulties. Management
should now consist of

1. Post spinal fusion from L5to S1


2. Primary repair with an iliac bone graft
3. Post spinal fusion of L4-5
4. A pantaloon body cast and 6 weeks of bed rest
5. Rest, NSAIDS, and limited dancing

Answer: 5
X-rays show a Grade I spondylolisthesis. Studies have shown that pts with grade
3 or 4 spondylolisthesis had a minimal success (8%) with nonoperative treatment,
whereas pts with grade 1 or 2 spondylolisthesis had significant relief of pain.
Conservative tx included rest, NSAIDs, abdominal strengthening exercises,
hamstring stretching exercises, or traction.
Question 98
Midfoot pain in a professional ballet dancer who has been amenorrheic
for more than 6 months is most likely to be a

1. Stress fracture of the proximal fifth metatarsal


2. Stress fracture of the base of the second metatarsal
3. Stress fracture of the neck of the second metatarsal
4. Morton‘s neuroma
5. Lisfranc‘s joint subluxation

Answer: 2
Post traumatic changes in the mid-and forefoot in dancers include talar spurring, metatarsal shaft
fractures, avascular necrosis of metatarsal heads, osteoarthritis, stress injury at the base of the
second metatarsal, and acquired pes planus. The two common causes of midfoot pain is
traumatic synovitis of the 2nd metatarsal. Unable to differentiate the two clinically, so need an
MRI or bone scan.
Question 99 -
A right-handed 35-year old man who underwent a Putti-Platt repair for
recurrent anterior instability 20 years ago now has increasing shoulder pain and
stiffness. Examination of the shoulder reveals internal rotation to the posterior
superior iliac spine and external rotation to 10 degrees with the shoulder
adducted. The supraspinatus and infraspinatus are moderately atrophied. What is
the most likely diagnosis?

1. C5 radiculopathy
2. Subscapularis rupture
3. Glenohumeral arthrosis
4. Rotator cuff arthropathy
5. Suprascapular nerve compression at the spinoglenoid notch

Answer: 3
Osteoarthrosis of the glenohumeral joint is a potential late complication of the anterior Putti-Platt
capsulorrhaphy. Disabling pain in the shoulder began an average of 13.2 after a Putti-Platt repair
that had been done for recurrent anterior unidirectional instability. Osteoarthrosis of the
glenohumeral joint resulted in substantial limitation of motion. Complications of the Putti-Platt
surgery include persistent pain, recurrent subluxation or dislocation, or residual weakness of the
shoulder; paresthesias of the musculocutaneous nerve, and infection. This late complication
develops when the repair is excessively tight, a 20-25 degree limitation of full external rotation is
desired and expected after rehabilitation. The most direct correlation with the severity of
osteoarthrosis was the degree of limitation of external rotation.
Question 100 -
Which of the following acetabular/femoral head components would be
expected to show an optimal combination of linear and volumetric wear?

1. 22-mm femoral head in combination with a metal-backed polyethylene


component
2. 22-mm femoral head in combination with an all-polyethylene acetabular
component
3. 28-mm femoral head in combination with a metal-backed polyethylene
component
4. 32-mm femoral head in combination with an all-polyethylene component
5. 32-mm femoral head in combination with a metal-backed polyethylene
component

Answer: 3
The size of the femoral head and the calculated mean annual rate of volumetric wear has a
significant relationship. Rate of volumetric wear was highest in assoc with 32 mm femoral lowest
in assoc with 22 mm heads. The predominant mechanisms of wear of the polyethylene were
abrasion and adhesions rather than fatigue-cracking or delamination on the subsurface.
Decreased thickness of the polyethylene has an adverse effect on the rate of wear of the metal-
backed components. Rate of linear wear is highest in assoc. with 22-mm heads and lowest in
relation to 32-mm heads, so the optimal size of a femoral head should be 28 mm where there is
minimal linear and volumetric wear.
Question 101 -
Work-related injuries to the lower back are most often related to which of the
following risk factors?
1. Female gender
2. History of cigarette smoking
3. L5-S1 spondylolisthesis on pre-employment radiography
4. Decreased strength of the lower extremities on pre-employment testing
5. Decreased flexibility of the lumbar spine on pre-employment testing

Answer: 2
The strongest variable for predicting subsequent low back pain is current or prior low back pain,
defined at time loss for back pain during the previous 6 months or at the time of initial
examination; relative risk 60%. Smoking was associated with a 40% increased risk of reporting
back pain.
Question 102
Flow cytometry of tumors measures the
1. Size of cells
2. Amount of DNA in cells
3. Nucleus-cytoplasm ratio
4. Specific DNA sequences
5. Specific messenger RNA sequences

Answer: 2
Flow cytometry is a method of quantitating components or structural features of cells primarily by
optical means. Ploidy and cell cycle analysis of cancers is the major diagnostic use. Cells are passed
single file through a laser beam by continuous flow and several parameters are measured
including Cell Diameter, proportional quantity of granular (DNA) within the cell, and using
fluorescent probes the total DNA or a specific DNA/mRNA sequence can be counted. In
examining tumors the amount of DNA in each cell is important for determining neoplasia.
Question 103 -
Which of the following nerves supply the muscles on each side of
internervous plane identified when performing the anterior (Smith-Petersen)
approach to the hip?
1. Femoral and obturator nerves
2. Femoral and superior gluteal nerves
3. Femoral and lateral femoral cutaneous nerves
4. Obturator and superior gluteal nerves
5. Obturator and lateral femoral cutaneous nerves

Answer: 2
The anterior (Smith-Peterson) approach to the hip utilizes the superficial
internervous plane between the sartorius (femoral nerve) and the tensor fascia
lata (superior gluteal nerve). The deep internervous plane is between the rectus
femoris (femoral nerve) and the gluteus medius (superior gluteal nerve).
Question 104
Which of the following terms describe a rehabilitative exercise in which
the foot is mobile and the motion of the knee is independent of hip and ankle
motion?
1. Isotonic
2. Isokinetic
3. Isometric
4. Open kinetic chain
5. Dynamic variable resistance

Answer: 4
Open chain exercises of the lower extremity are defined as "The foot is mobile, and motion at the
knee joint occurs independent of motion at the hip and ankle joints, as opposed to closed chain
exercises in which the foot is fixed and motion at the knee joint is accompanied by motion at the
hip and ankle joints in a predictable manner.
Question 105 -
A biological or an artificial material in which the direction of loading does
not influence its mechanical properties can be defined as
1. isotropic 2. anisotropic
3. homogeneous 4. nonhomogeneous
5. orthotropic

Answer: 1
If the material is isotropic, its material properties are not dependent on the orientation of the
material in the specimen and possess the same mechanical properties in all directions. 2-
Anisotropic-Have mechanical properties that vary with the orientation of loading. 3-
Homogenous- Have a uniform structure or composition throughout the material. 4-
Nonhomogenous- The opposite of homogenous 5-Orthotropic-unknown definition
Question 106
Following irrigation and debridement, what is the preferred method of
fixation for a displaced open tibia fracture with a 16-cm clean wound?

1. Closed reduction and cast immobilization


2. Uniplanar external fixation
3. Open reduction and internal fixation with a dynamic compression plate
4. Unreamed intramedullary rod
5. Multiple plane external fixator

Answer: 4
Unreamed nails disrupt the diaphyseal cortical circulation by about 30% as compared to reamed
nails that disrupt the circulation about 70%. This aids in healing of open fractures. Because of the
smaller diameter used with unreamed nails they are weaker and therefore are made solid without
cannulation for added strength. Unreamed nails have a lower rate of infection than plates.
External fixators are used for periarticular fractures with compromised soft tissue and when the
foreign body might attribute to higher rates of infection like a nail or plate.
Question 108 -
Which of the following radiographic views best shows the size and
displacement of a posterior wall fracture of the acetabulum?

1. Inlet view of the pelvis


2. Outlet view of the pelvis
3. AP view of the hip
4. Ilial oblique view (external oblique) of the hip
5. Obturator oblique

Answer: 5
This view best reveals the posterior acetabular wall and the anterior column of the pelvis. This
view is best taken by elevating the affected hip 45 degrees to the horizontal by means of a wedge
and directing the beam through the hip joint with a 15 degree upward tilt. The inlet view best
delineates posterior displacement of the hemipelvis. The outlet view best views the sacrum, the
sacroiliac joints, and the sacral foramina, caudad and cephalad displacement as well. The
standard AP radiograph is used in the initial trauma series as a screening tool. Ilial oblique views
best view the anterior wall of the acetabulum and the posterior column of the pelvis.
Question 109 -
What is the primary immediate source of energy for muscle?

1. Glycolytic pathway
2. Oxidative phosphorylation
3. Breakdown of fat
4. Breakdown of protein
5. Breakdown of adenosine triphosphate

Answer: 5
The basic source of energy for muscle contraction is ATP. ATP is also the immediate energy source
for muscle. The body then utilizes glucose to produce ATP. Glycolysis splits glucose to form two
molecules of pyruvic acid and two ATP.
Almost 90% of the total ATP formed by glucose metabolism is formed during oxidative
phosphorylation. This is accomplished by a series of enzymatically catalyzed reactions in the
mitochondria. When the body‘s stores of carbohydrates decrease below normal, glucose can be
formed from the breakdown of protein and fat via gluconeogenesis to yield more ATP.
Question 110 -
A 13-year-old quarterback feels a ―pop‖ in his knee while being tackled.
Radiographs of the knee and results of a Lachman‘s test are normal. Examination
reveals tenderness over the distal femoral physis. To help confirm the diagnosis,
management should first include

1. an MRI scan
2. arthroscopic examination
3. AP and frog-lateral radiographs of the pelvis and hips
4. varus and valgus stress radiographs of the knee
5. physical examination of the knee under anesthesia

Answer: 4
Injuries involving the distal femoral epiphysis cartilage plate are fairly common. Appropriate
management should include a complete knee exam with a standard series of X-rays. Of particular
importance is to notice any varus or valgus deformity. Stephens et al. (JBJS 1974:56A) measured
varus and valgus deformities clinically and roentgenographically. A varus or valgus deformity
greater than 3 degrees as compared with the uninjured side was considered clinically significant.
Such deformities are highly suggestive of a physeal injury. A MRI may show the injury, but is
expensive as is an arthroscopy or examination under anesthesia.
Question 111 -
A 47-year-old woman who reports mild, aching pain in her knee has no
history of trauma. Examination of the knee is normal. Figure 23a shows the AP
radiograph. A bone scan shows increased uptake at this site only. Figure 23b
shows the CT scan, and Figure 23c shows the histology from the CT scan-guided
needle biopsy. Treatment should include
1. extended curettage and polymethylmethacrylate cementation
2. extra-articular resection of the knee and an allograft arthrodesis
3. wide resection of the proximal tibia and custom prosthetic replacement
4. prophylactic internal fixation and postoperative irradiation
5. excision of the lateral condyle and reconstruction with a hemicondylar
allograft

Answer: 1
Patients with giant cell tumors typically present with local pain, swelling and tenderness.
Radiographs usually reveal a lesion destructive of both medullary and cortical bone. The lesion is
characterized by an expanding zone of radiolucency that is located eccentrically in the end of a
long bone in an adult.
The modern technique for the removal of a giant cell tumor involves wide decortication of all the
bone overlying the area of the tumor. The cavity is filled with methylmethacrylate bone cement
and covered with demineralized bone matrix to stimulate the restoration of strong cortical
boundaries. The other procedures are much more invasive and not necessary to treat a low-grade
neo-plastic lesion such as a giant cell tumor. Fewer complications and better functional results
have been found after intralesional excision and insertion of methylmethacrylate than other
techniques.
Question 112 -

Figure 24 shows the AP radiograph of a 22-year-old


woman who has pain with activity and crepitus at he second
metatarsophalangeal joint. Despite nonsurgical treatment, the pain has become
progressively worse over the past year. Treatment should include

1. silicone implant joint replacement


2. metatarsophalangeal joint arthrodesis
3. metatarsophalangeal joint debridement
4. resection of the metatarsal head
5. resection of the base of the proximal phalanx

Answer: 3
The use of NSAIDs along with soft metatarsal insoles may alleviate the stress on MTP joint.
Occasionally, an intraarticular injection may alleviate symptoms. Failed conservative treatment
should be followed by synovectomy of the MTP joint. When the proximal phalanx is
hyperextended, a flexor tendon transfer may also be used to produce some plantar flexion. Early
synovectomy may prevent the development of subluxation, dislocation, or hammer-toe
development. The other treatment options given are much more invasive and are typically
salvage procedures.

Question 113 -
Figure 25a shows the initial postoperative AP radiograph, and Figures 25b
and 25c show the current AP and lateral radiographs of a 46-year-old woman
who underwent open reduction and internal fixation of a distal fibula fracture
and placement of a syndesmosis screw 15 months ago. She has full function, but
the ankle swells with activity. The radiographs reveal that

1. Fixation of the syndesmosis has failed


2. Widening of the ankle mortise has led to the failure of fixation
3. Infection around the syndesmosis screw has led to osteomyelitis
4. The syndesmosis screw is broken
5. Motion between the tibia and fibula has caused loosening of the syndesmosis
screw
Answer: 5
This question is answered easily by reviewing the 3 figures shown. You can see that the
syndesmotic screw simply is not broken when viewed in the AP and lateral. Also, there is no
evidence of periosteal elevation or osteomyeltis. The ankle mortise has less than 3 mm of
widening and therefore, the fixation has not failed and answers #1 and #2 can be eliminated.
You can note the fine sclerotic border (Osteolysis) that has classically formed from the
micro-motion between the tibia and fibula. This motion has caused loosening of the syndesmotic
screw over the past 15 months and the resultant ankle swelling with activity.
Question 114 -
An MRI scan of the brain of a patient with spastic diplegia will most likely
show
1. Microcephaly
2. A temporal lobe cyst
3. An Arnold-Chiari type 1 malformation
4. Periventricular leukomalacia
5. Agnesis of the corpus callosum

Answer: 4
Microcephaly literally means small brain. This is used to identify certain primary developmental
anomolies as well as secondary changes seen in neonatal hypoxia and encephalomalacia. These
individuals demonstrate long survival, but with moderate mental retardation and variable spastic
weakness. Though they can show the physical characteristics of the question above, it is not a
diagnostic feature in spastic diplegics. Unless, of course, their spastic diplegia is specifically related
to developmental anoxic event (cerebral palsy) causing a compromise in total mass brain tissues.
Agnesis of the corpus callosum is a rare anomaly that is asymptomatic neurologically
unless tests designed specifically to test the transfer of information from one cerebral hemisphere
to another is employed. Additionally, the corpus callosum is intimately related to the development
of the limbic system which governs major CNS roles as behavior, memory, and emotions.
Therefore, this would not involve the neurologically challenged spastic diplegic individuals.
Question 115 -
Which of the following laws describes the mechanical effects believed to
play a role in the etiology of adolescent tibia vara?
1. Wolff‘s
2. Hooke‘s
3. Hilton‘s
4. Muller-Haeckel
5. Heuter-Volkmann

Answer: 5
This is one of those OITE questions that there is no way to make an educated guess. You either
knew this or simply picked the best sounding answer. Wolff‘s Law, which we are all familiar with,
states that the bone builds structural development most suited for the stresses applied. I could not
find Hooke‘s and Hilston‘s Laws in any orthopaedic or medical text and therefore, believe these
were simply ―fillers‖ to make five selections. Muller-Haeckel Law is from the Biogenetics principal
that living matter always arises from pre-existing living matter (very profound). Finally, the Heuter-
Volkmann Law is one that applies to many mechanisms in orthopaedics, but this question is
specifically regarding tibia vara. This law states that increased pressure/strain across the
epiphyseal plate on the medial aspect of the knee will inhibit its longitudinal growth. Conversely,
Depelch‘s Law not mentioned in the question, but certain to show up in future OITE‘s states that
distraction at the level of the physeal plate will cause increases longitudinal growth.
Question 116 -
Which of the following statements best describes the tension in the
different components of the posterior cruciate ligament when the knee is taken
from full extension to 90 degrees of flexion?
6. Both the anterolateral and posteromedial bands are isometric and do not
significantly change with flexion
7. The anterolateral band is lax and becomes tight in flexion, while the posteromedial
band is tight, and becomes lax in flexion
8. The anterolateral band is tight and becomes lax in flexion, while the posteromedial
band is lax and becomes tight in flexion
9. Both the anterolateral and posteromedial bands are lax and become tight in flexion
10. Both the anterolateral and posteromedial bands are tight and become lax as the
knee is flexed

Answer: 2
The PCL is considered to be the primary stabilizer of the knee because of its strength, 2X that of
the ACL, and its location near the central axis of rotation of that joint. Most authors describe two
inseparable components of the PCL: an anterior portion, which forms the bulk of the ligament,
and a posterior portion whose oblique fibers attach to the tibia. The posterior fibers are believed
to be taut in extension and lax in flexion; the opposite applies to the anterior fibers (see diagrams).
Like the ACL, some fibers of the PCL are believed to be taut during all ranges of knee motion. All
the other answers are just variations of answer 2.
Question 117
Which of the following Orthosis best relieves symptomatic hyperpronation
of the feet in a patient who runs regularly?
1. UCB orthosis
2. Rigid orthosis with a medical arch support
3. Semi-rigid orthosis with lateral forefoot posting
4. Semi-rigid orthosis with a medial arch support
5. Medial heel wedge attached to the running shoes
Answer: 4
UCB, rigid with medial support-incorrect: The use of a rigid Orthosis for a high arched, rigid
cavovarus foot is less effective because it attempts to push the rigid foot into a different position.
Selection of the Orthosis is dependent on the amount of control that is needed….Rigid orthotic
devices should be avoided if possible.‖ Mann. See Below.
Lateral forefoot posting-Incorrect: Often those with supination symptoms have lateral complaints,
and need to be addressed laterally. Medial posting utilized excessive pronation, with Achilles
tendinitis and pronation or with posterior tibial tendinitis. Medial heel wedge attached to the
running shoes-Incorrect: Medial support is most often placed in the arch. With hell wedging
reserved for more severe cases.
Question 118
What is the most common complication following metatarsal osteotomy
for a bunion deformity in an adolescent?

1. Hallux varus
2. Osteonecrosis
3. Recurrence of the hallux valgus
4. ―Transfer‖ second metatarsalgia
5. Physeal arrest of the first metatarsal

Answer: 3
Hallux varus-The question does not specify proximal or distal osteotomies, however it is the most
common complication with overcorrection of proximal 1st metatarsal osteotomies. Mann. Pg. 329.
―Transfer‖ 2nd metatarsaglia-most significant, not most common, complication of the Mitchell
Osteotomy.Mann pg. 319.
Physeal arrest of the first metatarsal-―While an open epiphysis cannot be considered an absolute
contraindication to an osteotomy in either the proximal phalanx, or proximal first metatarsal, it is
important at surgery to determine the exact location of the metaphyseal epiphysis to avoid injury.‖
Pg. 307 Mann, Surgery of Foot and Ankle.
In studies performed by Blais et. Al. A females full foot growth is usually achieved by 14 years and
at 12 years an average less than 1 cm of total foot growth remains with less than 50% of this
growth at the proximal epiphysis. Males‘ terminal growth expected at 16 years of age with 3cm
left at 12 years and approximately 1.5 cm of metatarsal growth.
Most studies show recurrence of Hallux Valgus deformity after surgical correction in the juvenile
as inordinately high.
Question 119
The best indication for a knee fusion after a failed total knee replacement
is

1. Aseptic loosening in a 70-year-old patient


2. Mechanical failure of a hinged knee prosthesis
3. Failed knee replacement complicated by reflex sympathetic dystrophy
4. Infection with soft-tissue deficit
5. A prior patellectomy

Answer: 4
Knee arthrodesis rather than reimplantation should be considered a) following failure of a
primary knee prosthetic arthroplasty when the extensor mechanism is insufficient, b) in the
immuno-compromised patient, c) in the knee with poor soft tissue coverage, d) when the
causative microorganism in the infected prosthetic knee is particularly virulent which may be
sensitive to only toxic antimicrobials. It should also be considered in the relatively young patient
with high functional demands, whose arthritic process only involves one joint.
Question 120 -
An infected total knee replacement with symptoms occurring within 4
weeks of surgery and no radiographic signs of osteomyelitis would be best
managed with
1. Knee fusion
2. Open irrigation and debridement
3. Arthroscopic irrigation and debridement
4. One-stage exchange arthroplasty
5. Two-stage exchange arthroplasty

Answer: 2
Treatment of an early infection demands thorough debridement of the wound and appropriate
parenteral antibiotics. Systemic treatment with appropriate antimicrobial agents should continue
for a minimum of 4 weeks following debridement for an early infection. An infection diagnosed
later than 4 weeks following surgery is less likely to have a successful result without removal of
the components. OKU V pg. 490.
Arthroscopic debridement not recommended secondary to missing cutaneous tracks and
soft tissue/muscle involvement.
Question 121 -
A 16-year-old girl who swims on her high school team reports pain in the
shoulder after swimming. History reveals a glenohumeral dislocation at age 14
years while doing the backstroke. Examination shows a positive anterior
apprehension sign. Treatment at this time should consist of

1. Putti-platt repair
2. Open Bankart repair
3. Injection of a subacromial corticosteroid
4. Arthroscopic transglenoid capsular shift
5. Rehabilitation of the scapular and rotator cuff muscles

Answer: 5
p.579: ―The Putti-Platt procedure is contraindicated in multidirectional instability (AMBRI);
tightening the front of the shoulder will only increase the likelihood of posterior instability. In
traumatic instability (TUBS) the data suggest that such a procedure, which limits external rotation
is not necessary if the Bankart lesion is solidly repaired.‖
p. 577: ―A vigorous effort to stabilize the shoulder with exercises is particularly indicated in
patients with multidirectional or posterior instability and in athletes requiring a completely normal
or supranormal range of motion.‖
p. 989: ―If the [swimmer] has symptoms of subluxation, a conservative program that strengthens
the external rotators is warranted. Surgery is seldom indicated.‖
Question 122
A 20-year-old man sustains an unsalvageable crush injury to his foot and
undergoes a below-knee amputation at the midtibial level. The stump is now
healed and painless, and the patient has been fitted with a temporary prosthesis.
Which of the following prosthetic components would best enable the patient to
play basketball and other recreational sports?

1. Silicone suction socket and an energy-absorbing foot


2. Silicone suction socket and a variable resistance ankle
3. Plastic suction socket, telescoping pylon, and a solid ankle cushioned heel
(SACH) foot
4. Plastic socket with a hinged thigh cuff and a SACH foot
5. Patellar tendon-bearing suction socket and a uniaxial hydraulic ankle

Answer: 1
Question 123
Which of the following types of osteosarcoma is associated with the best
prognosis & long survival?
1. Parosteal
2. Periosteal
3. High-grade intramedullary
4. Osteosarcoma occurring in Paget‘s disease
5. Osteosarcoma occurring in irradiated bone

Answer: 1
p. 194 (parosteal): Early adequate treatment [of parosteal osteosarcoma] should lead to cure in
most patients. A long-term survival rate of 80%-90% is to be expected for patients who have
parosteal osteosarcomas without dedifferentiation.‖
p. 163 (in Paget‘s disease): ―Although long-term survival is rare for patients with this type of
sarcoma, four patients have survived more than 10 years.‖
p. 164 (in irradiated bone): ―The location of these tumors in unresectable locations such as the
skull, clavicle, scapula, and spine explains the traditionally poor prognosis.‖
Question 124
A patient undergoes anterior spinal decompression through a left
retroperitoneal approach as treatment of an L2 burst fracture. Following surgery,
examination reveals the temperature of the right foot is cool when compared
with the left foot. Which of the following neurologic structures has most likely
been disrupted?
1. Cauda equina
2. Conus medullaris
3. Genitofemoral nerve
4. Lumbar sympathetic plexus
5. Lumbar parasympathetic plexus

Answer: 4
Chapter 6, p. 240: ―The sympathetic chain lies in the lateral aspect of the vertebral body and on
the most medial aspect of the psoas muscle. It is easy to identify as the tissue is cleared from the
front of the vertebrae.‖
Chapter 60, p. 777: ―[After sympathetic denervation], at first the blood flow rises markedly
because of lost vascular tone, but over a period of says to weeks the blood flow returns to almost
normal because of progressive increase tone of the vascular musculature itself, thus
compensating for the loss of sympathetic tone.‖
Question 125 -
A 14-year-old patient who has homocystinuria and is Risser 3 is referred for
surgical treatment of scoliosis. In addition to the usual risks associated with
posterior spinal fusion, the family should be advised that the patient ‗s underlying
condition significantly increases the perioperative risk of

1. Spinal pseudoarthrosis
2. Spinal cord traction injury with paralysis
3. Arterial and venous thromboses
4. Superior mesenteric artery syndrome
5. Crankshaft phenomenon
Answer: 3
Tendency towards venous and arterial thrombosis along with mental retardation, dislocation of
the lens and skeletal changes resembling Marfan‘s are all clinical features of homocystinuria.
Homocysteine is toxic and causes endothelial cell damage. Increased platelet stickiness is also
associated with the disease.
1 and 2 are concerns during spinal fusion but are not specific to this disease. 4 may occur with
homocystinuria but is not related to spinal fusion. 5 occurs with PSF in the younger population
because of the growth potential remaining.
Question 126 -
Radiographs of a 20-year-old college athlete who sustained an injury to
the ankle reveal no fractures or widening of the ankle mortise. Examination
shows swelling at the ankle region and pain with medial lateral compression of
the distal tibiofibular joint. Which of the following studies would best help in
confirming a diagnosis?

1. Inversion stress radiograph


2. MRI scan
3. CT scan
4. Nuclear bone scan
5. External rotation stress radiograph

Answer: 5
Pain with medial-lateral compression of the distal tib-fib joint, swelling in the area and history of
injury indicate disruption of the syndesmosis. External rotation stress of the ankle will open the
joint space medially confirming the diagnosis.
2 and 3 are occasionally utilized when there is questionable involvement of surrounding
bone or tendons. 4 rarely indicated for acute ankle sprain but can help in the diagnosis of RSD
following ankle injury.
Question 127 -
A healed fracture of the tibia that demonstrates 25 degrees apex posterior
angulation and 28 degrees varus angulation on AP and lateral radiographs is
most accurately described as a

1. Complex deformity with an angulation in two planes


2. Single deformity less than 20 degrees, apex posterolateral
3. Single deformity greater than 30 degrees, apex posterolateral
4. Single deformity less than 20 degrees, apex posteromedial
5. Single deformity greater than 30 degrees, apex posteromedial

Answer: 3
Deformities that are seen simultaneously on the AP and lateral roentgenograms of the same bone
are actually shadows of the true deformity. If, for example, angulation is seen at the site of a
fracture on both of the standard roentgenograms, then the true plane of angulation is
somewhere between the coronal and sagittal planes, and the actual amount of angulation is
greater than that visualized on either roentgenogram. Because the standard roentgenograms are
orthogonal (at right angles) to each other, it is possible to calculate the actual plane and angle of
deformity on the basis of dimensions measured from the roentgenograms.
Question 128
A normal functioning posterior tibialis tendon is best confirmed by which
of the following physical findings?

1. Pronation of the foot during the stance phase of gait


2. Heel inversion at the beginning of a single limb heel rise
3. Active inversion of the nonweightbearing foot
4. Active plantar flexion of the first ray against resistance
5. Active plantar flexion of the foot during the push-off phase of gait
Answer: 2
The main function of the Tib. Post. Tendon is inversion of the subtalar joint and adduction of the
forefoot. This initial inversion stabilizes the subtalar joint allowing the gastrosoleus complex to
forcibly invert the joint while plantarflexing the ankle. Heel inversion begins during flatfoot when
the Tib Post. fires at 7% of the walking cycle. The gastrosoleus kicks in at 30% of the cycle to
maintain inversion, which locks the midfoot for push-off.
Question 129 -
A forward bend examination of an asymptomatic premenarchal 12-year-
old girl who has waist asymmetry reveals a right thoracolumbar angle of trunk
rotation of 12 degrees. The radiograph shown in Figure 26 reveals a curve that
measures 31 degrees between T6 and T11 with the apex to the right. She is
Risser 1 and her triradiate cartilage has closed. Management should consist of

1. An MRI scan
2. Observation and repeat radiographs in 4 months
3. Application of a thoracolumbalsacral orthosis for 22 to 24 hours per day
4. Electrical stimulation at night
5. Physical therapy

Answer: 3
Treatment is based on the probability of curve progression. Major factors that influence curve
progression are skeletal maturity, curve magnitude and curve type. Candidates for bracing are
Risser 0, 1, or 2 and have a curve in the range of 20-40 degrees. Patients who present with curves
between 30-40 degrees should be braced on presentation. Acceptable frequency of follow-up
visits are at 4-6 month intervals. MRI scan would be indicated to evaluate a patient with atypical
scoliosis, the patient described is a fairly typical scoliosis patient. Scoliosis treatment by electrical
stimulation or physical therapy has not been shown to be effective.
Question 130 -
Which of the following advantages does the use of a vascularized fibula
graft have over a nonvascularized fibula graft?

1. Begins to remodel and hypertrophy more quickly


2. Provides a better scaffold for osteoconduction
3. Reduces the risk of early fracture
4. Reduces technical difficulty
5. Lowers donor site morbidity

Answer: 1
A vascularized fibula graft, because its osteogenic potential remains unhampered by loss of
vascularity it will begin to remodel and hypertrophy more quickly. Both types of grafts would act
equivocably as scaffolding for osteoconduction. Early risk of fracture is increased if the
nonvascularized fibula graft is over 12 centimeters in length as compared to a vascularized graft.
And a vascularized graft requires greater technical skills and a larger dissection to isolate the
vascular pedicle with associated increased donor site morbidity.
Question 131 -
A 14-year-old girl has idiopathic scoliosis with a 52-degree right thoracic
curve and a 36-degree left lumbar curve. The rotation of the apical vertebra
appears greater in the thoracic curve. A sagittal view radiograph shows the spine
to be virtually straight. The iliac apophyses are Risser 2. Treatment should consist
of

1. Anterior fusion of the lumbar curve


2. Anterior and posterior fusion of the thoracic curve
3. Posterior fusion of the thoracic curve
4. Posterior fusion of the thoracic and lumbar curves
5. Application of a brace until the iliac apophyses are Risser 4 or 5, followed by
surgical correction

Answer: 3
The patient‘s curve is beyond the limit for bracing to be effective (40 degrees). Therefore, surgical
treatment should be utilized. King type II curves (predominant thoracic curve with secondary
lumbar curve) historically has used selective posterior thoracic fusion with segmental hook
systems and rotation maneuvers to correct the thoracic curve and compensatory lumbar curve.
The patient‘s primary curve is thoracic and lumbar fusion would be contraindicated.
Question 132 -
A 20-year-old college football player sustains a forceful hyperextension
injury to his shoulder 4 months after undergoing an anterior capsular shift.
Examination 2 weeks later reveals anterior tenderness. He is unable to lift the
dorsum of his hand away from his back. What is the most likely diagnosis?

1. Subscapularis rupture
2. Type III SLAP lesion
3. Disruption of capsular shift
4. Isolated traumatic subluxation
5. Injury to the axillary nerve after dislocation

Answer: 1
Subscapularis rupture is most likely, given weakness with the lift-off test. The injury is usually
caused by either forceful hyperextension or external rotation of the adducted arm. Patients will
complain of anterior shoulder pain and weakness of the arm when used above and below
shoulder level. SLAP lesions usually occur with a fall onto an outstretched arm in abduction and
slight forward flexion. No mention was made of shoulder instability (answers 3&4), or deltoid
weakness (answer 5).
Question 133 -
The familial occurrence of Legg-Calve-Perthes disease may, in some cases,
be attributed to
1. hypophosphatemia
2. high dietary cholesterol intake
3. deficiency of lipoprotein A
4. deficiency of protein S and protein C
5. elevated levels of antithrombin III
Answer: 4
One of the suggested causes of Perthes disease is a hypercoaguable state in a child. This would
lead to thrombotic venous occlusion in the proximal femur resulting in venous hypertension and
osteonecrosis of the femoral head. Therefore look for an answer that would result in a
hypercoaguable state. There is no link between hypophosphatemia or high dietary cholesterol
intake and a hypercoaguable state. Elevated levels of antithrombin III would result in bleeding,
not coagulation. The referenced paper demonstrated a familial occurrence in protein S and
protein C deficiency and elevated levels of lipoprotien A. Protein C and S are antithrombotic
factors and lipoprotein A is a thrombogenic, atherogenic lipoprotein associated with
osteonecrosis in adults.
Question 134 -
Which of the following methods of treatment of a displaced Lisfranc
fracture-dislocation will most reliably lead to good functional results?

1. Weightbearing short leg cast


2. Nonweightbearing short leg cast
3. Removable splint and early motion
4. Open reduction and internal fixation
5. Elastic compression bandage with full weightbearing

Answer: 4
It is recommended that If ligament injury is documented in the presence or absence of fracture,
open reduction and internal fixation should be performed. According to gossans and De Stoop,
who studied 20 patients with Lisfranc injuries, 5 of 7 had a poor outcome with plaster
immobilization, 1 out of 2 had a poor outcome with reduction and plaster, and 7 out of I 1 had a
good result with ORIF. 2 of these had a poor result, and 2 had injuries severe enough to result in
amputation due to Clostridium infection secondary to open fractures.
Question 135 -
Which of the following metastatic tumors to bone carries the greatest risk
of complications from intraoperative bleeding?
1. Breast
2. Prostate
3. Gastrointestinal
4. Kidney
5. Multiple myeloma

Answer: 4
Question 136
A 15-year-old boy has a fracture of the proximal tibia extending from the
apophysis of the tubercle up through the posterior part of the proximal tibial
epiphysis and into the joint. What is the most likely mechanism of injury?
1. Varus stress
2. Valgus stress
3. Torsional loading
4. Hyperextension of the knee
5.Contraction of the quadriceps while axially loaded

Answer: 5
Tibial Tuberosity Fractures Fractures of the tibial tuberosity are uncommon avulsion injuries. Most
are sportsrelated and occur in older adolescents. Type I fractures represent an avulsion of a small
fragment of the tuberosity. Type II fractures involve the entire anterior tuberosity with extension
proximally to the level of the horizontal portion of the proximal tibial physis. Type III injuries
involve the entire tuberosity with extension proximally into the articular surface, a SalterHarris
type III fracture. Patients present with pain, swelling, and tenderness over the tuberosity. Patella
alta may be present. Surgical treatment of type I fractures is needed if patella alta (compared to
the normal uninjured side) and a significant bony prominence are present. Displaced types II and
III fractures are treated with open reduction and internal fixation. A cancellous interfragmentary
screw may be placed through the tuberosity into the metaphysis. Because this injury occurs in
patients near skeletal maturity, growth arrest with secondary genu recurvatum is rare.
Question 137 -

Figures 27a through 27c


show the radiographs of the femur of a 46-year-old man who has a fracture of
the right humerus, multiple rib fractures, and fractures of the right femur as a
result of a motor vehicle accident. There is a 10-cm clean wound over the
anteromedial thigh that communicates with the femoral shaft fracture. The
neurovascular examination of the right leg is normal. After meticulous irrigation
and debridement, management of the femoral fractures should consist of
1. Primary internal fixation at both fracture levels
2. External fixation as definitive ttt for both #
3. Skeletal traction and delayed internal fixation of both fractures
4. Primary internal fixation of the proximal fracture and delayed fixation of the
femoral fracture
5. Primary internal fixation of the femoral shaft fracture and delayed fixation of
the proximal #

Answer: 1
Debridement of wound and immediate reamed nailing performed on 67 patients with open
fractures of femoral diaphysis including 36% grade 1, 45% grade II. All fractures healed within
four months after injury.

Question 138 -
A high school distance runner reports a 3-week history of heel pain while
running. Examination elicits no pain with dorsiflexion or with palpitation of the
plantar fascia; however, pain is evident with palpitation over the muscular origin
of the abductor hallucis. What is the most likely etiology of the pain?

1. Heel spur
2. Plantar fascitis
3. Dysfunction of the tibialis posterior tendon
4. Compression of the first branch of the lateral plantar nerve
5. Compression of the calcaneal nerve

Answer: 4
The tibial nerve gives off two branches. In the distal 1/3 of the leg, the tibial nerve curves
anteroinferiorily into the sole of the foot behind the medial malleolus, deep to the flexor
retinaculum and between the tendons of the flexor hallicus longus and the flexor digitorum
longus muscles. Just distal to the flexor retinaculum, the tibial nerve divides into the medial and
lateral plantar nerve. At this site, the lateral plantar nerve gives off its first branch which lies
anterior and adjacent to the abductor hallicus.
Question 139
A 56-year-old laborer sustained a subcoracoid dislocation of the shoulder
as a result of falling off a scaffold 3 weeks ago. He now is unable to actively raise
his arm and has constant pain. What is the most likely diagnosis?

1. Displaced labral tear


2. Tear of the rotator cuff
3. Fracture of the glenoid rim
4. Palsy of the axillary nerve
5. Palsy of the musculocutaneus nerve
Answer: 2
Thirty-one patients who were unable to abduct the involved arm after reduction of a primary
anterior dislocation of the glenohumeral joint were found to have a ruptured rotator cuff. In their
series, the incidence of injury to the axillary nerve was 7.8% as compared with 100% for rupture
of the rotator cuff.
Question 140 -
A 40-year-old woman has progressive pain and limited range of motion in
her long finger. Figure 28a shows the radiograph, and Figure 28b shows a
biopsy specimen of the same lesion. What is the most likely diagnosis?

1. Enchondroma

2. Osteoblastoma
3. Giant cell tumor
4. Aneurysmal bone cyst
5. Fibrous dysplasia

Answer: 3
Giant Cell tumors (GCT) are relatively common in the appendicular skeleton, most common
during the 4th or 5th decades of life. GCT‘s are destructive lesions with variable amounts of
reactive bone at their margin. Successful treatment requires complete surgical excision.
Question 141 -
A patient reports persistent pain in the wrist 6 months after undergoing
open reduction and internal fixation of a Galleazi fracture. Radiographs of the
wrist in a neutral position are normal. Which of the following studies would best
evaluate the reduction of the distal radioulnar joint?

1. Arthrogram of the wrist


2. MRI scan of both wrists
3. CT scan of both wrists in the same position
4. Radiographs of the wrist in supination and pronation
5. Radiographs of the opposite wrist in the same position

Answer: 3
If late instability is suspected after the associated fractures have healed, comparison scans in
pronation (to detect volar subluxation) neutral (dorsal subluxation and diastasis of the DRUJ) and
supination (to confirm the degree of reduction) If early a single axial CT in any position can aid in
diagnosis. As described by Mino, a line drawn on both the dorsal and volar radial ulnar borders
will place an adequately reduced ulna between them
Question 142 -
A form of renal osteodystrophy that is characterized by pure osteomalacia
is caused by

1. Secondary hyperparathyroidism
2. Phosphate retention secondary to uremia
3. Insufficient renal synthesis of 1, 25 dihydroxy vitamin D
4. Aluminum deposition in bone from oral phosphate binders
5. Persistent acidosis aggravating the negative calcium balance

Answer: 4
There are many causes of rickets and osteomalacia. Renal osteodystrophy is a common
complication of chronic renal failure and is one of the most common causes of osteomalacia. Pure
osteomalacia is caused by the aluminum in phosphate binders used to treat hyperphosphatemia
in renal failure. Desferoxamine is an effective chelator of aluminum in patients with biopsy
documented aluminum-associated osteomalacia. Pure osteomalacia also can be caused by
hypophosphatemia. The other choices are part of the mechanism of bone changes in renal
osteodystrophy.
Question 143 -
A 25-yo man sustains the injury shown in Figures 29a and 29b as a result
of high-speed trauma. Examination reveals diffuse weakness in the lower
extremities that is slightly worse on the right side, and decreased rectal tone and
sensation. A CT scan is shown in figures 29c and 29d. Definitive treatment of the
injury to the spine is delayed because of a severe pulmonary contusion. At 15
days after the injury, the patient‘s neurological status remains unchanged.
Management should now consist of
1. Posterior fusion at T10-L3 with segmental instrumentation
2. Laminectomy and fusion of T12-L2 with segmental instrumentation
3. Bed rest in a hyperextension brace
4. L1 vertebrectomy and anterior decompression with strut graft fusion and
instrumentation
5. Progressive ambulation as tolerated in a custom-molded thoracolumbosacral
orthosis

Answer: 4
Anterior decompression is preferred for patients who have incomplete neurologic injury with a
compression or burst fracture, if the posterior ligament complex has not been disrupted.
Progressive ambulation with a TLSO may be indicated for a simple wedge compression fracture. If
the compression is greater than 40% and no neural deficit, then posterior stabilization may be
indicated to prevent further collapse and neural deficit. Wedge compression with subluxation or
dislocation indicates disruption of posterior ligament complex requiring posterior instrumentation
and fusion. Compression and distraction injuries of the middle complex should be treated by
Harrington or C-D distraction and compression instrumentation. (Bohlman: The Spine, 1992
pp1047-1056)
Question 144 -
A patient is in respiratory distress as a result of a high-speed motor vehicle
accident. After emergent intubation, the arterial blood is poorly oxygenated, and
examination shows left-sided tracheal deviation, absence of breath sounds on the
right side and tympany on percussion over the right side of the chest.
Management should include

1. Positive-pressure ventilation
2. An immediate radiograph of the chest
3. Adjustment of the position of the endotrachael tube
4. Insertion of a large-bore needle into the pericardial space
5. Insertion of a large-bore needle in the right second intercostal space,
midclavicular line
Answer: 5
This patient has a tension pneumothorax and needs immediate decompression to convert it to a
simple pneumothorax until definitive treatment with a chest tube can be performed. The other
choices are inappropriate.
Question 145
The axial stability of a 4-pin uniplanar external fixator used to treat a
patient who has a transverse midthird fracture of the tibia with a 5-mm fracture
gap can be most greatly increased by

1. Allowing the ends of the fracture to touch


2. Adding a second connecting bar
3. Adding one pin to each fracture fragment
4. Increasing the pin diameter from 4 mm to 6 mm
5. Decreasing the connecting bar-to-bone distance from 6 cm to 4 cm

Answer: 1
The mechanical behavior of an externally fixed fracture can be evaluated in axial, bending and
torsion loads. The axial stiffness is increased most by the load sharing effect of cortical contact and
compression, 94% of intact bone. Sidebar to bone spacing is proportional to the distance cubed.
The pin diameter is proportional to the diameter to the 4th power.
Bone contact allows load sharing between bone and fixator for compressive, torsional, and
certain bending loads. Without bone contact, the external fixator must support the full load. It is
also possible to apply compression across a fracture gap using an external fixator. With transverse
fractures, application of compression across the fracture site can greatly increase the stiffness of
the framebone system.
Question 146

Figure 30 shows the current radiograph of a 32-year-old


woman who had a giant cell tumor of the distal radius that was treated with
curettage/burring and packed with polymethylmethacrylate 2 years ago. What is
the most likely diagnosis?
1. Osteomyelitis
2. Malignant degeneration
3. Stress fracture
4. Local recurrence of the giant cell tumor
5. Bone resorption due to methylmethacrylate

Answer: 4
Number four is the correct answer because recurrence is the most likely cause of the lytic zone.
Since cement resists invasion by the tumor, lysis of the surrounding bone is inevitably produced in
a recurrence. A 1-2 mm lytic zone can be found normally, greater than 5 mm lytic zone is positive
for recurrence and the 3 – 5 mm lytic zone is very suspicious and should be followed by MRI and
image guided needle biopsy. No other sign of osteomyelitis or stress fractures are noted.
Question 148 -
Figure 32 shows the diagram of the flap ABCDE that is to be transposed to
cover the triangular defect of FAB. What portion of the flap is most vulnerable to
ischemia?

1. A
2. B
3. C
4. D
5. E

Answer: 2
The figure depicts a rotation I flap of the fourth digit. The flap pivots about point A. The greatest
stretch is appreciated at point B. The greater the tension on the skin, the greater the risk of
exceeding the tissues' ability to perfuse and survive. Pitfalls can be avoided by making the flap
larger than necessary, thereby decreasing tension. Maneuvers such as the "back cut" moving the
pivot point closer to the primary defect and "Burow's Triangle" excising skin from opposite the
original defect, allowing easy movement of the rotating flap can assist in this end.
Question 149 -
A 40-year-old woman has had pain in the metatarsophal joint of the
second toe for the past 6 months despite nonsurgical treatment. A dorsalplantar
stress test reproduces the pain, and there is 10 mm of dorsal subluxation of the
toe. Radiographs show a normal second metatarsophalangeal joint. Surgical
treatment should consist of synovectomy and
1. Advancement of the plantar plate
2. Resection of the second metatarsal head
3. Dorsiflexion osteotomy of the second metatarsal neck
4. Transfer of the flexor digitorum longus tendon to the dorsum of the toe
5. Transfer of the extensor digitorum comminus tendon to the metatarsal neck

Answer: 4
Number four is the correct answer because the flexor digitorum longus tendon will give excellent
plantar support when transferred to the dorsum of the toe and this removes the main dynamic
deforming force. Number one is incorrect because the planter plate will stretch in time and be
insufficient. Number two is incorrect because the radiographs show a normal second
metatarsophalangeal joint surface and it is not a fixed/dislocated joint and it is in a young patient.
Number three is incorrect because it is not a frank dislocation. Number five is incorrect as this is
reserved for mild deformity and would not be sufficient correction in this case.
Question 150 -
A 25-year-old patient who was wearing a seat belt in the back chair of a
car involved in a head-on collision undergoes a laparotomy. During surgery, an
injury to the sigmoid colon is identified and treated. Two days later the patient
has back pain when sitting in a chair. What is the most likely diagnosis?
1. Sacral fracture
2. Burst fracture of L5
3. Cauda equina syndrome
4. Distraction-flexion injury at L3
5. Distraction-extension injury at L3

Answer: 4
Number four is correct because it fits the injury pattern and symptoms of the scenario given
above. (Chance/Seat Belt Fracture) Number one is incorrect because it is the most ―likely‖
diagnosis in this injury pattern. It would need more of a direct blow type of injury to be true.
Number two is incorrect because it does not fit the injury pattern. The burst fracture is usually an
axial/vertical loading injury. Number three is incorrect because the patient is having back pain
only and no lower extremity or bowed/bladder complaints that you would typically find in a
cauda equina syndrome. Number five is incorrect because it does not fit the injury pattern of the
scenario given above with comparison to the number four answer.
Question 151
A 65-year-old man undergoes total shoulder arthroplasty. Examination in
the recovery room reveals the absence of voluntary deltoid and biceps
contraction, weakness of wrist extension, and absence of sensation over the
lateral arm and radial forearm. Management should now consist of
1. An MRI scan of the shoulder
2. An MRI scan of the cervical spine
3. Electromyographic and nerve conduction velocity studies
4. Immobilization in a sling and early passive range of motion exercises
5. Immediate return to the operating room for exploration of the brachial plexus

Answer: 4
One and two are incorrect because they would show artifact, which would interfere with evaluation of a
brachial plexus lesion unless a hematoma is suspected. Number 3 is incorrect as timing of the EMG is
important because electrophysiologic signs of denervation do not become apparent for more than two
weeks after the injury and therefore should be performed four to six weeks after surgery if no clinical
recovery is occurring. However, an electromyogram obtained early in the course of a neurologic injury may
demonstrate a preexisting degenerative lesion. Answer number five is incorrect because in the literature it
has been suggested that injuries to the brachial plexus showing no spontaneous improvement within
three months should undergo surgical exploration. The answer number four is correct because after the
neurological deficit is identified, efforts should be directed at maintaining passive motion of affected joint
through the recovery phase. Temporary static splinting can be beneficial as well.
Question 152 -
A 76-year-old woman who underwent primary total knee arthroplasty 9
months ago falls and sustains a transverse fracture of the proximal third of the
patella that is displaced 5 mm. Although the knee is painful, she is able to
ambulate and has a 5-degree extension lag. The component appears to be well
fixed. Treatment should now include

1. cerclage wiring
2. tension band wiring
3. removal of the patellar component
4. revision of the patellar component
5. immobilization of the knee and protected weightbearing

Answer: 5
In a retrospective study based on 21 patellar fractures, non-operative treatment was selected in
12 of the cases with only one unsatisfactory result. Patellar fractures are classified as nondisplaced
or displaced (greater than 2mm), with or without displacement of the polyethylene component
from the bone bed, and with or without loss of the integrity of the extensor mechanism. In the
cases of displaced fractures it appears that these patients do well with nonoperative therapy.
Nonoperative treatment was uniformly satisfactory with no adverse effect noted on extensor lag,
quadriceps strength, or knee score. The initial treatment of a nondisplaced patellar fracture after
total knee arthroplasty should always be nonoperative regardless of physical findings.
Question 153
Lymphatic metastasis is a common feature of which of the following
lesions?

1. Liposarcoma
2. Nodular fasciitis
3. Rabdomyosarcoma
4. Malignant fibrous histiocytoma
5. Extra-abdominal desmoid tumor

Answer: 3
Rhabdomyosarcoma is a high grade malignancy with a rapid growth pattern. Local recurrence
and distant metastasis after inadequate excision occurs in almost all instances and is uniformly
fatal. The primary site of metastasis is the lung. Lymph node metastasis occurs in about > 10% of
the patients, (synovial cell sarcoma = 25%). Whereas M.F.H. only occasionally metastasizes to
regional LN, and liposarcoma is slow growing & recurrences or local metastasis is < 10%.
Question 155
Which of the following conditions is most
commonly associated with the congenital hand problem shown in Figure 33?
1. Clubfeet
2. Thrombocytopenia
3. Congenital scoliosis
4. Ventricular septal defect
5. Arnold-Chiari malformation
Answer: 1
The condition shown in the figure is congenital constriction band syndrome. The probable causes
of the syndrome are hemorrhages and necrosis in mesenchymal tissues caused by exogenous
agents after the condensation of the digital rays. Patients have been grouped by Patterson in the
following clinical types; simple ring constrictions, ring constrictions accompanied by fusion of the
distal parts with or without lymphedema, ring constrictions accompanied by fusion of the distal
parts ranging from fenestrated or terminal syndactyly, and intrauterine amputation. Congenital
constriction band syndrome is an uncommon disorder that can present with many clinical
features including constriction bands, intrauterine amputations, syndactyly, acrosyndactyly,
caraniofacial defects such as cleft lip and palate, visceral anomalies from truncal bands, and fetal
death. Clubfoot is a frequently associated anomaly.
Question 156 -
A 35-year-old machinist sustains a crush injury to the forearm in an industrial
accident. Figure 34 shows the arm following skeletal stabilization and fasciotomy.
Wound closure is best accomplished by

1. delayed primary closure


2. free flap
3. pedicle groin flap
4. full-thickness skin graft
5. split-thickness skin graft

Answer: 5
Wound closure is one of the most critical factors in ensuring a functional result following a crush
injury. Skin coverage reduces edema, protein lead, and risk for infection, and minimizes healing
with fibrosis and subsequent loss of joint motion. Wound closure should be performed as soon as
all nonviable tissue has been debrided. Split-thickness skin grafts, however, do not provide
optimal coverage over exposed tendons, bones, or joint spaces. In a study presented in The
Journal of Trauma, once the wounds became covered by granulation tissue, split-thickness skin
grafts were placed as an initial measure to provide a further barrier to infection and protein leak.
The patients showed good functional results despite not utilizing flap coverage for their extensive
injuries.
Question 157 -
Figure 35 shows a postoperative radiograph of a femur fracture proximal
to a total knee prosthesis that was treated by open reduction and blade plate
fixation 9 months ago. What is the most likely reason the previously well seated
screw has backed out of the central portion of the plate?
1. Infection
2. Nonunion
3. Improper screw length
4. Osteonecrosis of the distal fragment
5. Use of a cortical screw instead of a cancellous screw

Answer: 2
[Radiograph: An A-P view of the distal femur. There is patchy increased radiodensity in the mid-
plate region at the level of the fracture, which was just distal to the middle-distal third junction.
The screw in the middle hole of the plate has backed out approximately 1 cm. There is no
obvious fracture line at this time. The lateral cortex is not intact.]
The referenced article clearly states that the major cause of failure of fractures to heal is non-union.
From the radiograph it appears the screw at one time was long enough. While the distal
fragment perhaps has some disuse osteopenia there is no focal area of what could be considered
osteonecrosis. We are not given any history of possible infection even though this could be the
case.
Question 159 -
Figures 37a & 37b show radiographs of a right handed, 78 year old man
who had painful G-H arthritis, moderate limitation of motion, and good strength
prior to replacement of the humeral head 2 years ago. At the time of treatment,
the rotator cuff was intact. He now has limited motion, weakness, and persistent
pain in the shoulder. What is the most likely diagnosis?

1. Infection
2. Tear of the rotator cuff
3. Loosening of the humeral component
4. Arthritis of the glenoid
5. Arthritis of the A-C joint

Answer: 4
[Radiograph: Shows a well fixed prosthesis, possible superiorly migrated. There is no ―joint space‖
between the prosthesis and the glenoid.]
The referenced text states that the most frequent complications in shoulder hemiarthroplasty are
due to rotator cuff and tuberosity problems or postoperative instability. Return of motion is
expected to be two-thirds to three fourths of pre-operative. 80-90% of patients undergoing
hemiarthroplasty experience pain relief. Of the 10-20% who fail to get relief, conversion to total
shoulder arthroplasty should be considered. The referenced article discusses the excellent results
of TSA and not the reasons for hemi failure. The point strongly emphasized in the article was that
for those patients requiring conversion to TSA, strongly suspect infection.
Question 160 -
What is the primary reason for choosing a bone graft substitute rather
than an autologous cancellous graft in the treatment of distal radius fractures
with metaphyseal comminution and impaction?

1. Reduced morbidity
2. Improved osteoinduction
3. Improved osteoconduction
4. More rapid revascularization
5. Lower risk of disease transmission

Answer: 1
Autologous grafts provide the best osteoinductive and conductive properties. These properties
lead to increased revascularization in incorporation of the graft. In the study quoted, there was a
10% minor and 6% major complication rate for iliac crest bone graft harvesting. The most
frequent complications included; superficial infections and seromas, as well as minor hematomas.
Major complications included herniation of abdominal contents, vascular injury, deep infection or
hematoma, and iliac wing fracture.
Question 161 -

Figures 38a &


38b show radiographs of a 40 year old man who underwent a Putti-Platt repair
for recurrent dislocations at age 22. He reports increasing pain in the shoulder
and limited motion for five years. Examination reveals 130 degrees of elevation
and 15 degrees of external rotation. Non-surgical treatment has failed.
Treatment should now consist of what?

1. Manipulation Under Anesthesia


2. Arthroscopic acromioplasty
3. Arthroscopic debridement of G-H joint
4. Replacement of the humeral head
5. Lengthening of the subscapularis and release of the anterior capsule

Answer: 5
[Radiograph: Well positioned G-H joint. Mild degenerative changes.]

Late onset of pain (average 13 years) was noted following this procedure in a small number of
patients. The pain is attributed to excessive G-H compressive forces due to limited external
rotation. NSAIDS and PT are first line treatments. If these fail, the authors demonstrated good
results with release of the anterior structures. Choices 3 & 4 would probably be reserved for
patients older than 50 with more advanced signs of degenerative disease.
Question 162 -
A 50-year-old alcoholic man has erythema and swelling in his entire
dominant upper extremity. He has a WBC of 15,000/mm3, a temp of 101 oF
(38.3 oC), and a blood pressure of 90/60 mm Hg. After hemodynamic
stabilization, the cellulitic forearm is longitudinally incised dorsally and volarly.
The forearm muscles are normal in appearance; however, the subcutaneous fat is
necrotic. A culture will most likely reveal

1. Bacteroides
2. E. coli
3. Staph. aureus
4. group A streptococcus
5. Clostridium perforingens
Answer: 4
Necrotizing fasciitis is frequently caused by strep infections. Cellulitis, which is an inflammatory
infection of the subcutaneous tissues, is usually due to Staph or Strep infections (Haemophilus is
also seen in children). Significant Streptococcal infections include erysipelas, which produces a
progressively enlarging red, raised painful plaque, severe toxicity, fever, leukocytosis and
bacteremia; Necrotizing fasciitis produces XXXXX. Clostridium perforingens typically causes gas
gangrene (which can also develop from gram neg. and strep infections), presenting as
progressive pain, distal edema, and a malodorous serosanguinous discharge.
Question 163 -
A 15-year-old girl has a thoracic kyphosis that causes mild pain.
Examination reveals a sagittal curve measuring 55 degrees and wedging of the
eighth through tenth vertebrae. The iliac apophyses are Risser 4. Management
should include

1. observation and exercises


2. bracing with a thoracolumbar orthosis
3. fusion of the posterior spine
4. fusion of the anterior spine
5. fusion of the anterior and posterior spine

Answer: 1
Scheuermann‘s Disease classically presents with >45o thoracic kyphosis and anterior wedging (5o
or more) at three sequential vertebrae. Disc narrowing, end-plate irregularities, scoliosis,
spondylosis, and Schmorl‘s nodes are also seen. It‘s more common in adolescents and males.
Normally, these patients are treated (1) in a brace if the curve is progressive and Risser 3 or less, (2)
with surgical fusion if >75o and Risser 3 or less, (3) with surgical fusion if >65o and Risser 4/5 if
necessary or symptomatic. Posterior instrumentation, anterior release and interbody fusion is the
treatment of choice for curves >75o, or those >55o on hyperextension. Other causes of kyphosis
include trauma, infection, spondylitis, bone dysplasia, neoplasia, neurofibromatosis.
Question 164
Figures 39a through 39c show a clinical photograph and the radiographs
of a 32-year-old woman who has been unable to actively extend her dominant
ring and small finger for the past two weeks. She has no history of trauma and
has minimal pain. Examination reveals full passive range of motion (ROM) of the
fingers. Active ROM of the wrist is extension of 40 degrees and flexion of 35
degrees. Active forearm pronation is 45 degrees, and supination is 50 degrees.
Treatment should consist of
1. Total wrist replacement and bridge grafts
2. palmar shelf arthroplasty and tendon transfers
3. Darrach distal ulna resection, dorsal tenosynosynovectomy, and tendon
transfers
4. Radioscaphate fusion, extensor tenodesis, and Darrach distal ulna resection
5. Total wrist fusion and tendon transfers

Answer: 3
The patient has acute rupture of the extensor tendons to the fifth and fourth fingers. Her X-rays
show generalized severe arthritis, consistent with rheumatoid arthritis, in the distal radius and
ulna, the carpals, and MCPs. There also appears to be a sharp osteophyte on the dorsal surface of
the distal ulna (Fig 39c). Rheumatoid arthritis affects both joints and tendons because both are
lined with synovium. The distal ulna typically becomes roughened with a sharp edge which acts
―like a buzzsaw‖ on the overlying tendons. This is called a Vaughan-Jackson lesion when extensor
tendons are ruptured at the distal radial-ulnar joint. Of the dorsal/extensor compartments,
typically the digiti minimi is involved first and further ruptures progress radially as the hand ulnarly
deviates and the other tendons sublux and are brought within range of the ―buzzsaw.‖ The cited
articles state that after a dorsal exposure the hypertrophic tenosynovium is removed from each
tendon sytematically, and the wrist joint then evaluated. Any bony spicules which may further
damage tendons are removed, and the distal ulna is excised. Tendon transfers, grafts, or repairs
are then performed. Surgical results tend to be better with single or double tendon ruptures than
with multiple tendon ruptures.

Question 165 -
A 75-year-old woman who has groin pain states that she had total hip
arthroplasty 15 years ago. The radiograph shown in Figure 40 reveals that the
left acetabular component is grossly loose. Revision of the acetabular component
should include use of a

1. constrained acetabular component


2. protrusion ring with morselized graft
3. cemented metal backed acetabular component
4. cemented all-polyethylene acetabular component
5. cementless hemispherical component with screw fixation

Answer: 5
The patient has an all-poly acetabular component, which is grossly loose and has migrated.
Intermediate and long-term results of revision THA‘s using a cemented acetabular component
have had high failure and re-revision rates. One of the reasons includes sclerotic bone lacking in
the trabeculae needed for cement. Threaded cups and bipolar implants have also had
unacceptably high failure rates. Published results using non-cemented components have shown
they performed much better in the intermediate-term, especially those supplemented with fins,
screws or spikes. The cited authors (Petrera and Rubash) preferred a titanium-mesh cup and place
two supplemental screws if the cup is unstable (assessed by manipulation intraoperatively).
Question 166 -
An 8-1/2-month-old male infant who has developmental dysplasia of the
hip was treated in a Pavlick harness for 3 months. At follow-up, examination of
the hip reveals full flexion and extension and abduction to 80 degrees. Figure
41a shows an AP arthrogram and Figure 41b an arthrographic view in flexion
and abduction. Management should now consist of
1. application of a hip abduction brace for 22 hours per day
2. application of a hip spica under anesthesia
3. discontinuance of all bracing and repeat radiographs in 3 months
4. open reduction of the hip and application of a spica cast
5. open reduction, varus osteotomy, and application of a spica cast

Answer: 4
The arthrograms (which are actually reversed from the question) reveal a superiorly dislocated hip
in both views. The head is lateral to Perkins‘ line, superior to Hilgenreiners‘ line, and Shenton‘s line
is broken. Tachdjian gives the following reasons for operative reduction: hip unreducible by
closed means, hip requires extreme position to maintain reduction, unstable reduction, non-
concentric reduction. Miller gives a little better explanation of treatment options: if under 6 mos.
and reducible: Pavlick Harness (confirm the reduction with U/S or x-ray); if under 6 mos. and
unreducible: Pavlick harness, and if unsuccessful then traction and closed reduction. A stable
reduction must be demonstrated in the harness within 2 to 4 weeks. Abduction bracing may be
used for residual dysplasia if the child is ambulating. If 6 to 18 months and unreducible: traction
and closed reduction (check reduction with arthrogram; medial dye pool < 5mm is good; if failed
closed reduction, then must operatively reduce), then cast for > 4 months followed with nighttime
bracing. If 12 to 18 months who fail close reduction, must OR. If >3 years: OR with pelvic
osteotomy.
Question 167
Which of the following findings is more suggestive of neurogenic rather
than vascular claudication in the differential diagnosis of leg pain?
1. Loss of skin hair on the feet
2. Absent pulses on vascular examination
3. Pain that originates proximally and spreads distally
4. Pain that is relieved by stopping and standing
5. Pain that is worse when the patient walks uphill rather downhill

Answer: 3
All of the answers except answer 3 are suggestive of vascular claudication. Additional signs and
symptoms of vascular claudication include diffuse aching/cramping/tired pain that is worse with
exertion, is usually in the calves (also may be in feet, thighs, hips, and buttocks). Neurogenic
claudication, on the other hand, usually is sharply painful, occurs in the back, buttocks, thighs,
calves; is worse with spine extension and walking; is better/less with spine flexion and lying
recumbent; pulses and skin are unaffected.
Question 168 -
Injury to the C7 nerve root results in weakness primarily of the
1. wrist flexors and finger flexors
2. elbow flexors and wrist flexors
3. elbow flexors and finger flexors
4. elbow extensors and wrist flexors
5. elbow extensors and wrist extensors

Answer: 4
Level Key Muscles
Diaphragm
Deltoid, elbow flexors, diaphragm
Elbow flexors, wrist extensors
Elbow extensors, wrist flexors
Finger flexors (distal phalanx of middle
finger)
Finger abductors (5th digit), intrinsics of hand
Segmental innervation to intercostal muscles, abdominal and paraspinal muscles
L1, L2, L3 Hip flexors
L4 Quadriceps
Tibialis anterior
Toe extensors, hip abductors
Ankle plantarflexors, peronei
Question 169 -
Which of the following conditions is most likely inherited as an autosomal
dominant trait?
1. Syndactyly
2. Macrodactyly
3. Camptodactyly
4. Preaxial polydactyly
5. Postaxial polydactyly

Answer: 5
Postaxial polydactyly involves polydactyly of the little finger, preaxial polydactyly usually involves
the thumb or the index finger. Postaxial polydactyly is further divided into Type A, in which the
well formed extra digit artier with the fifth or an extra metacarpal and Type B, a small extra digit
that is poorly formed and often is little more than a skin tag. Postaxial polydactyly is inherited as
an autosomal dominant trait with marked penetrance.
Question 170
Which of the following surgical options after resection of a sarcoma about
the knee would require a patient to expend the greatest amount of energy while
walking?

1. Arthrodesis
2. Rotationplasty
3. Above-knee amputation
4. Osteoarticular allograft
5. Endoprosthesis (custom arthroplasty)
Answer: 3
Discussion: The answer to this question is based on a study by Harris which
tested the effeciency , rate and percent of max rate in ambulation in amputees,
those with arthrodeses and arthroplasties. They found that Above knee
amputees expended the most energy, followed by arthrodesis patients, and then
arthroplasty patients.
Question 171 -
Chronic flatfoot deformity is most commonly associated with a contracture
of the
1. Plantar fascia
2. Spring ligament
3. Deltoid ligament
4. Intrinsic tendons
5. Gastorcnemius-solelus complex

Answer: 5
According to Mann a contracted Achilles tendon that limits dorsiflexion is the underlying
pathology behind symptomatic (rigid) flatfoot. Due to chronic Achilles contracture there is over
time attenuation of the spring ligament with progressive rocker bottom deformity.
Deltoid ligament, plantar fascia, or intrinsic tendon contracture would not cause this deformity.
Question 172 -
The pharmacologic action of botulinum-A toxin can be best described as

1. Prevention of presynaptic release of acetylcholine


2. Prevention of synthesis of presynaptic acetylcholine
3. Activation of acetylcholinesterase at the motor end-plate
4. Blockage of postsynaptic action of acetylcholine until reserves are depleted
5. Stimulation of release of presynaptic acetylcholine until reserves are depleted

Answer: 1
DISCUSSION: BotulinumA toxin acts by interfering with presynaptic acetylcholine release at
cholinergic nerve terminals without destroying nerve endings, nerve terminals, or neuromuscular
junctions. Thus, the toxin blocks neuromuscular control and functionally denervates the muscle.
Question 173
With respect to femoral component design, stress relief osteopenia in the
proximal femur following noncemented total hip arthroplasty appears to be most
strongly influenced by the

1. stiffness of the femoral component.


2. head offset of the femoral component.
3. femoral component material modulus of elasticity.
4. extent of the femoral component porous coating.
5. Presence of a femoral component collar.

Answer: 1
Stress relief changes seem to be most strongly related to component stiffness. However, stress
relief changes are probably multifactoral and involve stiffness, location and extent of porous
coating and the presence or absence of a collar. Host factors also influence stress relief.
Question 174 -
The stability of the longitudinal arch of the foot during standing with
equal weight on both feet is due primarily to

1. plantar fascia and quadratus plantae tendon.


2. ligamentous structures connecting the tarsal bones.
3. shape of the tarsal bones and the intervening joints.
4. activity of the intrinsic muscles of the foot.
5. activity of the posterior tibialis and the peroneus longus muscles.

Answer: 2
The longitudinal arch is stabilized by heavy ligamentous structures surrounding the tarsal joints
with passive assistance from the plantar aponeurosis. EMG studies have shown little or no intrinsic
muscle activity during quiet standing.
Question 176 -
A 32-year-old has diffuse pain, weakness, and limited overhead motion in the
shoulder as a result of falling on his outstretched arm 2 months ago. Examination
reveals medial scapular winging, and an electromyogram shows denervation of
the long thoracic nerve. Management should consist of
1. scapulothoracic fusion
2. strengthening of the periscapular muscles
3. pectoralis minor-fascia lata graft transfer to the scapula
4. pectoralis major-fascia lata graft transfer to the scapula
5. exploration of the long thoracic nerve, with sural nerve graft

Answer: 2
Most cases of isolated serratus anterior palsy resolve spontaneously, usually within 6 to 9 months
after traumatic injury and within 2 years after an infectious cause. Pectoralis major-fascia lata
graft is an effective treatment for persistent winging.
Question 177 -
A 45-year-old man who has medial compartment disease and chronic
medial knee pain with varus alignment of 5 degrees is best treated with a

1. tricompartmental knee replacement


2. unicompartmental knee replacement
3. medial compartment meniscal allograft
4. valgus-producing distal femoral osteotomy
5. valgus-producing proximal tibial osteotomy

Answer: 5

This pt is less than 50-60 years of age, he has less than 12-15 degrees of varus
angulation, and he has only medial compartment degeneration. If he were older
and if more than one compartment were involved he would not be a candidate
for osteotomy, if the angulation were greater he would be better served by a
distal femoral osteotomy.
Question 178 -
Which of the following factors is most likely to predispose a patient to
dislocation of the patellar component following total knee arthoplasty?

1. Internal rotation of the femoral component


2. External rotation of the tibial component
3. Lateral placement of the femoral component
4. Medial placement of the patellar component
5. Excessive resection of the patella

Answer: 1
The experimental data for this answer came from Anouchi et al The Effects of Axial Rotational
Alignment of the Femoral Component on Knee Stability and Patellar Tracking in Total Knee
Arthroplasty Demonstrated on Autopsy Specimens. This study looked at knee stability, patellar
tracking, and patellofemoral contact points with the femoral component positioned in 5 degrees
internal, 5 degrees external, and neutral alignment in relation to the posterior femoral condyles.
Total knee arthroplasty was performed on four cadavaric specimens without lateral release.
Internally rotating the femoral component produced abnormal laxity seen at 30, 60, and 90
degrees of flexion. There was no gapping noted in the neutral or externally rotated specimens.
The normal pattern for patellar tracking was a gentle curve with maximal deflection at 15 and 60
degrees of flexion. The maximal medial displacement were lowest for the externally rotated
specimens.
Although contact areas could not be quantitatively measured accurately, the contact areas were
more evenly distributed between the medial and lateral sides of the patella in the externally
rotated specimens than they were in either the internally rotated or neutral specimens.
You have to be careful interpreting this data at least in reference to knee stability. In this study a
perpendicular tibial cut was made. The normal tibia has a 30 degree varus slope and thus more
bone is resected from the lateral surface. External rotation of the femoral component
compensates for this.
No tests were done with lateral placement of the femoral component or medial placement of the
patellar component.
Question 179 -
What is the most common sequela of turf toe (hyperextension of the first
metatarsophalangeal joint)?
1. Hallux rigidus
2. Hallux valgus
3. Neuroma of the first web space
4. Fracture of the sesamoid
5. Rupture of the flexor hallucis longus

Answer: 1
The reference article Turf Toe by Rodeo et al 1990 looked at 80 professional football players and
occurrence of turf toe. This study consisted of a questionnaire and physical exam. In a nut shell,
85% of players with turf toe sustained their initial injury on artificial turf. Although turf toe
accounted for 6 cases per year and ankle sprains 24.7 cases per year turf toe accounted for more
missed games. The advent of artificial surfaces and lighter, more flexible shoes for use on artificial
turf are suspected causes for the increased incidence of turf toe. The most common mechanism is
a hyperextension injury to the MTPJ of a foot in a slightly dorsiflexed position. The injury is a
sprain to the capsuloligamentous complex.
What they found was a decreased ROM of the first MTPJ in patients with turf toe and
stated that it demonstrates the POSSIBILITY of long-term sequelae to the turf toe injury and that
hallux rigidus must be considered in the athlete with progressively limited ROM. Other possible
sequelae cited were hallux valgus, production of a dorsal osteophyte, calcification in periarticular
soft tissue, and chondromalacia of the head of the first metatarsal. NO MENTION WAS MADE OF
THE MOST COMMON SEQUELAE OF TURF TOE!
Clanton & Schon Chapter 27 Mann Foot & Ankle state that it remains unclear whether
these sprains (turf toe) will ultimately result in arthritic changes in the affected joint. No mention
is made of hallux rigidus. Again in Mann Chapter 14 on hallux rigidus states that the etiology is
degenerative arthritis of the first MTPJ and what predisposes the patient to degenerative arthritis
is not known. No mention is made of turf toe.
Question 180
A radiograph of a 12-year-old boy who has had an insidious onset of pain
in the right hip for the past 6 weeks shows diffuse narrowing of the joint space.
Examination reveals that he is afrebile, and the range of motion of the hip is less
than 50% of normal in all planes. Laboratory studies show an erythrocyte
sedimentation rate of 21 mm/hr and a WBC of 11,000/mm3. What is the most
likely diagnosis?

1. Sickle cell crisis


2. Idiopathic chondrolysis
3. Hemophilic arthropathy
4. Osteoid osteoma of the femoral neck
5. Legg-Calve-Perthes disease

Answer: 2
First, sickle cell crisis is a localized area of bone marrow infarction with excruciating pain. Swelling
of the extremity and limitation of motion are usually mild. Temperature elevation is usually mild
but is >39 degrees celsius in 29% of patients. It is also limited to 3-5 days in duration.
This patient has no history of hemophilia given. Hemophilic arthropathy begins with a
hemarthrosis.
In osteoid osteoma the pain is typically unrelenting, sharp, boring, worse at night, and
relieved with aspirin. It is not associated with joint space narrowing.
The most common age for Legg-Calve-Perthes disease is 4-8 years. It causes AVN of the
femoral head and widening of the medial joint space is an early radiographic finding.
In Bleck‘s report on Idiopathic Chondrolysis JBJS 1983 nine cases were seen at the
reporting institution between 1973 and 1978. The average age was 11.5 years. All the patients
were otherwise healthy and had no history of systemic illness of previous trauma. All the patients
reported the insidious onset of pain in the anterior part of the hip. All had a decreased passive
ROM. Radiographic examination showed regional osteoporosis, premature closure of the femoral
capital physis, narrowing of the joint space, and lateral overgrowth of the femoral head on the
neck. All laboratory examinations were negative for evidence of infection or rheumatoid arthritis.
Treatment consists of administration of aspirin, active non-loading exercise of the hip, and
protected weight-bearing with crutches.
Question 181 -
Figures 42a and 42b show the sagittal and axial MRI scans of a 24-year-old
patient who has sciatia. Which of the following combinations of physical findings
is most consistent wit the MRI studies?
1. Decreased ankle jerk and positive femoral nerve stretch test
2. Decreased knee jerk and positive straight-leg raising sign
3. Gastrocnemius-soleus complex weakness and positive straight-leg raising sign
4. Weakness of the extensor hallucis longus and positive straight-leg raising sign
5. Weakness of the extensor hallucis longus and positive femoral nerve stretch
test
Answer: 3
Figures 42a and 42b show a posterocentral disc herniation between L5 and S1 and will affect the
S1 nerve root. This will cause sensory changes in the posterior calf and plantar foot, motor loss in
the gastrocnemius/soleus complex, reflex changes in the ankle jerk, and a positive straight leg
raise sign. A positive femoral nerve stretch test may indicate L2, L3, or L4 nerve root irritation.
EHL function is mainly L5.
Question 182 -
An otherwise healthy 65-year-old man has had chronic pain in his
prosthetic knee for the past 9 months. Repeated aspirations reveal a coagulase-
negative staphylococcus infection. To eradicate the infection while maintaining
the best possible joint function, management should consist of

1. Long-term administration of IV and oral antibiotics


2. Open soft-tissue debridement, retention of prosthetic components, and IV
antibiotics
3. Immediate exchange arthroplasty with antibiotic-impregnated cement
4. Two-stage surgical prosthetic exchange and IV antibiotics
5. Resection arthroplasty and IV antibiotics

Answer: 4
Postoperative wound infections following total joint arthroplasty are generally classified as acute
or chronic. The time period is either 2 or 4 weeks from the time of implantation depending on
whom you read. This will, generally, determine if you can attempt a one-stage procedure. The
other considerations are the bacteria‘s resistance to antibiotics and whether or not it produces
glycocalyx. If the microorganism elaborates glycocalyx it is highly likely that it will remain after
surgical removal of the implants and debridement of the joint.
Present recommendations are to avoid a one-stage reconstruction in a patient in whom a
glycocalyx elaborating microorganism has been isolated. In one study 52% of the isolates of S.
epidermidis and 28% of the isolates of S. aureus elaborated glycocalyx. The microbiology
laboratory can be asked to determine if the microorganisms elaborate glycocalyx. In the present
case it should be assumed that the staphylococci elaborate glycocalyx and are resistant to
antibiotics. A two-stage procedure is indicated for these reasons alone.
Antibiotic therapy alone has been used for a select group of patients who could not medically
tolerate either a one-stage or a two-stage arthroplasty. The patient in this case is listed as
otherwise healthy.
The decision to perform a resection arthroplasty as a definitive procedure without reimplantation
is based on the bacteria‘s resistance to antibiotics, quality of the local soft tissues, the complexity
of the reconstruction, the patient‘s refusal to have another operation, the patient‘s overall health,
or a combination of these factors. None of which appear to be present in this case.
Question 183 -
Figures 43a and 43b show the radiographs of an 8-year-old boy who was
referred by his gym teacher because of an awkward running pattern. The patient
denies any pain in his hips. Examination reveals a mild Trendelenberg gait and
decreased internal rotation of the left hip to 25 degrees compared to 40 degrees
on the right. What is the most likely diagnosis?

1. SCFE
2. MED
3. Perthes disease
4. Hypothyroidism
5. Chondrolysis

Answer: 3
The referenced article is a current concept review on the treatment of Legg-Calve-Perthes Disease
and does not specifically mention diagnosis. Self limited non-inflammatory deformity of the
weight-bearing portion of the femur, likely due to osteonecrosis. Usually seen in 4-8 year old
males with delayed skeletal maturity. Family history, low birth weight, and abnormal birth
presentation.
Symptoms include-pain, effusion (from synovitis), and a limp, decreased ROM especially
Abduction internal rotation. Trendelenburg stance is common.
The key in this question is the age, decreased ROM, Trendelenburg gait. The prognosis is directly
related to the age at presentation, after 8 years old the prognosis is poor. SCFE(Slipped Capital
Femoral Epiphysis)-Usually seen in obese adolescent boys with a family history. African American
more common. Often related to endocrine abnormalities, presenting with externally rotated gait,
decreased internal rotation, thigh atrophy, with hip or knee pain, symptoms vary with the
acuteness of the slip.
Hypothyroidism is often a finding with patients presenting with SCFE and chondrolysis is a known
complication of SCFE.
Question 184 -
A 31-year-old man who is a recent immigrant from Guatemala has had
pain in his back and thighs for the past 12 months. History notes a recent
diagnosis of gout, and the patient reports falling a distance of 3 feet on his
buttocks immediately before the pain began. Examination reveals that he is
neurologically intact. Plain radiographs are shown in Figures 44a and 44b, and
T2-weighted MRI scans are shown in Figures 44c and 44d. The most likely cause
of the pathologic fracture is

1. gout.
2. osteoporosis.
3. eosinophilic granuloma.
4. tuberculosis of the spine.
5. metastatic disease of the spine.

Answer: 4
The plain films demonstrate lumbar AP and Lateral radiographs with 32 degree anteriorly
wedged compression fracture of L1. On closer evaluation one notices the adjacent disc spaces are
narrowed. The center of the vertebra is sclerotic with the anterior inferior endplate irregular.
The MR demonstrates involvement of L1 and adjacent disc spaces. With peri-vertebral edema and
mass involving the posterior aspect of the vertebral body, placing pressure on the spinal cord.
There is also involvement of the L2-3 disc with early signal changes.
The slow clinical course of the patients symptoms, being from a third world country and the
findings on imaging studies, with an anterior wedge compression fracture. The level most
commonly involved with TB is lower thoracic and upper lumbar. The anterior wedging results in
the classic ―Gibbus‖.
Some of the MR findings are also consistent with metastatic disease, but with the localized mass,
peri-vertebral abscess make this choice more unlikely.
The usual findings on MR found with TB are: Confluently decreased signal intensity of the
vertebral bodies associated interspace with poor distinction between these on short TR/short TE
images: Abnormal increased signal of the disk on long TR/long TE images with an abnormal
configuration (i.e., absent intranuclear cleft): Increased signal of the vertebral endplates at the
abnormal disk level on long TR/long TE images.
Question 185
Osteoarthritic cartilage is characterized by decreased

1. water content.
2. Synthesis of type I collagen.
3. Proteoglycan content.
4. Activity of chondrocytes.
5. Synthesis of hyaluronate.

Answer: 3
Water content is increased early in the OA cartilage and is associated with the loss of
proteoglycan and an increase in the collagen/proteoglycan ratio. Collagen synthesis is increased,
but the majority of the material synthesized is Type II, not Type I collagen. Type I collagen
increases slightly, particularly in the osteophyte region.
Chondrocytes in OA are much more metabolically active than those of normal cartilage.
Studies reveal measuring the levels of utilization on sulfate and H-glucosamine informing
components of cartilage matrix. The amount of DNA also rises in OA cartilage most noticeable at
the articular chondrocyte.
The synthesis of hyaluronate increases. Recent data suggests 1) hyaluronate that is synthesized is
abnormal and, hence, does not allow aggregation; or 2) The excess synthesis is a response to a
rapid degradation of synthesized product. This is in light of the fact that synthesis increases, yet
concentration of hyaluronate decreases.
The correct answer is that proteoglycan content decreases. The synthesis of proteoglycan
increases in OA, but due to the increased rate of breakdown of proteoglycan there is a net result
of lowered proteoglycan content.
Question 186 -
Which of the following metastatic carcinomas has the worst long-term
prognosis?
1. Lung
2. Breast
3. Prostate
4. Thyroid
5. Renal

Answer: 1
Question 187 -
The Magnetic resonance imaging signal characteristics of a high-grade
soft-tissue sarcoma are best described as

1. T1-low, T2-low.
2. T1-low, T2-high.
3. T1-moderate, T2-low.
4. T1-high, T2-low.
5. T1-high, T2-high.

Answer: 2
After plain radiographs of the affected area have been obtained, magnetic resonance imaging
modality is the best imaging modality for detecting and characterizing the lesion, regarding
definition of normal muscle, fascial boundaries, and the tumor mass. Although MR imaging is not
specific in determining whether lesions are benign or malignant, it can be useful in evaluation
other characteristics, such as size, pattern of growth, integrity of natural boundaries, and
homogeneity.
Intravenous contrast agents are not necessary to evaluate neurovascular structures. Both the T1
& T2 weighted images are essential to detect and characterize soft tissue lesions.
Most Tumors have long T1 and T2 relaxation times, therefore, in most instances signal intensity
alone is of limited benefit. Exceptions are lipoma, hematoma, intra-lesional hemorrhage.
Hemorrhage may occur in some soft tissue lesions, especially sarcomas, leading to the low T1 and
high T2 intensity sound on MR. Miller, indicates that Water, CSF, acute hemorrhage and soft tissue
tumors appear dark on T1 and light on T2.
Question 188
A ―p value‖ of 4% (p=0.04) indicates that the

1. hypothesis is incorrect or invalid


2. interobserver error rate is 4%.
3. Standard deviation is 4% higher or lower than the mean.
4. Sample size is 4% larger than required to be clinically significant.
5. Probability that the differences noted between two study groups were due to
chance alone is 4%.

Answer: 5
The paper cited is an excellent review in detail about confidence intervals including the
mathematical equations.
The p value is the probability that the chance selection of patients might suggest a difference in
treatment that was not real. Only with a small p value are we willing to believe that the observed
difference in treatment is very likely real and not simply due to chance.
The confidence interval provides a measure of the magnitude of the possible difference between
two groups of patients, regardless of whether or not the p value was small. This makes the
confidence interval more informative than the p value when different treatments are compared.
This is much beyond the scope of this review.
Question 189 -
What is the predominant collagen type in osteoarthritic articular cartilage?
1. I
2. II
3. IV
4. IX
5. X

Answer: 2
A question as to whether the collagen of OA cartilage remains type II has arisen in recent years.
Conflicting evidence exists, but the consensus suggests that the newly formed cartilage collagen
which attempts to repair the disease‘s defect, is mostly type II rather than type I. Type I collagen
may be increased slightly, particularly in the osteophyte region. Scientists are just beginning to
assess the variation in types IV, V, IX, and X collagen in OA. Such alterations may explain some of
the characteristics of the disease and, perhaps, how these characteristics might influence the
water content and proteoglycan distribution within the tissue.
Question 190
Which of the following conditions associated with rheumatoid arthritis of
the cervical spine is shown in the flexion-extension views in figures 45a and 45b?

1. Cranial setting
2. Cranial subluxation
3. Odontoid fracture
4. Lysis of the arch of the atlas
5. Atlantoaxial subluxation

Answer: 5
Rheumatoid synovitis in the cervical spine causes pathologic changes in the ligaments with distention
and rupture, articular cartilage destruction, and, in bone osteoporosis, cyst formation, and erosion. The
specific deformity caused is dependent on the structure destroyed. The inflammatory cells involved in
rheumatoid inflammation have been identified in biopsy specimens at the atlantoaxial junction and
reflect the same cell types as found peripherally. Atlantoaxial subluxation is ascribed to erosive synovitis in
the atlantoaxial, atlantoodontoid, and atlantooccipital joints as well as the synovium-lined bursa between
the odontoid and the transverse ligament. In the figure (fig 10-1) contained in the text there are flexion
and extension views depicting a widening of the space between the odontoid and the posterior aspect
of the anterior ring. Atlantoaxial impaction (upward translocation of the odontoid, pseudobasilar
invagination, cranial settling, vertical subluxation) describes the settling of the skull on the atlas and the
atlas on the axis, resulting from erosion and bone loss in the occipitoatlantal and atlantoaxial joints. In the
figure (10-2) Atlantoaxial impaction is seen on a lateral x-ray. The dens is noted above the clivus and is
circumferentially eroded by synovitis. The ring of the atlas lies low on the body of C2.
In the context of viewing a lateral cervical spine to evaluate cranial setting, multiple lines can be used to
assess anatomic relationships (i.e. McRae‘s Chamberlin‘s, or McGregor‘s) McRae‘s and Chamberlin‘s use
the foramen magnum as one land mark. Commonly, however, the margins of the foramen magnum are
difficult to precisely identify. McGregor‘s line connects the posterior margin of the hard palate to the most
caudal point of the occiput and is easier to use. The tip of the odontoid should not project more than 4.5
mm above this line.
Anterior subluxation of more than 10-12 mm implies destruction of the entire ligamentous complex.
An atlantoaxial distance greater than 3.5mm is considered abnormal in an adult. The subaxial cervical
spine is affected through involvement of the facets, interspinous ligaments, and intervertebral discs
(spondylodiscitis). The initial site of destruction has been postulated to be through synovitis of the
neurocentral joints with erosion of the adjacent disc and bone causing subluxation, or through primary
facetal arthritis and ligamentous laxity causing secondary chronic discovertebraltrauma and destructive
hypermobile segmentsis associated with disc destruction and toward the C2-C3 and C3-C4 segments,
typically lack osteophytes, and often are at multiple levels, giving a stepladder appearance.
Question 191 -
A patient undergoes an acute repair of a laceration of the median nerve in
the antecubital fossa. A lack of functional recovery 6 months later is most likely
due to
1. Retrograde collapse of the endoneurial tubes
2. Irreversible atrophy of the denervated muscles
3. Elongation of the axons across the zone of injury
4. Sprouting of the axons at the neuromuscular junction
5. Misdirection of the axons across the zone of injury

Answer: 5
Functional recovery after nerve injury-The outcome of peripheral nerve injuries is quite variable. Variables
hypothesized to have an important role in determining the outcome of nerve repair include: (1) the age
of the patient; (2) the type of nerve injured; (3) the distance the regenerating axons must grow to reach
the target organ; (4) the length of the injured zone; (5) the timing of the nerve repair; (6) the status of the
target organ at the time it is reinnervated; and (7) the technical excellence of the surgeon.
Functional recovery is generally complete after a crush injury because the basement membrane
and endoneurium are left intact, and the damaged axons can regenerate within their original
endoneurial tubes and reinnervate their original target organ. After a complete lesion to the nerve,
however, functional recovery of movement is often quite poor. The loss of functional recovery probably
is related to the failure of the axons to regenerate and the misdirection of regenerating axons, which
leads to inappropriate innervation of denervated muscles. Inappropriate innervation is thought to result
in a loss in the ability to accurately recruit individual muscles and motor units within a muscle, resulting in
the loss of motor control.
Question 192
To prevent injury to the posterior interosseous nerve during the approach
for reduction and fixation of a fracture of the radial head, anterior retraction
should be performed with the forearm
1. Maximally pronated and elbow extended
2. Maximally pronated and the elbow flexed
3. Maximally supinated and the elbow flexed
4. Maximally supinated and the elbow extended
5. In neutral rotation, with the elbow extended

Answer: 2
Position of the patient-Place supine on the operating table, with the affected arm positioned over the
chest. Pronate the forearm.
Deep surgical dissection-Fully pronate the forearm to move the posterior interosseous nerve away from
the operative field. Incise the capsule of the elbow joint longitudinally to reveal the underlying capitulum,
the radial head, and the annular ligament. Do not incise the capsule too far anteriorly; the radial nerve
runs over the front of the anterolateral portion of the elbow capsule. Do not continue their dissection
below the annular ligament or retract vigorously, distally, or anteriorly, because the posterior
interosseous nerve lies within the substance of the supinator muscle and is vunerable to injury.
Dangers: Nerves-The posterior interosseous nerve is in no danger as long as the dissection remains
proximal to the annular ligament. Pronation of the forearm keeps the nerve as far from the operative
field as it possible can be. To ensure the safety of the nerve, take great care to place the retractors directly
on the bone and be careful in their placement. Because the posterior interosseous nerve actually may
touch the bone of the radial neck, directly opposite the bicipital tuberosity, placing retractors behind it
poses a risk.
Question 193 -
A 25 year-old amateur baseball player sustained a dorsal fracture-
dislocation of the proximal interphalangeal joint of his long finger. He underwent
closed reduction 3 hours ago. Examination reveals mild laxity of the radial
collateral fragment involving 30% of the volar articular surface of the middle
phalanx. Management should now include

1. open reduction and internal fixation


2. buddy taping to the adjacent index finger
3. early motion with application of a dynamic banjo splint
4. application of a cast with the hand in a ―safe position‖ for 3 weeks.
5. dorsal extension block splinting

Answer: 5
ORIF is most commonly used to treat displaced intrarticular, unstable, or unreducible injuries. Buddy
taping is used for isolated volar plate injuries. Dynamic splints are used for volar dislocations in this
presentation. Casting in the intrinsic plus position is falling out of favor for earlier range of motion options
with PIP fracture dislocations.
―Intra-articular fractures that involve the base of the middle phalanx are usually one of three types 1.
Dorsal chip fracture 2. Volar lip fracture, usually combined with a dorsal dislocation of subluxation of the
middle phalanx 3. Lateral chip fracture, representing avulsion of bone by the collateral ligament.‖
Kuczynski has suggested that the volar plate is less mobile in the PIP joint than it is in the MP joint.
―What must always occur with dorsal dislocation, however, is rupture of the volar plate. According to
Bowers, the plate is virtually always disrupted from its distal attachment into the base of the middle
phalanx. This may with or without a small avulsion chip fracture.‖
If the fracture involves more than a third of the joint or is unstable then the PIP joint must be stabilized in
a reduced position with early range of motion while restricting PIP hyperextension.
The preferred method of treatment is dorsal extension block splinting for three weeks, then protected
range of motion until united.
Question 194 -
An orthopaedic surgeon who is the developer of a knee arthroplasty
system is discussing treatment options with a patient who has tricompartmental
osteoarthritis. As a part of this discussion, the orthopaedic surgeon has an
obligation to disclose

1. The name of the manufacturer


2. The manufacturer‘s potential liability
3. The physician‘s clinical performance
4. The physician‘s materials testing data
5. Any royalties the physician receives from the manufacturer

Answer: 5
This topic is listed in Appendix D of the Code of Ethics for Orthopaedic Surgeons/American Academy of
Orthopaedic Surgeons under sections III B and C.
III. Conflicts of Interest
B. Where there are financial interests involved in the ownership of a pharmacy, rehabilitation center,
imaging equipment, surgery center, or health care facility where the orthopaedic surgeon‘s financial
interest is not immediately obvious, the orthopaedic surgeon must disclose that financial interest to the
patient and to colleagues.
C. When an orthopaedic surgeon receives anything of value, including royalties, from a manufacturer,
the orthopaedic surgeon must disclose this fact to the patient and to colleagues. It is unethical for an
orthopaedic surgeon to receive compensation (excluding royalties) from a manufacturer for using a
particular device or medication. Reimbursement for administrative costs in conducting or participating in
a scientifically sound research trial is acceptable.
Question 195 -
Which of the following nerves lying between the gluteus medius and
minimus is at risk for injury in a lateral approach to the hip?

1. Femoral
2. Obturator
3. Inferior gluteal
4. Superior gluteal
5. Lateral femoral cutaneous

Answer: 4
Femoral nerve is located in the anterior neurovascular bundle which does not lie between the gluteus
medius minimus. Obturator arises from lumbar plexus supplies and runs with the adductor muscles.
Inferior gluteal runs with the posterior neurovascular bundle which do not lie between these muscles.
The lateral femoral cutaneous arises inferior and medial to the ASIS which is anterior and medial to this
area as well.
The direct lateral approach (Transgluteal approach). No true intervenous plane (―split the fibers‖ of the
gluteus medius ―distal to the point where the superior gluteal nerve supplies the muscle‖). ―Do not (split)
more than 3 cm above the upper boarder of the trochanter.‖
Question 196
Figure 46 shows the lateral radiograph of a 5 year-old girl who has a
fracture of the left tibia as a result of minimal trauma. Examination shows no skin
lesions or birthmarks, and family history is unremarkable. Laboratory studies are
normal. Management at this time should include

1. open biopsy and plating


2. open biopsy and a long leg cast
3. open biopsy and wide resection of the tumor
4. a long leg cast and observation
5. intramedullary stabilization and observation

Answer: 4
The history of this pathologic fracture is unremarkable. What is notable, is the patient‘s age, location, and
x-ray appearance. Radiographically, there is an isolated, eccentric, intracortical osteolytic (bubble-like)
tibia shown in figure.
The differential diagnosis would include Adamantinoma (rarely seen under age 10, but commonly seen
in the tibia), Fibrous dysplasia (usually older child and commonly seen in the tibia), Fibrous dysplasia
(usually older child and commonly polyostotic), and Osteofibrous dysplasia (almost always prepubescent
in age, isolated tibia/fibula, and eccentric osteolytic lesions).
―The radiographic features are so characteristic that the diagnosis can often be made, with confidence,
from the radiographs alone, before or without histological confirmation.‖
Therefore, any open biopsy or procedure answers would be wrong for test (OITE) purposes. However, it
is not uncommon for thse to be biopsied and the pathology is also characteristic with two fundamental
patterns: fibrous tissue surrounding bone trabeculae bordered by active osteoblasts and zonal
architecture. Also, the absence of epithelia tissue would exclude adamantimona. Of note regarding
Osteofibrous dysplasia, when biopsy/resection is performed, the benign tumor commonly locally returns
if the patient is under age 10 to 15 years, but doesn‘t if older.
―Marginal subperiosteal excision should not be attempted in patients who are less than fifteen years old,
because most likely the lesion will recur. In many cases a biopsy appears to be unnecessary, because
clinically and radiographically the lesion is so typical that the physician can be reasonably confident
about the diagnosis. The occasional pathological fracture can be treated by plaster-cast immobilization.‖
Question 197 -
A 25-year-old woman who sustains a direct blow to the anteromedial
aspect of her leg while playing basketball has immediate pain and cannot walk.
Examination of the knee reveals an increase in posterior translation and external
rotation at 90 degrees of flexion. At 30 degrees of flexion, posterior translation
and external rotation decrease. Radiographs are normal. Which of the following
structures has been injured?
1. Posterolateral complex
2. Posterior cruciate ligament
3. Lateral collateral ligament
4. Posterior cruciate ligament and posterolateral complex
5. Posterior cruciate ligament and medical collateral ligament

Answer: 2
External rotation of the knee at 30 degrees of flexion averages 29 degrees with a range of 10-45.
External rotation of the knee at 90 degrees of external rotation averages 37 degrees with a range of 15-
70. There is no significant difference between men and women. External rotation of the knee at 30 & 90
degrees of flexion correlated with the amount of ligamentous laxity. Therefore, with this examination
technique, external rotation at 90 is expected to be greater than when it is at 30. Which is indicative of
an injury to the posterior lateral corner of the knee. An increase in posterior translation is indicative of a
PCL injury when performed at 90 degrees but is less specific at 30 degrees.
Question 198 -
A patient sustains a closed dorsal dislocation of the proximal
interphalangeal joint of the middle finger without an associated fracture. Closed
treatment results in a cocentric stable reduction. The finger is not immobilized.
Which of the following conditions may appear 1 year later?
1. Triggering
2. Lateral instability
3. Swan-neck deformity
4. Boutonniere deformity
5. Loss of distal interphalangeal joint flexion

Answer: 3
Swan neck deformity describes a posture of the finger in which the PIP joint is hyperextended and the
DIP joint is flexed. Initially this is a dynamic imbalance that occurs when a patient attempts maximal
digital extension. This dynamic finger imbalance can progress to a static deformity. There are many
etiologies for SND and include injuries resulting in volar plate laxity (e.g. dorsal dislocation of the PIP.),
spastic conditions such as stroke & CP, RA, fractures of the middle and proximal phx healed in extension.
Question 199 -
Examination of a 3-year-old boy who slammed his finger in a door 3
months ago reveals 0 to 40 degrees of proximal interphalangeal joint motion.
Radiographs are shown in Figures 47a and 47b. Management should consist of

1. Volar osteotomy
2. Observation only
3. Flexor tenolysis
4. Volar plate arthoplasty
5. A hinged distraction external fixator

Answer: 1
Subcondylar fractures of the proximal and middle phalynx occur at the neck of
the phalynx, usually as a result of a crush injury, and almost exclusively in the
pediatric age group. The distal fragment rotates dorsally and the degree of
displacement may be misjudged if a true lateral is not obtained. If malunion
occurs there is block to flexion. Subcondylar fossa reconstruction by removal of
bone through a palmar approach removes this boney block.
Question 201
What is the standard interval for placement of an anterolateral portal in
ankle arthroscopy?
1. Peroneus brevis to peroneus longus
2. Peroneus tertius to extensor hallucis longus
3. Peroneus tertius to superficial peroneal nerve
4. Extensor hallucis longus to deep peroneal nerve
5. Extensor hallucis longus to extensor digitorum longus

Answer: 3
As with arthroscopy of the knee, the anterolateral (AL) portal is the primary diagnostic portal used for
initial placement of the arthroscope. The AL portal is made 5 mm below the joint line just lateral to the
extensor tendons. The lateral cutaneous branch of the superficial peroneal nerve lies near this portal
region. From this approach, one can visualize the anteromedial (AM), anterocentral (AC), and most of
the AL areas of the tibiotalar joint. With the addition of laterally based external distraction
instrumentation, the surgeon can usually advance the arthroscope posterocentrally and posterolaterally
to visualize most of the articulation and the structures in the central and posterior compartments: the
intraarticular aspects and synovium of the distal tibiofibular syndesmosis, the posterior tibiofibular
ligament, the transverse ligament, and the synovial plicae that overlie the transverse ligament.

Question 202 -
A clinical trial is being conducted on a new orthopaedic device that is
different from existing devices that are moderately successful, but have frequent
complications when used to treat fractures in the elderly. To comply with
international standards for clinical trials, the investigator must include in the study
design
1. reassurance that Medicare will pay for the treatment.
2. consent forms that patients or their guardians are able to understand.
3. a detailed description of the device, omitting the fact that it is part of a study.
4. a provision that the patient‘s care will be discontinued if he or she does not
enroll in the study.
5. a provision that the study will be carried out to completion, whether or not the
device is as effective as those currently in existence.

Answer: 2
In any research on human beings, each potential subject must be adequately informed of the aims.
methods, anticipated benefits and potential hazards of the study and the discomfort it may entail. He or
she should be informed that he or she is at liberty to abstain from participation in the study and that he
or she is free to withdraw his or her consent to participation at any time. The physician should then
obtain the subject‘s freely-given informed consent. preferably in writing.
Question 203
Figure 48 shows a current lateral radiograph of a 23-year-old man who
sustained a closed femoral diaphyseal fracture 5 months ago. Treatment
consisted of placement of a retrograde femoral nail for the femoral fracture. The
patient now reports a sudden onset of pain in the midthigh and cannot bear
weight on his leg. Management should consist of
1. an onlay iliac crest bone graft.
2. limited weightbearing and observation.
3. removal of the implant and limited weightbearing.
4. removal of the implant and insertion of a reamed femoral nail.
5. removal of the implant and insertion of an unreamed femoral nail.

Answer: 4
Moed‘s article basically discusses the application of retrograde unreamed femoral nailing in selected
multiply injured patients. He reports higher rates of non-union and longer time to union than antigrade
with and without reaming and retrograde nailing without reaming. Despite this, the article maintains its
advantages: elimination of need for fracture table; decrease in intraopperative manipulation; shortened
duration of procedure; and decreased blood loss. The author discusses treatment of non-unions with this
technique insofar as he describes conversion of statically locked nails of the three delayed unions (N=22)
to dynamic constructs by removal of the proximal locking screw.
In Webb, et al. one hundred five consecutive patients with a diagnosis of delayed union (61 patients) or
nonunion (44 patients) of the femoral shaft from February 1968 to November 1983 were managed by
intramedullary reaming and nailing. The procedure was accomplished by closed techniques in 82 of the
cases. Adequate follow-up study was obtained in 101 patients; all but four showed clinical and radiologic
union at an average of 20 weeks following the procedure, with an overall union rate of 96%. The four
patients whose fractures failed to consolidate with this treatment had repeat procedures with placement
of thicker nails, and all subsequently healed.
The fracture pictured for this question is an oblique, distal 1/3 diaphyseal femur fracture with a butterfly
fragment, an axially and rotationally unstable fracture configuration. The clinical history and x-ray are
consistent with delayed union, therefore response #4 is the only sensible management option.
Question 204 -
To maximally resist apex anterior angulation in the tibia, the pins of a
unilateral external fixator should be oriented in which of the following planes?

1. Coronal
2. Sagittal
3. Anteromedial, midway between the sagittal and the coronal
4. Proximal pins sagittal, distal pins coronal
5. Proximal pins coronal, distal pins sagittal

Answer: 2
The structural and geometric fixator properties that best neutralize the prevailing anteroposterior and
transverse bending moments at a tibial fracture site were analyzed in anatomic specimens. Clinically and
mechanically, anterior unilateral frames were most effective, particularly when applied with relatively stiff
components with a maximal spread between the pins in each main bony fragment and with placement
of the longitudinal rod.
Question 205 -
Resurfacing the patella during a total knee replacement is strongly indicated
when the diagnosis is
1. Rheumatoid arthritis
2. Posttraumatic arthritis
3. Degenerative osteoarthritis
4. Osteonecrosis of the tibial plateau
5. Osteonecrosis of the medial femoral condyle

Answer: 1
A study by Boyd, et. al. was performed to determine patellar complications after TKA, with the use of an
unconstrained prosthesis, with and without resurfacing of the patella. ―Chronic pain (post-operative,
peripatellar pain) in the group that had not had resurfacing was noted in 40 (13 percent) of the 300
knees that were affected by inflammatory arthritis (rheumatoid arthritis [RA], juvenile RA and
miscellaneous subcategories), and in 11 (6 percent) of the 195 knees affected by degenerative
osteoarthrosis…A revision to resurface the patella was performed in all 51 knees that caused chronic pain
and had not had resurfacing of the patella.
Hence, chronic pain after TKA without resurfacing the patella was more common in knees affected by
rheumatoid arthritis than in those affected by degenerative osteoarthritis. The authors of the study
recommend that the patella be resurfaced when an unconstrained prosthesis is used in patients with a
diagnosis of inflammatory arthritis and to a lesser degree osteoarthrosis.
Question 206
A 34-year- woman has pain at the base of the thumb that worsens é pinching
activities. Nonsurgical treatment has failed to provide relief. Examination reveals
that the basilar joint is hypermobile, tender and painful when stressed. A
radiograph of the trapeziometacarpal joint shows normal contour with widening
when compared with the opposite side. Management should consist of
1. Trapeziometacarpal arthrodesis
2. Osteotomy of the thumb metacarpal
3. Arthrotomy and joint debridement
4. Ligament reconstruction using one half of the flexor carpi radialis
5. Trapezium resection, tendon interposition, and reconstruction of the ligament
Answer: 4
Painful instability of the thumb carpometacarpal (CMC) joint as manifested by ―idiopathic hypermobility
of the basal joint is not uncommon, particularly in women, and would seem to be a significant factor in
producing the arthrosis that so frequently afflicts this joint.‖ ―Extra-articular ligament reconstruction to
stabilize the thumb carpometacarpal (CMC) (basal) joint by routing a portion of the flexor carpi radialis
(FCR) through the base of the thumb metacarpal … is recommended only for patients in stage I and
stage II disease (i.e., patient‘s having zero to only slight cartilage attrition). With 95 % of stage I and stage
II patients having achieved and maintained good or excellent results, (restoration of stability, pain
reduction and possibly retarding joint degeneration) … we feel the procedure has proved predictable
and durable. Despite limited success in patients with significant articular deterioration (stage III and stage
IV), we do not recommend ligament reconstruction alone in such cases. Instead we would recommend
articular resurfacing or implant arthroplasty that would depend on the number of trapezium surfaces
involved.‖
Question 207 -
The change over time in strain of a material under a constant load is
defined as
1. Creep
2. Relaxation
3. Energy dissipation
4. Plastic deformation
5. Elastic deformation

Answer: 1
Question 208 -
What is the usual mechanism of injury for the fracture shown in Figures
49a and 49b?

1. bending
2. axial loading
3. high-speed rotation
4. direct impact from anteromedial
5. crush from anteromedial to posterolateral

Answer: 2
Figures 49a and 49b show a severely comminuted, displaced fracture of the distal tibial metaphysis with
intra-articular extension, i.e. pilon fx, or the so-called ―distal tibial explosion fracture.‖ According to the
article by Kellam JF and Waddell JP, there are ―two mechanisms that can cause this fracture, a rotational
pronation dorsiflexion force, and an axial compression force with the foot in either neutral or dorsiflexion
at the moment of impact. Because of these mechanisms of injury, two fracture patterns were observed in
the series of patients reported. The rotational pattern results in several large metaphyseal fragments, and
usually a fibular fracture of a short oblique type above the level of the ankle joint…The second and
unfortunately the more common fracture pattern is caused by a severe axial compression force which
causes marked comminution of the distal tibial metaphysis, loss of bone substance due to impaction,
superior migration of the talus, and loss of the ankle cartilage space.‖
Question 209 -
Gamma ray irradiation for sterilization of ultra-high molecular weight
polyethylene in an oxygen environment can have what effect on the material?

1. Increase stiffness
2. Increase fracture toughness
3. Increase fatigue strength
4. Decrease mechanical strength
5. Decrease wear rate

Answer: 4
Gamma irradiation of Ultrahigh Molecular Weight Polyethylene leads to free oxidation with resultant
breaking of polymer chains, changes in the crystalline structure, and deterioration of the mechanical
properties of the polymer. Fracture toughness, fatigue strength, and mechanical strength all decrease
while the wear rate of irradiated UHMWPE increases. Stiffness is another matter. Initially after irradiation,
crosslinking and stiffness actually increase, but in the long term stiffness will decrease as crosslinks
continue to break secondary to irradiation induced oxidative damage.
Question 210 -

The radiograph shown in Figure 50a and the CT


scan shown in Figure 50b reveal a lesion in the left femoral neck of a 12-year-old
boy who has pain in the left hip. The most likely cause of the osteopenia of the
left proximal femur is

1. disuse osteopenia
2. paraendocrine effect of the tumor
3. abnormally increased density on the right side
4. side effect of the treatment of the lesion
5. extensive tumor involvement of the left hip

Answer: 1
Figure 50A: AP pelvis with a 1 cm lucency in the calcar region of the femoral neck and diffuse osteopenia
of the proximal femur. Figure 50B: CT scan of the same patient showing a well demarcated lcm lesion in
the femoral neck with an obvious nidus. Careful evaluation of the radiographs is critical in this question.
First the obvious osteoid osteoma must be recognized. From there answers 2 and 3 can be eliminated.
Neither radiograph reveals any sign that treatment has occurred, and the CT scan shows no signs of
extensive involvement, thus answer 5 should also be eliminated. The real key is believing that such
diffuse osteoporosis could occur in such a young child, as Jones described in his article from 1969.
Question 211 -
Which of the following neurovascular structures is at greatest risk during
the introduction of acetabular component fixation screws during total hip
replacement?

1. Sciatic nerve
2. Superior gluteal artery
3. Profunda femoris artery
4. Femoral artery and nerve
5. External iliac artery and vein

Answer: 5
Wasielewski et al found on reviewing the literature that vascular injuries during acetabuIar screw
placement are an uncommon yet devastating complication of total hip arthroplasty. Damage to the
external iliac artery was the most frequent injury yet injury to the external iliac vein and the superior
gluteal artery has also been reported Based upon their anatomic study and development of a quadrant
system they found that the posterior superior and posterior inferior quadrants of the acetabulum are the
safest locations for screw placement because of better bone stock as well as less neurovascular structures
as compared to the anterior quadrants.
Question 212 -
Which of the following is considered the minimum ankle/brachial ischemic
index level that will allow satisfactory healing of wounds in patients who are
candidates for amputation of the foot and ankle?

1. 0.25
2. 0.35
3. 0.45
4. 0.55
5. 0.65

Answer: 3
Question 213
In patients older than age 50 years who experience shoulder dislocation
or proximal humerus fracture, the incidence of associated neurologic abnormality
documented by electromyogram is as high as
1. 10%
2. 20%
3. 50%
4. 70%
5. 90%

Answer: 3
When diagnosed by EMG, the prevalence of axillary and other types of nerve
lesions in glenohumeral dislocations and humeral neck fractures is 20 to 30%. In
patients older than 50 years, it is as high as 50%. Prospective study in Sweden, 88
patients over 14 months. Purpose was to study clinically and
electromyographically patients with humero-scapular joint trauma and obtain
info. about the frequency and distribution of nerve lesions. 50% of patients over
50 years old had a nerve lesion as compared to 15% of patients below 50 yrs.
Deltoid (axillary n.) was the only nerve studied. There were no differences
between dislocations and fractures with respect to the total number of nerve
injuries.
A nerve lesion should always be suspected in a patient who does not recover
functionally within one to two months, especially if there has been a dislocation
and the patient is more than 50 years old. An EMG can be done @ 3 wks. If a
nerve lesion is found, than a more active P.T. program should be used.
Prospective study in Finland, 238 patients over 3 years. Purpose was to
analyze the early complications of primary humeral dislocation.
Complications were divided into nerve lesions and rotator cuff ruptures. 38
of 50 nerve lesions occurred in patients over 50 years old. and all cuff ruptures
occurred in patients over 50.
Question 214
ofhat parameter is most commonly used to estimate the maximum tension a
muscle can generating?
1. Length
2. Moment arm
3. Total volume
4. Physiologic cross-sectional area
5. Distribution of slow and fast twitch fibers

Answer: 4
The mass or volume of a muscle is proportional to its work capacity, and the fiber length of a muscle is
proportional to its potential excursion. By dividing the fiber length into the volume of each muscle, the
cross-sectional area of the muscle is determined.
The concept of physiologic cross section of a muscle from Weber and Fick, identifies the critical importance
of the cross sectional area of all the fibers of a muscle as proportional to maximum tension. (Relationship
between muscle size and muscle strength).
Question 215 -
Demyelination diseases as multiple sclerosis and Guillain-Barre $ create
neurologic symptoms by
1. decreasing initiation of action potentials.
2. increasing action potential amplitude.
3. blocking the opening of gated sodium channels.
4. decreasing the number of functional motor units.
5. slowing or stopping action potential propagation through the axon.

Answer: 5
These diseases cause demyelinated regions of the axon and thus have a higher capacitance and lower
membrane resistance (opposite of normal) and thus affects an action potential‘s saltatory propagation.
When an action potential is propagate down a myelinated axon and reaches a demyelinated region its
conduction becomes slowed or may even stop. This loss of conduction had effects on behavior.
Question 216
Figure 51 shows the standing AP radiograph of a 56-year old woman who
has multiple toe deformities and pain beneath the metatarsal heads. Shoe
modification has failed to provide relief. In addition to correction of the proximal
interphalangeal joint deformities, surgical treatment should consist of

1. resection of the metatarsal heads of the first through fifth toes.


2. Silastic MP joint arthroplasties of the first through fifth toes.
3. fusion of the hallux MP joint and resection arthroplasty of the 2 nd through
fifth metatarsal heads.
4. fusion of hallux MP joint and distal osteotomy of the 2nd through 5th MT.
5. plantar condylectomy of the 2nd through 5th MT heads & resection of proximal
phx of the hallux.

Answer: 3
Surgical correction of severe rheumatoid forefoot deformities with resection arthroplasties of the lesser
metatarsal phalangeal joints and arthrodesis of the first metatarsal phalangeal joint resulted in a significant
long-term improvement with respect to shoe wear, pain and the ability to stand and walk in 95% of the
patients. There was minimal recurrence of the deformity. Previous procedures attempt to correct the lesser
MTP joint deformities and a resection-type arthroplasty procedure to the 1st MTP joint. Recurrent
symptomatic deformities were found in the latter.
A modification was then used that maintained the proximal phalangeal bases and used K-wires to
fixate the MTP arthroplasty and IP joints which resulted in improved cosmetic result and simplified post-op
management. Equal results were seen w/ no increase in recurrence or complications.
Question 217
An active, right-handed 71-year-old woman fell on her left shoulder and
sustained the injury shown in the radiographs in 52a and 52b and the CT scan in
52c. Management should consist of

1. hemiarthroplasty
2. open reduction and internal fixation
3. closed reduction and percutaneous pinning
4. a sling and early pedulum exercises
5. a sling and swathe for 6 weeks, followed by shoulder rehabilitation
Answer: 1
Prosthetic hemiarthroplasty is the accepted form of treatment for badly displaced fractures and fracture
dislocations, including 4-part fractures, head splitting fractures and fractures with impression defects
involving more than 45 % of the humeral head. Displacement is classified as >1cm or angulated > 45
degrees. About 15% of all shoulder fx‘s are considered displaced.

Question 218

Figure 53 shows the MRI scan of a 53-year-


old carnival worker who has pain and swelling in the left shoulder as a result of
attempting to stop a roller coaster car with his arm. Examination reveals
decreased ROM, apprehension, and inability to move the dorsum of his hand
away from his back. Treatment should consist of
1. open acromioplasty
2. open Bankart repair
3. open subscapularis tendon repair
4. inferior capsular shift
5. a supervised physical therapy program

Answer: 3
This patient has an acute tear of the subscapularis tendon both by MRI and physical exam. Treatment of
choice is open repair. Nonoperative treatment is not indicated.
Question 220 -
A 45-year-old man sustains the shoulder injury shown in the radiographs
in Figure 55a and 55b and the CT scan in Figures 55c and 55d. Management
should consist of
1. a sling and swathe, with pendulum exercises in 10 days
2. open reduction and internal fixation through an anterior approach
3. open reduction and internal fixation through a posterior approach
4. immobilization with a splint in 45 degrees of abduction for 6 weeks
5. arthroscopically assisted reduction and percutaneous screw fixation
Answer: 3
Displaced intra-articular fractures of the glenoid fossa, as in this case, are best treated with open reduction
and internal fixation through a posterior approach. ORIF through an anterior approach is very difficult and
is not recommended. Significant disabilities are seen if these fractures are treated conservatively including
chronic instability and DJD..............................................................
Question 221 -
Figure 56 shows the radiograph of a 38-year-old patient with polyarticular
rheumatoid arthritis who has severe pain in the shoulder. Non-surgical treatment
has failed to provide relief. Treatment should now consist of

1. Arthrodesis
2. Repair of the rotator cuff
3. Replacement of the humeral head
4. Resection arthroplasty
5. Total shoulder arthroplasty

Answer: 3
Long term prospective study involving 67 humeral head replacements for OA &RA. "Based on this
experience, we would recommend that humeral head replacement alone be used sparingly in patients
with OA or RA. Certainly in patients who have glenoid bone deficiency precluding placement of a glenoid
component…" as is apparent in this radiograph.
Question 222
Figure 57a and 57b show the radiographs of a 57-year-old man who has
pain in

the ulnar side of the wrist and


hand. Examination shows tenderness at the base of the hypothenar area.
Additional diagnostic testing should include
1. AP and lateral radiographs of the elbow
2. Diagnositc arthroscopy
3. Aspiration of joint fluid
4. An erythrocyte sedimentation rate and CBC
5. A diagnostic lidocaine injection

Answer: 5
"The major symptom of dysfunction at the pisotriquetral joint is pain at the hypothenar eminence. It is often
vague and poorly localized. The patient may complain of clicking or locking sensation with certain motions
of the wrist. Careful clinical evaluation reveals that there is usually well localized tenderness to pressure over
the pisiform. Side-to-side passive motion of the pisiform may reproduce pain and occasionally crepitus.
Forced hyperextension of the wrist and resistance to palmar flexion and ulnar deviation, as well as to
pronation and supination, may reproduce the symptoms. Excessive mobility of the pisiform is a common
finding with pain often produced at the extremes of its mobility. A small injection of a local anesthetic agent
into the isotriquetral joint at the ulnar aspect of the wrist with relief of symptoms helps confirm the
diagnosis."
Question 223 -
Acondroplasia and other chondrodysplasias are caused by mutations in
the
receptors of which of the following families of growth factors?
1. Insulin-like growth factor (IGF-1)
2. Fibroblast growth factor (FGF-1)
3. Platelet-derived growth factor (PDGF)
4. Transforming growth factor beta (TGF-B)
5. Bone morphogenetic proteins (BMP)

Answer: 2
"The gene responsible for achondroplasia has been mapped to chromosome
4p16.3 (ref 7,8); the gentic interval encompassing the disease gene contains a
member of the fibroblast growth factor receptor (FGFR3) family which is
expressed in articular chondrocytes.
Question 224 -
The diagnostic distinction between a benign enchondroma and a low-
grade
intramedually chondrosarcoma is based primarily on the
1. clinical history and radiographic findings.
2. technetium bone scan
3. flow cytometry pattern of extracted chondrocytes
4. immunohistochemical staining patterns of a biopsy specimen
5. histologic features of a biopsy specimen stained with hematoxylin-cosin

Answer: 1
"The diagnostic strategies for benign bone tumors center on the initial radiographic presentation." … "For
example calcified, lytic phylangeal lesion strongly suggests the presence of cartilage that is consistent with
enchondroma."
Question 225 -
The flap outlined in Figure 58 is based on septocutaneous perforators
from what artery?

1. Radial
2. Radial recurrent
3. Posterior interosseous
4. Superior ulnar recurrent
5. Superficial radial circumflex

Answer: 3
This is known as the posterior interosseous forearm island flap, a fasciocutaneous or fascial flap based on
perforation of the posterior interosseous artery. It can be proximally or distally based. Indications include
two main categories: Inadequate scar or scar contraction; Complex trauma with gross skin loss, either acute
or post primarily referred. The graft can be transported either proximal to the elbow or distally to the hand,
Palm or wrist, or for coverage of amputated fingers.

Question 226
What defect in collagen synthesis is caused by a lack of vitamin C?
1. Impaired hydroxylation of proline
2. Failure of cleavage in procollagen
3. Defective binding sites for hydroxyproline
4. Failure to incorporate glycine into the helix
5. Diminished production of collagen through the rough endoplasmic reticulum

Answer: 1
Scurvy results from vitamin C deficiency. Metabolically, vitamin C deficiency produces a decrease in
chondroitin sulfate synthesis (enzymatic impairment of the conversion of glucose to galactosamine) and a
deficiency in collagen synthesis by impaired hydroxylation of proline. The greatest problem in collagen
synthesis is seen in the metaphysis. The metaphyseal bone becomes weakened and microfracture,
hemorrhage, debris, and fibrous tissue accumulates. Collapse of the metaphysis and continued lateral
growth produces the typical spurs associated with scurvy.

Question 227 -
While under a physician‘s care, a 45-year-old man verbally abuses the staff
and nurses who are attempting to carry out orders. A decision to discharge the
patient is best carried out by
1. Asking the legal staff to seek a court injunction
2. Copying the patient‘s chart and giving it to him as he leaves
3. Having the patient sign a written legal contract that specifies acceptable
behavior
4. Continuing care of the patient until an appropriate referral can be arranged
5. Transferring the patient to another orthopaedic surgeon without disclosing
the real
reason for why the patient is being transferred

Answer: 4
The other foils in this answer would be both unethical and would be a violation
of ―anti-dumping‖ laws. This would further open the physician, hospital and staff
to possible litigation.
Question 228 -
A consecutive series of 50 patients is randomized to receive either
treatment A or treatment B. At a 10-year follow up, patient satisfaction with
treatment is measured. Which of the following statistical calculations will provide
the most information regarding the magnitude of possible differences between
the two groups of patients?

1. Meta-analysis
2. Confidence interval
3. Analysis of variance (ANOVA)
4. Statistical significance (p-value)
5. Survivorship analysis (Kaplan-Meier)

Answer: 2
The confidence interval provides you a range of values that with a given level of confidence (usually 95%)
contains the truc value. The confidence interval also reflects the limitations of data presented (i.e. if no sepsis
occurred during a two year follow-up of THR, should we believe there will never be any recurrences of
sepsis rate if we perform a hundred of these exchanges in the future.) Confidence internals are also
essential when presenting long-term results based on survivorship curve. Without confidence intervals the
picture presented to the readers can be misleading. In these cases more data are needed before any
proelic (?) statements can be made.

Question 229 -
A 19-year-old man sustains a complete spinal cord injury at the C7 level as
a result of diving into a lake. He has a blood pressure of 90/50 mm Hg, a pulse of
60/min, and respirations of 20/min. These values most likely signify

1. Spinal shock
2. Neurogenic shock
3. Hypovolemic shock
4. Pulmonary embolism
5. Fat embolus syndrome

Answer: 2
Neurogenic shock is defined as vascular hypotension with bradycardia as a result of spinal injury. The first
few minutes after spinal cord injury are associated with hypertension and tachycardia, with a subsequent
drop in pressure and pulse rate.
Question 230 -
Figures 59a and 59b show the plain radiographs, and Figures 59c and
59d show the CT scan of a 77-year-old woman who has had pain in her back and
both buttocks for the past 6 months. She reports that the pain radiates down her
right thigh and leg when she is standing. What is the most likely diagnosis?

1. Lumbar spinal stenosis


2. Metastatic disease of the spine
3. Rheumatoid lumbar spondylitis
4. Isthmic spondyloloisthesis
5. Degenerative spondylolisthesis at L4-5 and L5-S1

Answer: 1
DISCUSSION: Plain radiographs of this patient's Lumbar spine show degenerative changes. CT scan shows
narrowing of the spinal canal and the patient's symptoms are consistent with lumbar stenosis. Measuring
the AP diameter of the osseous canal, by CT, yields a correct diagnosis only 20% of the time. Whereas
measurements of the cross sectional area of the dural sac by CT or of the AP diameter of the canal by
myelography should lead to a correct diagnosis in 83% of patients.
Question 231
A 65-year-old man has had ―catching‖ in front of his knee since he had a
total knee arthroplasty 9 months ago. Examination reveals a palpable and
audible snap in the anterior aspect of the knee at about 40 degrees of flexion as
the knee is being actively extended. A radiograph of the prosthetic knee will most
likely show
1. Patella alta
2. A metal-backed patella
3. Varus malalignment of the knee
4. A posterior cruciate-substituting femoral component
5. Lateral subluxation of the patella on a Merchant‘s view

Answer: 4
The patellar ―clunk‖ syndrome is an infrequent complication of TKA. It is diagnosed clinically by a clunking or
clicking sensation or sound as the flexed knee is extended usually at about 30-40 degrees of flexion.
Pathologically, the clunk is produced by a suprapatellar fibrous nodule seen superior to the patellar
component at re-operation. This nodule has been seen to catch in the intercondylar notch in primarily first
generation TKAs. Current component designs have decreased this phenomenon through better
engineering of femoral components. Treatment is by arthroscopic debridement or open arthroplasty
resection. The nodule may be recurrent.
Question 232
Which of the following radiographic findings indicates that the injury to
the great toe shown in Figures 60a and 60b should be reducible by closed
manipulation?

1. The sesamoids are separated


2. The sesamoid is fractured
3. The proximal phx is on the neck of the metatarsal
4. The dislocation is dorsal and centered
5. The proximal phalanx is hyperextended

Answer: 1
In the case of first MP joint dislocation, though rare, there is a complete disruption of the intersesamoid
ligament, at separation of the sesamoid seen or x-ray (Type II). This dislocation can be readily reduced. A
Type I lesion/dislocation yields no disruption of the sesamoid mass though a dislocation is present. This type
I lesion are usually irreducible if attempted closed.

Question 233
Osteolysis, after total knee arthroplasty performed without cement, most
often occurs in the
1. Patella
2. Tibial stem
3. Distal femoral interface
4. Posterior femoral interface
5. Sites of screw fixation for the tibia

Answer: 5
As stated in the above article, the number one location for osteolysis is at the sites of screw fixation for the
tibial component. Development of osteolysis on the tibial side of the implant may be influenced by three
factors. First gravity and weight bearing through the medial side of the knee tend to localize the debris
particulate polyethylene on the tibial side. Second, on the femoral side if the osteolytic process is initiated
along the implant-bone interface, the flanges of the femoral implant obscure a radiographic diagnosis.
Finally, the addition of screws to the tibial implant provide avenues for the migration of debris into the bone.
In the patients with osteolysis all had very large amounts of polythylene and metal particles less than one
micrometer in size leading to intense histiolytic response.
Question 234

Figure 61 shows the radiograph of a 28-year-old


professional football player who sustained a hyperextension injury to the great
toe. He continued to play with pain and loss of push-off strength. What is the
most likely diagnosis?
1. Hallux rigidus
2. Fracture of the sesamoid
3. Disruption of the plantar plate
4. Osteonecrosis of the metatarsal head
5. Rupture of the flexor hallucis longus

Answer: 3
Upon review of the radiograph give, there is no evidence of fracture or osteonecrosis. Upon review of the
article above the most likely diagnosis is ―Turf-Toe‖ or the disruption of the plantar plate. The patient has
mechanism of injury, hyperextension and sequelae, decreased push-off strength, which is consistent with
this diagnosis. Rupture of the FHL would most likely result in inability to continue playing while hallux
rigidus is a potential chronic sequelae with associated dorsal osteophyte formation.
Question 235
Figures 62a and 62b show the radiographs of a 50-year-old man who has
a long-standing history of wrist pain. What is the most likely cause of pain?

1. Gout
2. Sepsis
3. Old trauma
4. Rheumatoid arthritis
5. Charcot arthroplasty

Answer: 3
With review of the article below and close examination of the radiographs, the most likely cause of this
patients pain is a SLAC wrist. The #1 cause of SLAC wrist is old trauma. The article below reviewed more
than 4000 wrist radiographs and concluded that the normal progression of the SLAC wrist follows these
guidelines: Degenerative changes first occur between the tip of the radial styloid and the scaphoid then
progress along the scaphoradial joints. The radius-lunate joint is shared as the process progresses to the
capitate-lunate joint.
Question 236
Figure 63 shows the AP radiograph of the arm of a 9-year-old boy who
has pain in his right shoulder as a result of falling off his bicycle. The most
appropriate next step in management is

1. Immobilization with a sling


2. Aspiration and steroid injection
3. Biopsy, curettage, and allograft bone grafting
4. Percutaneous Kirschner wire fixation
5. Percutaneous injection of autogenous bone marrow

Answer: 1
Upon review of the radiograph in fig. 63 it has the appearance of a classic
Unicameral Bone Cyst of the proximal humerus of a skeletally immature
individual. Also noted is a nondisplaced fracture of proximal humerus through
the cyst. The article above gives accepted treatment protocols for unicameral
bone cysts:
Radiographically Active Cyst (age 4 to 8 years old with pathologic fracture)
1. Align and immobilize
2. Observe for spontaneous healing
3. Reassess after 2 to 4 months and if recurrent, follow procedure for pathological
fracture of radiographically active cysts described in the article above.
Question 237
An elderly woman with radiographic evidence of spinal stenosis reports difficulty
walking and calf pain that is relieved by rest and a change of position. The most
likely cause of pain is ischemia of the
1. Nerve roots
2. Spinal cord
3. Sciatic nerve
4. Peroneal nerve
5. Conus medullaris

Answer: 1
Pain and numbness in the buttock, thigh and legs during walking or prolonged standing in the upright
posture-that is relieved by various postures-is caused by spinal stenosis. Neurogenic claudication may result
from nerve root ischemia secondary to increased epidural pressure, intermittent compression of nerve roots
and venous congestion.
Question 238 -
The concept of an ―effective joint space‖ surrounding a prosthetic hip
replacement refers to the

1. Extent of bone penetrated by reaming, prosthetic components, or acrylic


cement
2. Periprosthetic regions that are accessible to joint fluid and particulate wear
debris
3. Enveloping pseudocapsular scar tissue that develops around a prosthetic hip
following surgical capsulectomy
4. Soft tissue and bone that are potentially contaminated with bacteria in the
setting of a prosthetic joint infection
5. Potential intrapelvic joint space communication created by acetabular fixation
screws or intrapelvic extruded acrylic cement

Answer: 2
The term ―effective joint space‖ was initiated in the article that this question was referenced from. It is
defined as all the regions that are accessible to joint fluid. The significance of the effective joint space is that
patterns of joint fluid flow (preferential flow) will determine the concentration and pattern of particulate
wear debris. Where there is wear debris there is the potential for lytic and linear bone loss secondary to
macrophage concentrations.
Question 239 -
Almost 50% of all long-term complications following total knee
arthroplasty are due to
1. Early and late infection
2. Periprosthetic fracture of the femur
3. Failure of the patellofemoral and extensor mechanisms
4. Aseptic loosening of cementing tibial components
5. Asceptic loosening of cemented femoral components

Answer: 3
50% of failures are due to the patellofemoral component and extensor
mechanisms. These include failure of the metal backed components,
patellofemoral instability, component loosening, patellar fracture, osteonecrosis,
and failure of the extensor mechanism. The incidence of the other choices are far
less.
Question 240 -
A 6-year-old boy sustained a fracture of the medial malleolus as a result of
twisting his ankle. The fracture line extends from the joint surface proximally and
medially to include a small fragment of the metaphysis, with a displacement of 3
mm. Management should consist of

1. Acceptance of the current position of the ankle


2. Open reduction and fixation in the epiphysis only
3. ORIF with a small-fragment T-plate from the malleolus up to the metaphysis
4. Closed reduction by eversion of the ankle and application of a bivalved long
leg cast
5. CRUGA and fixation with a malleolar screw from the malleolus into the
metaphysis

Answer: 2
Salter-Harris III and IV fractures of the medial aspect of the distal tibia commonly cause growth disturbances,
such as shortening and varus angulation, unless an anatomical reduction of the physis is obtained. The
authors of the reference (Drs Kling, Bright, and Hensinger) felt that these fractures needed to be ―perfectly
reduced‖ to minimize any gap, primarily to prevent a bone bridge from forming. According to Spiegel, any
of these fractures with 2 mm or more should be openly reduced.
The authors felt that ―anatomical reduction‖ must include closure of the physeal gap and restoration of
rotational alignment of the fracture fragment. They recommended that these fractures be treated with
ORIF, consisting of small, smooth Kirschner wires from one epiphyseal fragment to the other. The pins
should diverge or converge to prevent any slippage or rotation. They do not recommend crossing the
physis with fixation devices, for this may lead to further injury.
Question 241
A 37-year-old laborer sustained a fracture of the posterior acetabular wall.
Two years following operative management, the patient reports severely limited
hip motion and back pain. Radiographs reveal extensive mature heterotopic
ossification with preservation of the hip joint space. Management should now
consist of

1. Resection arthroplasty and local radiation


2. In situ fusion of the hip
3. Excision of heterotopic bone, total hip arthroplasty, and oral indomethacin
4. Excision of heterotopic bone and local radiation
5. Excision of heterotopic bone, hemiarthroplasty, and oral indomethacin

Answer: 4
Heterotopic ossification is very common after acetabular fracture surgery. It is most common with the
extensive exposures, rates up to 45-100%. The HO is most extensive when these approaches are utilized
and no prophylaxis is provided. The ilio-inguinal approach has the least common incidence of HO. Routine
prophylaxis consists of either 1) Indomethacin 25 mg tid for 4-6 weeks, beginning POD #1 or 2) Low dose
irradiation 1000 rads in divided doses or 700 rads single dose, begun before POD #4. Surgical excision is
only considered when the HO severely reduces hip mobility. It is recommended to have a preop CT scan
for your surgical plan, utilize the initial incision and to use great caution around the (unreadable).
Question 242
A 4-year-old boy who was struck by a motor vehicle has a right
pneumothorax requiring placement of a chest tube, an open (Gustilo) grade IIIA
midshaft fracture of the left femur, and a closed displaced subtrochanteric
fracture of the right femur. He also sustained a subdural hematoma that requires
neurosurgical intervention. In addition to irrigation and debridement of the left
femur, treatment should include

1. Closed reduction of both fractures and immediate spica casting


2. Bilateral skin traction for 3 weeks, followed by spica casting
3. External fixation of both femora
4. External fixation of the left femur and a long leg cast brace for the right femur
5. External fixation of the left femur and use of a reamed intramedullary nail in
the right femur

Answer: 3
In children ages 3 to 5, who have sustained multiple trauma, the treatment priority is initially their more
serious life threatening injuries. Once other injuries are stabilized, then femoral fractures can be treated.
External fixation is rarely needed in this age group, except in the multi-trauma patient with ipsilateral
fractures or contralateral fractures. It is also useful in patients with head injuries, in open fractures, and when
traction has not prevented significant shortening and angulation of the fracture.
Question 243
A 6-year-old boy has had increasing pain and a mass in the suprapatellar
region of the right femur for the past week. Examination of the mass reveals it
may be firm, immobile, and tender to palpitation. The patient has no systemic
symptoms. Laboratory studies show a WBC of 7000 per cubic millimeter, a
hematocrit of 40%, and an erythrocyte sedimentation rate of 10 mm/hr.
radiographs are normal. Figures 64a and 64b show saggital and axial T1-
weighted MRI scans. Figure 64c shows frozen section pathology of the biopsy
specimen. What is the most likely diagnosis?

1. Synovial sarcoma
2. Soft-tissue abcess
3. Rhabdomyosarcoma
4. Eosinophilic granuloma
5. Nodular pigmented villonodular synovitis

Answer: 2
Multiple hints in this history, MRI and pathology section leads to the diagnosis of soft tissue abscess. The
sarcomas are slow growing and mostly are asymptomatic. The mass is tender and enlarging over the past
week. PVNS would give the patient a painful boggy joint and this mass is supracondyler. Esinophilic
granuloma would give a punched-out lesion in the long bones on the plain radiographs. The best clue is
the slide given which shows inflammatory cells. PVNS would show hemosiderin stained giant cells, synovial
sarcoma would reveal a biphasic pattern of spindle cells, E.G. would show eosinophils and histiocytes, and
rhabdomyosarcoma would have cross striation within the tumor cells.
Question 244
The bending stiffness of a slotted stainless steel intramedullary nail will be
increased most by
1. Changing to a titanium nail
2. Changing to a nonslotted nail
3. Changing the cross-sectional shape of the nail
4. Increasing the diameter of the nail by 3 mm
5. Increasing the diameter of the interlocking screws

Answer: 4
To increase stiffness (bending) of an IM nail the diameter must be increased. The moment of inertia is
increased by the 4th power of the diameter. The modulus of elasticity of titanium alloy is half of stainless
steel. Cross sectional shape influences fixation. Interlocking screws influence rotation.
Question 246 -
Figures 66a and 66b show the radiographs of an 8-year-old girl who has a
form, immobile mass of her middle finger at the proximal interphalangeal joint.
Figure 66c shows the histopathology of the biopsy specimen. What is the most
likely diagnosis?

1. Fracture healing
2. Chondrosarcoma
3. Periosteal chondroma
4. Periosteal osteosarcoma
5. Dysplasia epiphysealis hemimelica

Answer: 3
Periosteal chondroma is a benign chondroid tumor that arises under or in the periosteum on the surface of
cortical bone. Has a scalloping of the cortex with a well defined margin between the tumor and bone. The
tumor bed shows a variable amount of sclerosis. Also a variable amount of calcification exists. Histologically,
as in this case, there is chondroid matrix. The predilection for the proximal end of long bones, particularly
the humerus, and the bones of the hands, has been noted. This tumor must be differentiated from
periosteal chondrosarcoma, which penetrates the cortical bone and is generally larger with a soft tissue
mass. Histology will also show more polymorphism and hyperchromatic nuclei of chondrosarcoma.

Question 247
To be an expert orthopaedic witness in a trial, an orthopaedic surgeon
must

1. Demonstrate competence in the subject of the case


2. Be fellowship trained in the subject of the case
3. Be paid on a contingency basis
4. Be board certified by the American Board of Orthopaedic Surgery
5. Have been involved in the case as a consultant

Answer: 1
An expert witness need not be involved in the case, be certified, or fellowship trained. The only requirement
is that the surgeon show competency in the subject matter. There are very nonspecific requirements to be
called to the stand as an ―expert witness‖.

Question 248 -
A 12-year-old Little League pitcher has had pain in the dominant shoulder
for the past week that prevents him from pitching. Examination reveals normal
strength, full range of motion, normal stability, and mild tenderness about the
proximal humerus. Radiographs with comparison views of the opposite shoulder
show widening of the proximal humerus physis. Management should include

1. Diagnostic arthroscopy
2. Arthroscopy and subacromial decompression
3. Reduction and fixation of the proximal humeral epiphysis
4. Temporary cessation of throwing
5. Physical therapy for rotator cuff strengthening

Answer: 4
Little League‘s shoulder is a stress fracture of the proximal humeral epiphyseal plate. The classic history is the
adolescent pitcher who has increased his pitching level and now has poorly localized aching shoulder pain
after attempts to throw. PE will show discomfort at the proximal humerus and weakness. Early x-ray may
show no pathology. Later it shows physis widening, fragmentation, demineralization, periosteal stripping,
callus.
Treatment requires cessation of repetitive physeal stress. There is no long-term sequelae. The
athlete can return to same sports in the following season with emphasis on preseason conditioning.
Question 249 -
The lateral fragment of bone (Segond fracture) associated with an injury of
the anterior cruciate ligament is the result of an avulsion of the
1. Oblique popliteal ligament
2. Lateral capsule
3. Popliteal tendon
4. Fibular collateral ligament
5. Posterior oblique ligament

Answer: 2
The segond fragment is avulsed by three fibers which are under tension to this area at proximal lateral
aspect of tibia. These fibers are intimate with lateral capsule consisting of 1) posterior fibers of anterolateral
femoratibial ligament 2) the anterior expansion of arcuate ligament 3) biceps tendon expansions from
posteriolateral direction.
Question 250 -
Successful healing of a meniscal repair is most likely associated é which of the
following tear patterns?
1. Radial tear
2. Parrot-beak tear
3. Vertical tear in the ―red-red‖ zone
4. Vertical tear in the ―red-white‖ zone
5. Vertical tear in the ―white-white‖ zone

Answer: 3
The blood supply to the meniscus has been well documented, and is primarily limited to the horns and
outer one third of the meniscus. Both animal and human studies have clearly demonstrated healing of
tears in the outer one third of the meniscus.
Arnoczky et al described the vascular supply to the meniscus and showed that tears within the outer one
third of the meniscus have the best healing potential. Rim width is the most important prognostic factor for
meniscal repairs; the vascular one third of the meniscus is most conductive to healing. Scott et al found
better results when the tear was within 2mm of the meniscosynovial junction. Stone et al found improved
healing with rim widths of 6mm or less. Tenuta et al found best results within a rim width of 3mm or less.
Question 251
At what position is the anterior inferior glenohumeral ligament the
primary constraint to anterior glenohumeral translation?
1. 0 degrees of abduction, with neural rotation
2. 40 degrees of flexion and 60 degrees of internal rotation
3. 45 degrees of flexion and 45 degrees of external rotation
4. 90 degrees of abduction with neutral rotation
5. 90 degrees of abduction and 90 degrees of external rotation
Answer: 5 According to Turkel‘s classic article:
11. At 0 degrees of abduction, the subscapularis stabilizes the joint;
12. At 45 degrees of abduction, stability provided a combined effect of the subscapularis, the middle of
the glenohumeral ligament and the superior fibers of the inferior glenohumeral ligament;
13. At 90 degrees of abduction, the inferior glenohumeral ligament affords stability, especially with
external rotation.
Dynamic stabilizers of the shoulder are the rotator cuff and the negative intraarticular pressure of the
glenohumeral joint. The static stabilizers are comprised of the spatial orientation of the joint (humeral head
with 30 degrees retroversion and glenoid with 7 degrees of retroversion), the coracohumeral ligament, the
capsule and the glenohumeral ligaments (superior, middle and inferior).
SGHL-Primary role limiting inferior translation in the adducted shoulder. Secondary role in posterior
capsule limiting posterior dislocation when the shoulder is adducted, flexed and internally rotated.
MGHL-Contributes to anterior stability, along with the subscap and superior part of IGHL, when
the arm is abducted 45 degrees however not when at 0 or 90 degrees. It also limits external rotation when
the arm is in the lower ranges of abduction. Has a secondary role in anterior stability with the arm abducted
90 if the anterior band of IGHL is cut. Also, has secondary role limiting inferior translation with the arm
adducted and in external rotation.
IGHL-With an abducted arm in external rotation; the Anterior Band moves forward to stabilize the joint
anteriorly while the Posterior Band moves under the head to stabilize it inferiorly. With an abducted arm in
internal rotation; the Anterior Band moves under the head and the Posterior Band rotates posteriorly to
stabilize the joint posteriorly.

Question 252
Which of the following nerves is most commonly injured during
arthroscopy of the ankle?
1. Sural
2. Saphenous and its branches
3. Posterior tibial and its branches
4. Deep peroneal and its branches
5. Superficial peroneal and its branches

Answer: 5
Neurological complications of account for approximately 49.1% of all complications in Ankle Arthroscopy.
Nerve injuries resulted from direct trauma of portal placement. Nerves affected in order: 1) Superficial
peroneal 2) Sural 3) Saphenous 4) Deep peroneal.
Question 253 -
The primary effect of deep freezing on musculoskeletal allografts is a
reduction of

1. Strength
2. Stiffness
3. Shelf life
4. Antigenicity
5. Risk of HIV transmission

Answer: 4
Deep freezing alters biological and immunological properties. Many investigators have shown that frozen
bone is less immunogenic than fresh bone and freeze-dried is even less. Freezing does increase shelf-life but
has no effect on strength, stiffness, or risk of HIV.
Question 254 -
A physician who is currently in a fellowship program receives notification
of a pending malpractice suit that refers to an incident that occurred 2 years ago
when the physician was a resident. The physician would be insured if the
residency program provided what type of insurance coverage?

1. Indemnification
2. Occurrence
3. Excess liability
4. Claims-made
5. Nose

Answer: 2
Occurrence covers anything that has taken place; indemnification covers the compensation against hurt,
damage, or loss; excess liability covers a specified amount.
Question 255
Which of the following plain radiographic views of the shoulder best
reveals a Hill-Sachs lesion of the humeral head?
1. Lateral Y
2. Scapular AP
3. Neutral rotation AP
4. Internal rotation AP
5. External rotation AP

Answer: 4
Hill-Sachs lesion is an impression fracture of the posterosuperior aspect of the humeral head, produced by
contact with the anteroinferior glenoid when dislocated. Hill-Sachs lesion is demonstrated on plain AP
radiograph in internal rotation.
Question 256 -

Figure 67 shows the AP


radiograph of both knees of a 26-year-old woman. A review of the patient‘s
medical record will most likely reveal a history of

1. Trauma
2. Hemophilia
3. Reiter‘s syndrome
4. Rheumatoid arthritis
5. Systemic lupus erythematosus

Answer: 5
Avascular necrosis of the knee is associated with the use of corticosteroids or alcohol 90% of the time.
Radiographically evident lesions progress until the necrotic cancellous bone collapses away from the
subchondral plate, resulting in the classic crescent sign; earliest sign of mechanical failure of the condyles.
Generally seen in a younger age group (less than 50 years old). Of patients with corticosteroid induced
AVN of the knee; 81% have systemic lupus erythematosus, 9.5% inflammatory bowel disease, and 9.5%
polymyositis.

Question 257

Figure 68 shows the MRI scan of a 13-year-old boy


who has had knee pain and swelling following training lessons for ski racing for
the past 6 months. The only abnormal finding on physical examination is an
effusion. Management should consist of

1. Cast immobilization for 6 weeks


2. Activity modification and re-evaluation in 2 months
3. Internal fixation with or without bone grafting
4. Retrograde drilling of the defect without articular cartilage penetration
5. Drilling of the defect directly through the articular cartilage

Answer: 2
The lesion is osteochondritis dissecans. The primary determinant of treatment is an age of the patient at
presentation. The presence of open physes classifies the lesion as the Juvenile form. It is theorized that, in
both adult and juvenile forms, the articular cartilage softens as it loses the support of the subchondral layer
of bone. If the disease process is not arrested, additional trauma causes separation of a bone fragment, and
a crater remains. Most children who have juvenile osteochondritis dissecans and open physes can be
successfully managed non-operatively. Cahill proposed limitation of activities until the patient was free of
symptoms as well as protected weight bearing with use of splints or crutches. He recommended that non-
operative treatment be abandoned if symptoms persist for 3 months.
Question 258 -
Following closed reduction for the injury shown in Figures 69a and 69b,
treatment should consist of
1. repair or reconstruction of the medial collateral ligament

2. repair or reconstruction of the medial


and lateral collateral ligaments
3. immobilization for 5 days or less
4. immobilization for 14 days
5. immobilization for 25 days

Answer: 3
Repair or reconstruction of the medial collateral ligament-The mechanism of dislocation during a fall on the
outstretched hand would involve the body rotating internally on the elbow, which experiences an external
rotation/valgus moment as it flexes. Posterior dislocations should therefore be reduced in supination. If
valgus stability in pronation is demonstrated, the AMCL can be assumed to be intact, and rehabilitation in a
hinged cast-brace with the elbow in full pronation can be commenced immediately.
Repair or reconstruction of the medial and lateral collateral ligaments-Acute dislocations can be reduced in
supination and tested for valgus stability in pronation. Treatment is determined by the stability following
reduction. When there are fractures, the principle is to fix the bones so that the only limitation is the
ligaments and then to repair them if the elbow is not stable enough to permit early motion.
Immobilization for 14 days-The longer the immobilization had been, the larger the flexion contracture (p
less than 0.001) and the more severe the symptoms of pain were. The results indicate that early active
motion is the key factor in rehabilitation of the elbow after a dislocation. Simple dislocation of the elbow in
the adult. Results after
closed treatment. Immobilization for 25 days- See above.
Question 259
A 3-year old child from an isolated mountain area is evaluated for multiple
medical problems, including vomiting, loss of appetite, polyuria, and failure to
thrive. History reveals the child was normal at birth. The parents, who appear
healthy, are second cousins and have two other children who are normal. The
parents state that they know of another family member who died at age 6 years
after a similar medical history. Radiographs of the lower extremities show
bowing of the long bones with cupping and widening of the physes. What is the
most likely diagnosis?
1. Cystinosis
2. Hypophosphatemia
3. Renal osteodystrophy
4. Primary hyperparathyroidism
5. Nutritional vitamin D deficiency
Answer: 1
Hypophosphatemia-The patients who were symptomatic had obtundation,
hemolytic anemia, rhabdomyolysis, and hepatocellular injury that began during refeeding and resolved
with treatment. The signs and symptoms, pathophysiology, and treatment of refeeding
hypophosphatemia are reviewed.
Renal Osteodystrophy-Musculoskeletal complications in patients with chronic renal failure are common and
may be related to the disease itself or to treatment. The altered metabolism in patients with chronic renal
failure leads to renal osteodystrophy, which consists of osteomalacia and secondary hyperparathyroidism
[1]. Erosive changes attributable to secondary hyperparathyroidism may be easily confused with
rheumatoid arthritis, seronegative spondyloarthropathies, infection, or even malignancy.
Primary Hyperparathyroidism-Preferential involvement of cortical bone with apparent preservation of
cancellous bone in primary hyperparathyroidism was confirmed by percutaneous bone biopsy. Over 80%
of patients had a mean cortical width below the expected mean, whereas cancellous bone volume in over
80% of patients was above the expected mean.
Skeletal disease in primary hyperparathyroidism J Bone Miner Res 1989 Jun; 4(3):283-91
Nutritional vitamin D deficiency-Rickets, osteomalacia, and renal osteodystrophy are disorders of the
mineralization of bone that result from a lack of available calcium, phosphorus, or both. The diseases that
result from numerous mechanisms present with a symptom-sign-radiographic complex with such a high
degree of stereotypy that laboratory investigation is often required to distinguish one form from another.
The disorders in children, known as rickets, produce bowing and other deformities of the long bones and
dwarfism. These disorders are principally related to the profound effect of the deficiency states on the
epiphyseal plate; whereas the same disorders in adults produce an often severe osteopenia and pathologic
fractures. Because of newer developments in our understanding of the factors affecting calcium-
phosphorus
hemostasis and vitamin D metabolism, many of the children and Rickets, osteomalacia, and renal
osteodystrophy.
Question 260 -
A 26 year-old marathon runner reports lateral knee pain after hill training.
Examination reveals no effusion; and results of Ober‘s test are positive. What is
the most likely diagnosis?

1. Lateral meniscus tear


2. Popliteus tenosynovitis
3. Iliotibial band friction syndrome
4. Peroneal nerve entrapment
5. Biceps tendinitis

Answer: 3
Lateral meniscus tear-The accuracy and interexaminer reliability of the McMurray test for the diagnosis of
meniscal tears were compared with arthroscopic findings in a prospective study of 104 consecutive
patients awaiting elective arthroscopy. The only significant McMurray sign found to correlate with meniscal
injury was a thud elicited on the medial joint line with a medial meniscal tear (P = 0.05) that had a fair
interexaminer reliability (kappa = 0.35). The sensitivity of a medial thud was 16%, and the specificity was 98%
with a positive predictive value of 83%. Examiner experience had little effect on the accuracy of diagnosis of
medial meniscal tears. This study supports the continued but limited emphasis on the McMurray test in the
clinical diagnosis of meniscal tears.
Popliteus tenosynovitis-An isolated lesion of the popliteus often presents as a tendon avulsion whereas
major damage to the posterolateral corner of the knee involves combined ligamentous injuries. In patients
with an acute haemarthrosis and lateral
pain in a stable knee, the diagnosis of isolated avulsion of the popliteus tendon should be suspected.
Peroneal nerve entrapment-Peroneal nerve entrapment is one of the less common causes of exercise-
induced leg pain in competitive athletes. This type of lower extremity peripheral nerve dysfunction is usually
associated with activities that subject the nerve to constant compression or repetitive trauma.
Biceps tendonitis-A statistically significant correlation (P = 0.01) was found between increased anterior
translation with the knee at 25 degrees of flexion as demonstrated by the Lachman test and injury to the
biceps-capsuloosseous iliotibial tract confluens. Additionally, adduction laxity at 30 degrees of flexion
correlated with a Segond fracture (P = 0.04). These data establish, in part, the relationship of the biceps
femoris complex injury to anterior translation instability.
The biceps femoris muscle complex at the knee. Its anatomy and injury patterns associated with acute
anterolateral-anteromedial rotatory instability.
Question 261
A patient in the recovery room has weakness of the extensor hallucis
longus and tibialis anterior muscles following a total knee replacement. Initial
management should consist of

1. Observation
2. Removal of the prosthetic components
3. Operative exploration and decompression of the peroneal nerve
4. Nerve conduction velocity studies
5. Loosening of the primary dressings and knee flexion to 30 degrees

Answer: 5
Operative exploration and decompression of the peroneal nerve-Five patients were treated by operative
exploration and decompression of the peroneal nerve for peroneal nerve palsy complicating total knee
arthroplasty (TKA). All patients had failed to demonstrate improvement in the peroneal nerve function
despite extended conservative care. The procedure was performed five to 45 months after the index TKA.
Patients were evaluated and graded preoperatively and postoperatively using the Modified Nerve Palsy
Scale of Weber, Daube, and Coventry. All patients demonstrated improved nerve function. Four of five
patients had full peroneal nerve recovery. All patients were able to discontinue their ankle-foot orthoses.
This is a rare
complication of TKA, and when conservative nonoperative measures do not lead to sufficient improvement
in nerve function, consideration may be given to operative decompression of the peroneal nerve.
Question 262
A 45-year-old woman has a deep soft-tissue mass in the thigh. The MRI
scan Shows a 12-cm heterogenous mass that is deep to the fascia, and the CT
scan Shows three small (5 to 10 mm) peripheral, noncalcified pulmonary nodules.
What is the stage of disease according to the staging system of the
Musculoskeletal Tumor Society?

1. 1
2. 2
3. I
4. II
5. III

Answer: 5
Question 263
In total hip arthroplasty, increasing the perpendicular distance from the
center
line of the femur to the center of rotation of the femoral head (femoral offset)
results in

1. decreased tissue tension


2. decreased abductor lever arm
3. decreased joint reaction force
4. increased body weight over lever arm
5. increased polyethylene wear rate

Answer: 3
The advantages to increasing femoral offset at THA are reported to
include an increased range of motion, better mechanical advantage for the abductors and
decreased instability because of better soft tissue tension. According to Charnley,
increasing the femoral offset should improve the abductor lever arm which should decrease the abductor
force required for walking, and therefore decrease the energy requirement for gait as well as the overall
joint reactive force. The largest possible disadvantage of increasing the femoral offset is increasing the out of
plane bending moment which puts stress on the prosthetic stem. Poly wear is a direct effect of surface area
contact which is not changed with femoral Offset.
Question 264 -
For which of the following conditions will a rehabilitation program for
shoulder instability most likely result in a satisfactory response?

1. recurrent traumatic anterior dislocation


2. recurrent traumatic posterior dislocation
3. traumatic subluxation with no previous dislocation
4. traumatic anterior subluxation
5. atraumatic involuntary subluxation

Answer: 5
In a study by Burkhead and Rockwood, shoulder instability was classified with criteria applying to whether
a patient had traumatic or atraumatic subluxation of the
glenohumeral joint. In this classification Type I is a traumatic subluxation without previous dislocation, Type
II is a traumatic subluxation after previous dislocation, Type IIIA is an atraumatic, voluntary subluxation in
patients with psychological problems, Type IIIB is and atraumatic, voluntary subluxation in a patient without
psychological problems and Type IV is an involuntary subluxation. In their study they found that
shoulders that have traumatic instability (type I or type II) had a 15 per cent chance of a good or excellent
outcome with a rehab program as compared with atraumatic subluxations (type III or type IV) which had
an 83 per cent good to excellent result. Since answer 5 is the only atraumatic type of subluxation it would
statistically stand the best chance for improvement with a rehab program.

Question 265 -
A branch of what nerve is at risk for injury when vigorous superior/medial
retraction is applied to the interval between the teres minor and the infraspinatus
during a posterior approach to the shoulder?

1. radial
2. axillary
3. suprascapular
4. thoracodorsal
5. long thoracic

Answer: 3
In the posterior approach to the shoulder the suprascapular nerve is located in the superior aspect of
operative field coursing through the spinoglenoid notch and on the undersurface of the infra-spinatus
muscle. When the interval between the teres minor muscle and the infraspinatus muscle is retracted
tension is placed on the infraspinatus muscle as well as the suprascapular nerve, which could damage it.
The axillary nerve runs through the operative field, but well below and is not retracted supramedially, so
although there is a risk for injury to the axillary nerve it is not from the retraction superior medially. The radial,
thoracodorsal and long thoracic nerves all lie anterior to the scapula.

Question 266
Figures 70a and 70b show the sagittal MRI scan and axial CT of a patient
who has decreased range of motion in the cervical spine. In which of the
following directions would the cervical motion be most significantly limited?

1. Flexion
2. Extension
3. Axial rotation
4. Left lateral bending
5. Right lateral bending
Answer: 3
MRI and CT demonstrate an abnormality in the alantoaxial region (C1-C2). See chart in reference. ―C1-C2 -
Flexion/Extension 30 degrees - Sidebending 10 degrees - Rotation – 70 degrees‖
Question 267
A 32 yr old man with oxalosis is scheduled for a surgical treatment of spinal
stenosis. Which of the following organs is most likely to show signs of systemic
oxalosis during a preoperative assessment?

1. Skin
2. Lung
3. Brain
4. Heart
5. Kidney

Answer: 5
―Oxalosis is a genetic transmitted, autosomal recessive disorder of glyoxalate metabolism...Nephrolithiasis
and nephrocalcinosis, secondary to calcium oxalate hypersaturation in the patient‘s kidney, usually cause
an initial presentation of renal colic and/or asymptomatic gross hematuria...[and later] chronic renal failure‖
This finding would be detected on either UA or BUN/Cr labs.

Question 268 -

Which of the following lists the correct order of the branches of the
axillary artery from proximal to distal?

1. Thoracoacromial, lateral thoracic, subscapular


2. Thoracoacromial, anterior humeral circumflex, posterior humeral circumflex
3. Posterior humeral circumflex, subscapular, thoracacromial
4. Subscapular, thoracacromial, anterior humeral circumflex
5. Lateral thoracic, anterior humeral circumflex, thoracacromial

Answer: 1
Question 269 -
A patient who has a malignant bone tumor of the hip undergoes
resection and hip arthrodesis. While walking, mean oxygen consumption would
be approximately what percent of that observed in someone who has a normal
hip?

1. 50%
2. 100%
3. 130%
4. 200%
5. 300%

Answer: 3
―Unilateral arthrodesis has been shown to increase O2 consumption in normal
gait by 32%.‖

Question 271 -
Which of the following medicolegal relationships between an attending
surgeon and a resident assistant applies when a patient files a malpractice suit
relating to surgical complications following a total knee arthroplasty?

1. Respondeat superior
2. Indemnity agreement
3. Hold harmless agreement- attempt to shift liability from company to physician
4. Comparative negligence-% of involvement
5. Contributory negligence- resident contributed to the negligence

Answer: 1
―Let the master answer‖. A resident has been authorized to act for or represent the supervising physician.
As an agent for the supervisor, all acts of the resident are considered to be under the direction of the
supervisor. This is the definition for respondeat superior. Hold harmless agreement attempts to shift liability
from company to physician. Comparative negligence refers to the % of involvement. Contributory
negligence is where the resident is accused of contributing to a negligent act.
Question 273 -

The radiographs shown in Figures 71a through 71c, and the CT scan shown in
Figure 71d reveal an acetabular fracture that should be classified as
1. t-type
2. both column
3. transverse
4. anterior column
5. anterior column posterior hemitransverse

Answer: 2
The fracture shown represents a both column fracture described by Letournel and Judet. The fracture is a
combination of a posterior column fracture and an anterior column fracture. T-type, transverse and
hemitransverse all have a transverse element to them. The fracture shown involves more than just the
anterior column.
Note the classic ―Spur Sign‖ seen in these radiographs. This is pathognomonic of a both-column fracture of
the acetabulum.
The other defining feature of the both column fx (as evident by these films) is that there is no intact
acetabulum connected to the bone fragment which is connected to the ipsilateral SI joint.
Question 274 -
What is the most common nerve injury following a Monteggia fracture-
dislocation of the forearm in adults?
1. Posterior interosseous
2. Anterior interosseous
3. Radial
4. Median
5. Ulnar
Answer: 1
According to Rockwood and Green the most commonly injured nerve associated with a Monteggia
fracture is the posterior interosseous nerve, a branch of the radial nerve.
Question 275 -
Which of the following conditions is associated é the highest mortality in patients
é a pelvic fracture?
1. Shock from hypovolemia
2. Associated rupture of the bladder
3. Arterial bleeding on pelvic angiogram
4. Presence of a hematoma in the perineum and scrotum
5. Fractures of both the anterior and posterior pelvic ring

Answer: 1
The highest rate of mortality rate after a pelvic fracture is associated with hypovolemic shock. Most of which
is secondary to extensive intrapelvic arterial injuries.

Potrebbero piacerti anche